12.07.2015 Views

Suggested Answers of BT2 Revision Package - ASKnLearn

Suggested Answers of BT2 Revision Package - ASKnLearn

Suggested Answers of BT2 Revision Package - ASKnLearn

SHOW MORE
SHOW LESS
  • No tags were found...

You also want an ePaper? Increase the reach of your titles

YUMPU automatically turns print PDFs into web optimized ePapers that Google loves.

ST ANDREW’S JUNIOR COLLEGEH1 ECONOMICS (8819)SUSGGESTED ANSWERS OF<strong>BT2</strong> <strong>Revision</strong> <strong>Package</strong> 2012ContentsSection A: Case Studies1. 2010 PJC Prelims H1 Paper CSQ1: Tourism in Singapore2. 2010 AJC Prelims H1 Paper CSQ1: Land Transport in Singapore3. 2010 YJC Prelims H1 Paper CSQ1: Housing Market in Singapore4. 2011 RI (JC) Prelims H1 Paper CSQ1: Rare Earths5. 2010 NYJC Prelims H1 Paper CSQ2: Economic Recovery in Spain and Germany6. 2010 TJC Prelims H1 Paper CSQ2: China and India Trade Growth7. 2010 TPJC Prelims H1 Paper CSQ2: Subprime Crisis in China8. 2010 RI (JC) Prelims H1 Paper CSQ2: Foreign Investment9. 2011 MJC Prelims H1 Paper CSQ2: The Road to Recovery


2010 PJC Prelims H1 Paper CSQ1: Tourism in Singaporea (i) Using Table 2, summarize the trends in tourism receipts. 2MarksGeneral trend1§ Overall increasing trend (tourist receipts rose by 55%)Refinement§ Tourism receipts from India rose the most by 89%§ The lowest increase in tourism receipts were from Japan (1.4%)§ There was a steady increase in tourism for all countries during thisperiod with the only exceptions being Japan in 2006 and the USA in20081a (ii) Explain the possible reasons for the above trends. 2Recall : Total revenue = Price x quantity§Rise in tourism receipts can be due rising number or tourists, or anincrease the prices tourists are willingness to pay for Singaporean goodsand services§The Singapore government has launched various initiatives includingsports events tourism. This has raised the supply <strong>of</strong> tourism. The highpr<strong>of</strong>ile events may have raised awareness <strong>of</strong> the country as a touristdestination leading to increased demand as well. This has led to anincreased number <strong>of</strong> tourist visiting and thus, an increase in tourismrevenue.§Rising incomes have led to increase in demand for normal goods suchas tourism, especially in rapidly growing economies like China and India.This has caused tourist numbers to rise and accounts for the rapid rise intourism receipts.Marking Scheme§§Explanation <strong>of</strong> increase in tourism receipts through a rise in touristnumbers (Quantity) or an increase in expenditureUse <strong>of</strong> data1m1mb (i)Using a demand and supply diagram, explain how hosting the FormulaOne Grand Prix and the imposition <strong>of</strong> a sales tax would affect themarket for hotel accommodation.There would be an increase in the number <strong>of</strong> tourists visiting Singaporeduring the Formula One Grand Prix. This would lead to an increase in the61


demand for hotel accommodation. The demand curve for hotelaccommodation shifts to the rightThe proposed sales tax will lower pr<strong>of</strong>its <strong>of</strong> hotels. Hence, firms will reduceoutput and it will be reflected as a leftward shift <strong>of</strong> the supply curve for hotelaccommodation1Price P1 S1 S0 Figure 1 1P0 Q0 Q1 D0 D1 Quantity <strong>of</strong> hotel accommodation With demand increasing and supply falling, a shortage results so room rateswill definitely increase with those at track side hotels rising the most. (Extract2)Equilibrium quantity is indeterminate as it depends on the extent <strong>of</strong> the shift<strong>of</strong> increase in demand and fall in supply. (theory)But since there are more tourists coming to Singapore, there is evidence thatQ will increase, that is increase in demand is more than the fall in supply.(application)111c (i) What is meant by Price elasticity <strong>of</strong> demand. 2Definition –Price elasticity <strong>of</strong> demand (PED) is the degree <strong>of</strong> responsiveness <strong>of</strong> quantitydemanded <strong>of</strong> a good due to a change in the price <strong>of</strong> the same good, ceterisparibus.1§§It is calculated by taking the percentage change in quantity demandeddivided by the percentage change in price. ORIf PED exceeds 1, a rise in the price <strong>of</strong> the good will cause a more thanproportionate increase in the quantity demanded. Conversely, a PED <strong>of</strong>less than 1 implies that a rise in price will lead to a less thanproportionate increase in quantity demanded.1


c (ii)Using the concept <strong>of</strong> price elasticity <strong>of</strong> demand, explain whyconsumers staying at the track-side hotels are more likely to bear aheavier tax burden compared to those staying further away.Spectators who stay at the track-side hotels will get a very clear view <strong>of</strong> therace. Hence demand will be more price inelastic than that <strong>of</strong> hotels furtheraway, since there are very few substitutes for these hotels by virtue <strong>of</strong> theirsupreme location. (note: another determinant <strong>of</strong> PED that can be used is %<strong>of</strong> income spent as most consumers are high income earners )The tax will cause supply to fall and the supply curve to shift left. Against arelatively price inelastic demand curve, the price increase will be morecompared to that <strong>of</strong> a price elastic demand curve.This greater price increase means more <strong>of</strong> the tax burden has been shiftedto the consumers since quantity demanded will fall less than proportionately3111d (i)Explain how the existence <strong>of</strong> negative externalities results in marketfailure.DefinitionMarket failure occurs when the forces <strong>of</strong> demand and supply fail to allocateresources efficiently. Thus, society’s welfare is not maximized.Negative externalities are spillover costs incurred by third parties that are notinvolved in the production or consumption <strong>of</strong> the good and are incurredwithout compensation.Application to contextThese third parties receive no compensation within the operation <strong>of</strong> themarket. In the case <strong>of</strong> the Formula One (F1), residents near the track will besubject to noise pollution as the race takes place. The noise, louder than ajumbo jet, may cause discomfort and reduce their quality <strong>of</strong> life.ExplanationHence the existence <strong>of</strong> external costs means that Marginal Social Costsexceed Marginal Private Costs.411


Cost/Benefits/$ Welfare loss MSC =MPC+MEC SS=MPC 2mfor aclearexplnDD=MPB=MSB MPC=SS Figure 2 0 Qs Qe F1 races Welfare comparison between Qe and QsWith reference to Figure 1, producers <strong>of</strong> the F1 event ignore the existence <strong>of</strong>external costs and will maximize their pr<strong>of</strong>its at output Qe, whereMPC=MPB. Qe is not the socially optimum level <strong>of</strong> output as MSC>MSBresulting in a welfare loss (shaded area). This welfare loss means thatwelfare at Qe will be less than that at Qs where welfare <strong>of</strong> society ismaximized since MSC=MSB. Hence overproduction occurs at Qe, orresources have been overallocated to the production <strong>of</strong> the F1 races,resulting in market failure.d (ii)Discuss one possible measure to correct negative externalities arisingfrom events like the F1.Traffic congestion is likely to result as the night race is held in the heart <strong>of</strong>town. One possible measure to deal with this negative externality arisingfrom the event would be the use <strong>of</strong> moral suasion to encourage spectators touse public transport instead if private transport. LTA can work with STB toinform the public about road closures, add more train trips and lengthenoperation hours so that the public can access the race area without havingto drive.The public may not be receptive to this as some feel that trains are a poorsubstitute since they are usually crammed and do not provide seamlesstravel like cars will. This may discourage the public from visiting places thatare around the race areas and cause businesses there to be adverselyaffected. The benefit <strong>of</strong> such a measure is that it is relatively easy toimplement and reach the majority <strong>of</strong> the public given the buzz arising fromsuch an event.3


Marking Scheme§§Explanation <strong>of</strong> measure (1m) with application to given context <strong>of</strong> F1 (1m)Limitation <strong>of</strong> this measure (1m)eDiscuss the likely macroeconomic impact <strong>of</strong> promoting sports eventtourism on the Singapore economy.IntroThe likely macroeconomic impact <strong>of</strong> sports event tourism can be measuredin terms <strong>of</strong> achieving the economy’s macroeconomic aims such as lowunemployment, sustained growth and healthy BOP.8BOP§Holding the Formula One would help revive the tourism sector. There willbe an increase in export revenue as they will purchase tickets to theevent, demand accommodation, visit food and beverage outlets. Thetrade balance improves and thus, the Balance <strong>of</strong> Payments deficit, ifany, will be corrected.Unemployment§ Besides increasing net exports, hosting the event, estimated at S$150million, will also involve a rise in investment and government spending.National income rises, via the multiplier effect. The increase in the outputwill also translate into higher derived demand for labour. Cyclicalunemployment falls. The increase in jobs in the tourism sector itself willmean employment for many low-skill workers from declining industriessuch as manufacturing, thereby reducing structural unemployment.Economic Growth§A transfer <strong>of</strong> skills and technology also occurs as a result <strong>of</strong> hostingthe event. E.g. the Formula One imposes strict requirements for theconditions and safety <strong>of</strong> the track and would need to train firms/workersin the domestic industries to meet them. This improves the productivity <strong>of</strong>the local factors <strong>of</strong> production. Aggregate supply increases as a result <strong>of</strong>the increase in productive capacity. Potential growth occurs and this,combined with the increase in AD from tourism receipts etc, will help theSingapore economy grow a new source from which sustainedeconomic growth can depend on.However, a greater dependence on sports tourism can also mean lessstability for the economy as it is extremely vulnerable to external shocks.A global recession would have a larger impact on sports tourism as it is aluxury good (Demand is income elastic and a fall in income will lead to a


more than proportionate fall in quantity demanded). It is also extremelyvulnerable to terrorist and health scares and export revenue can be easilyaffected e.g. SARs or H1N1 epidemics within the region. A fall in sports andthus tourism receipts can lead to a multiple fall in NY via the reversemultiplier. This can lead to higher unemployment and lower standards <strong>of</strong>living as well.Furthermore, a large increase in government expenditures on promotingsports events tourism can strain government budget and entail opportunitycosts in terms <strong>of</strong> cutback in other areas such as education and health care.The large-scale event can also pose a drain on the economy’s resourcesduring this period, crowding out other private investments. This wouldreduce the desired impact on national income and growthConclusionOverall, an investment in sports tourism is likely to bring about significantbenefits for the economy given tourism receipts and reduce unemployment,especially the structural unemployment due to loss <strong>of</strong> low-skill jobs fromdeclining manufacturing industries. However, as other countries in the regiondevelop their own tourist attractions, it may prove a formidable task to staycompetitive. As such, there is a need to <strong>of</strong>fer a comprehensive package bybundling various attractions or even tie up with other countries in the regionto complement and their services as well.Marking SchemeL1 1-3mL2 4-6mL3 7-8mDescriptive answers that lack analysis or reference to data.Lopsided answersBalanced answers that analysis both the costs and benefits <strong>of</strong>holding the F1 but explanation may contain gapsBalanced discussion with analysis and clear reference to data. Aclear stand is taken and justified.


2010 AJC Prelims H1 Paper CSQ1: Land Transport in Singaporea (i) With reference to Figure 1, describe the main features in the generation <strong>of</strong>trips between 1997 and 2008. [2]The number <strong>of</strong> trips grew rapidly with private transport trips growing faster than publictransport trips.(ii) Using Table 1, suggest possible reasons for the features observed. [4]• Increase in population. According to Table 1, Singapore’s population hadgrown from 3.8 million to 4.8 million between 1997 and 2008, a 26 percentincrease. This has led to increase in total ridership.• Economic growth. Real GDP per capita grew by 33 percent between 1997and 2008. Private transport is perceived to be more <strong>of</strong> a luxury good thanpublic transport. With higher income, the increase in demand for privatetransport is greater than the increase in demand for public transport. Hence,this explains why private transport trips grow faster than public transport.Note: 1 reason for the general rising trend, 1 reason for the more rapid increase in private transport trips.(b)With the help <strong>of</strong> a demand and supply diagram, account for the change inthe price <strong>of</strong> Certificate <strong>of</strong> Entitlement (COE) in Extract 2. [6]Price <strong>of</strong> COE fell. In particular it fell by 30% for small cars (Extract 2, para 2).Demand for COEs falls• Fall in demand for cars due to soaring inflation, rise in cost <strong>of</strong> car usage(fuel, parking charges, ERP) (Extract 2 para 2) à demand for COEs als<strong>of</strong>alls.Supply <strong>of</strong> COE is perfectly price inelastic and unchanged• Supply is perfectly price inelastic. The total number <strong>of</strong> COEs allotted isbased on projections <strong>of</strong> how many vehicles will be scrapped in thecurrent year (Extract 3 para 1) <strong>of</strong> which a certain number would bereleased every month for bidding• It is unchanged at the moment because it would only be next month thatthe supply <strong>of</strong> COE be adjusted by the Government (Extract 2 para 7).Diagram to showNote: Supply curve needs to be vertical to reflect the fixed amount <strong>of</strong> COEssupplied (0m for the diagram if the supply curve is upward sloping)Illustrate with diagram.


Price S P 1 P 2 D 1 D 2 Market for COEsQuantity (c) (i) What is meant by price elasticity <strong>of</strong> demand? [2]Price elasticity <strong>of</strong> demand measures the degree <strong>of</strong> responsiveness <strong>of</strong> quantitydemanded <strong>of</strong> a good to a change in its own price, ceteris paribus.It is measured by taking the ratio <strong>of</strong> the percentage change in quantity demanded <strong>of</strong>the good to the percentage change in the price <strong>of</strong> the good itself.(ii) To what extent is the car sellers’ revenue affected by the new COE quota?Thesis: The new COE quota would affect the car sellers’ revenue based on theprice elasticity <strong>of</strong> demand <strong>of</strong> cars[6]In Extract 3, the new COE quota would be nearly 30 per cent smaller, whichmeans a fall in supply <strong>of</strong> COE. This translates to higher prices <strong>of</strong> cars, ceterisparibus.For motor traders <strong>of</strong> smaller cars, a rise in the price <strong>of</strong> cars is likely to reducetheir total revenue. Demand for smaller cars (less than 1600cc) tends to beprice elastic. This is because buyers <strong>of</strong> smaller cars are likely to be lessaffluent. As such, the cost <strong>of</strong> buying a car takes up a much larger proportion <strong>of</strong>their income. Hence, with a rise in the price <strong>of</strong> cars, the quantity demanded forsmaller cars will fall more than proportionately, leading to a fall in total revenuefor the motor traders.For motor traders <strong>of</strong> larger cars, a rise in the price <strong>of</strong> cars is likely to increasetheir total revenue. Demand for larger cars (1600cc and above) tends to beprice inelastic. As buyers <strong>of</strong> such cars are richer, the cost <strong>of</strong> buying a car takes


up a relatively smaller proportion <strong>of</strong> their income. Hence, with a rise in the price<strong>of</strong> cars, the quantity demanded for larger cars will fall less than proportionately,leading to a rise in total revenue for the motor traders instead.Anti-thesis: Car sellers’ revenue can also be impacted by other factorsHowever, there can be other factors, besides the new COE quota (hence highercar prices) that can affect car sellers’ revenue. These are factors that shift thedemand curve and are due to changes in tastes and preferences and incomelevels.With a s<strong>of</strong>tening economy (Extract 2 para 3), the level <strong>of</strong> income falls. Thiswould cause the demand curve to shift to the left hence reducing total revenuefor all the motor traders. However, motor traders <strong>of</strong> smaller cars are likely to beimpacted more as demand from buyers <strong>of</strong> smaller cars is likely to fall relativelymore since this group <strong>of</strong> buyers are the hardest hit.Moreover, there can be changes in tastes and preferences towards driving.With more new ERP gantries and a recent increase in parking charges (Extract2, para 5), people may have shifted their preferences towards public transportinstead. This would reduce the demand for cars and subsequently total revenuefor all the motor traders.EvaluationIn conclusion, the impact on car sellers’ revenue has to be analysed from manyfactors, not just the new COE quota.(d)Evaluate the relative effectiveness <strong>of</strong> the COE system and Electronic RoadPricing in managing traffic congestion in Singapore.Why manage traffic congestion in Singapore? [Brief]• Correct market failure due to congestionExplain how COE and ERP workCOE:• Quota system which limits the growth <strong>of</strong> the car population based onthe number <strong>of</strong> vehicles taken <strong>of</strong>f the road in the preceding six-monthperiod (Extract 3)• Targets at car ownership to reduce traffic congestion• It is a lump-sum payment to increase the fixed cost <strong>of</strong> driving. Itdoes not vary with number <strong>of</strong> trips made.[10]


ERP• Works by charging drivers an amount when they enter congestedareas/roads during peak hours• Targets at car usage as it allows people to own cars but only penalisesthose who enter congested roads• It is a pay-per-use principle to increase the private marginal cost <strong>of</strong>driving• Illustrate with externalities diagram how ERP works to bring the level <strong>of</strong>road usage closer to the socially optimal levelCOE is more effective than ERP in managing traffic congestion• COE directly controls the car population in Singapore. On the otherhand, ERP does not control the car population in Singapore. It merelyreduces traffic congestion in areas with ERP and diverts the problem toother roads along alternative routes not covered by ERP, hence causingtraffic congestions elsewhere.• It is easier to determine the allotment <strong>of</strong> COEs as there is a “formula” t<strong>of</strong>ollow. On the other hand, it is difficult to estimate the extent <strong>of</strong> marginalexternal cost for ERP and thus the tax rate may not be accuratelydetermined.• Relatively lower cost <strong>of</strong> implementing COE system as opposed to highcosts <strong>of</strong> setting up the gantries• As price <strong>of</strong> the COE is <strong>of</strong> a much larger amount, people are more likelyto be deterred from buying a car. On the other hand, once a car isbought, the amount deducted by ERP may seem relatively “insignificant”and people may not feel the pinch. Hence, demand for travel on affectedroads becomes price inelastic after a while as the convenienceoutweighs the ERP charge. This results in a less than proportionate fallin quantity demanded for road usage given a rise in ERP charges.ERP is more effective than COE in managing traffic congestion• COE does not control the usage <strong>of</strong> cars (which is the main cause <strong>of</strong>congestion) as car owners may view COE as a fixed cost and may wishto spread out the cost over more trips. It may encourage more usage. Onthe other hand, ERP is more effective as it tackles the root <strong>of</strong> theproblem which is excessive usage <strong>of</strong> some roads.• Moreover, the determination <strong>of</strong> allotment <strong>of</strong> COE can be flawed. Theprevious system <strong>of</strong> allotting COE had led to an almost 25% increaseinstead <strong>of</strong> the target annual 3% increase in vehicle population in the pastfive years. On the other hand, ERP is a more flexible and responsiveinstrument which can be periodically adjusted down or up depending onthe traffic speed on the roads (Extract 1)EvaluationBoth methods have their strengths and weaknesses. But in controllingcongestion, it would be more effective if we were to target car usage and toreduce the incentive for commuters to travel by car or if they still wish to travel


y car, to choose to go by a different route instead. Moreover, as mentioned inMinister’s speech, ERP is the only one that deals directly with the problem byrequiring individuals to take account the costs <strong>of</strong> congestions.However, this does not mean that these are the only methods to manage trafficcongestions. The government need to supplement these with roadbuilding/widening and better public transport system.Marks Scheme for part (d)Knowledge, Application, Understanding and AnalysisL3 For an answer with well-balanced approach and clear comparison<strong>of</strong> relative effectiveness <strong>of</strong> the two methods with thoroughexplanation. There is good reference to case evidenceL2 For an answer that is balanced with some evidence <strong>of</strong> comparisonbetween the two methods but limited/underdeveloped explanation.There is some reference to case evidenceL1 For an answer that shows an understanding <strong>of</strong> COE and ERP butnot able to explain how they manage traffic congestion. NOcomparison between the two for relative effectiveness.6-8m4-5m1–3mAllow up to 2 additional marks for EvaluationE2 Evaluative comments with justification.E1 Evaluative comments, unexplained.2m1m


2010 YJC Prelims H1 Paper CSQ1: Housing Market in Singapore(a)Describe the changes in house prices in Singapore from 2005 to the end <strong>of</strong>2008.[2]House prices increased at an increasing rate from 2005 to 3 rd qtr <strong>of</strong> 2007 andthen increased at a decreasing rate till 2 nd qtr <strong>of</strong> 2008. From the 2 nd qtr to theend <strong>of</strong> 2008 house prices fell.(b) Explain the likely value <strong>of</strong> the price elasticity <strong>of</strong> supply <strong>of</strong> houses in Singapore. [2]The supply <strong>of</strong> houses is likely to be price inelastic. PES is less than 1. (1mark)With a rise in the house prices due to the increase in demand for houses, thequantity <strong>of</strong> houses supplied is not able to increase readily as new projects takeat least 3 years to complete and new and ready properties may not bereleased by developers who may hold back launches as they expect prices toincrease further in the near future. (1 mark)(c)With the help <strong>of</strong> a diagram, explain how high global fuel and commodity pricesresulted in inflationary pressures in the country.[4]High global fuel and commodity pricesà high cost <strong>of</strong> production sinceSingapore is reliant on imported fuels and commodities in its productionprocess à SRAS falls à SRAS curve shifts up, hence exerting upwardpressure on the general price level à cost push inflation.AD-AS diagram to show cost push inflation is requiredDiagram ---1 markExplanation --- 3 marks(d)Using economic analysis, explain how inflationary pressure and the financialcrisis have affected house prices.[6]


Inflation rate increased due to rising global fuel and commodity pricesà cost<strong>of</strong> production increased à SRAS shifts upThe global financial crisis resulted in world wide fall in income à X fell à ADshifts downThe combined impact <strong>of</strong> the shifts in the AS and AD à economy contracts.This caused workers to be retrenched and unemployment rate rose.The rise in the unemployed numbers would mean that some people would notbe able to meet their mortgage repayments and would have their housesrepossessed à supply <strong>of</strong> houses increased in the market.This increase in supply coupled with low demand as people are less confident<strong>of</strong> their ability to pay up their housing loans or to meet mortgagerequirementsà house prices fell.Explanation <strong>of</strong> inflationary pressure and global financial crisis on the economyand relate to demand for and supply <strong>of</strong> houses --- 4 marksAnalysis <strong>of</strong> changes in demand and supply and the impact on house prices ---2 marks(e)Using the information provided in Extract 1, explain the impact <strong>of</strong> changes inhouse prices on the macro economy.[5]Houses are a form <strong>of</strong> wealth for most people. A change in house prices has asignificant impact on consumer wealth and consumer confidenceà henceconsumer spending is affected.Falling house prices à a decline in wealth à people become less confidentabout borrowing and spending à C falls à AD falls and the economyweakens with national income and employment level falling.Rising house prices due to the economic recovery allowed home owners toremortgage their houses, at lower interest rates, and have cash for spending.Also, home owners would feel richer when house prices soar since houses arethe biggest form <strong>of</strong> wealth for them. Hence C increases stimulating theeconomy à national income and employment level would increase.(f) (i) Explain one measure the Singapore government has taken to boost thehousing market.[3]A relaxation <strong>of</strong> foreign ownership rules on apartments would make iteasier for the significant number <strong>of</strong> expatriates working in Singapore andforeigners to purchase houses à demand for houses increasesà


quantity transacted increases and house prices increase significantlysince supply <strong>of</strong> houses is in general price inelastic.OrAn increase <strong>of</strong> the maximum loan-to-value ratio from 80% to 90% àpeople can obtain more loans à demand for houses increases àquantity transacted increases and house prices increase significantlysince supply <strong>of</strong> houses is price inelastic.OrA reduction <strong>of</strong> cash down payments from 10% to 5% for home purchaseà demand for houses increases à quantity transacted in the marketincreases and house prices increase significantly since supply <strong>of</strong> housesis price inelastic.Max 2 marks if PES is not mentioned.(ii)With reference to the data provided where appropriate, and themicroeconomic goals <strong>of</strong> the government, discuss whether there isjustification for intervention in the housing market.Microeconomic goals <strong>of</strong> the government:-allocative efficiency-equity[8]On attaining allocative efficiencyWithout government intervention, the price <strong>of</strong> houses and the quantity <strong>of</strong>houses transacted in the free market are determined entirely by themarket forces <strong>of</strong> demand and supply. The market-clearing quantity wouldnot be at the socially optimal level as the free market will not take intoconsideration the positive externalities (social cohesion and socialstability) <strong>of</strong> homeownership, resulting in under-consumption and henceallocative inefficiency.Marginal social benefit (MSB) <strong>of</strong> housing exceeds the marginal privatebenefit (MPB) and the quantity transacted in the market is lower than thesocially optimum. Unless there is some form <strong>of</strong> government intervention,there will be under-consumption, resulting in welfare loss to society.[Explain with the help <strong>of</strong> a diagram to show market failure --- underconsumption<strong>of</strong> housing]To ensure a higher level <strong>of</strong> home ownership, house prices must be madeaffordable. All governments are obliged to provide public housing throughdirect intervention in supplying housing and making them affordable to


the lower income group, or through indirect intervention in the form <strong>of</strong>subsidies, tax policies and control <strong>of</strong> land supply.Most governments intervene in the housing market by building andselling new houses/flats and pricing them at affordable rates usually at asignificant discount <strong>of</strong>f the market value <strong>of</strong> similar dwellings. This helpsraise homeownership.To further improve affordability, subsidies like housing grants andsubsidized housing loans, and tax policies like reduction <strong>of</strong> capital gainsand property taxes would make homeownership attractive and henceraise MPB upwards towards MSBà prices fall and the quantitytransacted in the market would increase and welfare loss would then bereduced.Since price <strong>of</strong> public housing is below the market price, quantitydemanded will exceed quantity supplied. There is therefore need toimpose restrictions like ceilings on household incomes for eligibility topurchase and criteria for subsidies in order to better direct the assistanceto the lower income group.In the resale and private property markets, it would be more efficient toleave market forces to determine the quantity and price <strong>of</strong> houses.However, a boom may result in high house prices and cause theformation <strong>of</strong> an asset bubble in the country. Hence, there is a need forthe government to keep watch over house prices and intervene to controlthe rapid increase in prices. Measures such as tightening loans andreleasing more land to build houses would decrease the demand andincrease the supply <strong>of</strong> houses respectively preventing house prices fromsoaring to reach heights which would be beyond what the first-timebuyers can afford. Soaring house prices may lead to overheating in theeconomy and threaten stability.On attaining equityThrough home ownership, people in the country acquire a store <strong>of</strong> valuewhich they can encash in times <strong>of</strong> need. Home ownership also protectsthe lower income group from rising rents in times <strong>of</strong> inflation. This isespecially helpful to the lower income group, providing some form <strong>of</strong>financial security.House value appreciates with the country’s growth, the benefits <strong>of</strong> growthpercolate to the lower income groups as well. This contributes equity andsocial stability.EvaluationWhile government intervention is needed and justified to ensure thathome ownership is at the socially optimal level to maximize society’swelfare and that wealth is distributed to the lower income group as well,there is opportunity cost to be considered. There may be funding problem


with the large amount <strong>of</strong> subsidies <strong>of</strong>fered and the funds would bedirected away from other development projects which can raiseproductive capacity and contribute to the economy’s growth. The rightamount <strong>of</strong> subsidies to increase the consumption <strong>of</strong> housing to theoptimal level is <strong>of</strong>ten difficult to ascertain. There is also the concern thatachieving a high level <strong>of</strong> home ownership might mean that people arecommitting large proportion <strong>of</strong> their savings to housing resulting in thepossibility <strong>of</strong> insufficient liquidity for old age needs.Therefore, regular review <strong>of</strong> government housing policies to ensure thatcosts and benefits <strong>of</strong> government intervention must be appropriatelybalanced. It is important for the government to watch the housing marketand to act judiciously so as not to allow booms and busts in the marketas instability in this market can affect the economy and the variousincome groups.L1L2L3Statements <strong>of</strong> microeconomic goals and descriptiveanswer on either the general reasons for or the measures<strong>of</strong> government intervention without directly addressing thequestionClear explanation <strong>of</strong> market failure and rationale forgovernment intervention to achieve allocative efficiencyand equity. Some attempt to evaluate government actions.Thorough explanation on government intervention in thehousing market to achieve allocative efficiency and equityand evaluation <strong>of</strong> the actions. Answer arrives at a clearand reasoned conclusion.1-34-67-8


2011 RI (JC) Prelims H1 Paper CSQ1: Rare Earthsai) Compare the trend in the prices <strong>of</strong> the rare earths in Chart 1.[2m]Similarity• Generally prices <strong>of</strong> all 3 – rose.Differences• Sharpest increase was seen after Oct 2010• Prices <strong>of</strong> Terbium & Dysprosium were significantly higher than that <strong>of</strong> Neodymiumthroughout the period• Price <strong>of</strong> Terbium was more volatile than those <strong>of</strong> Dysprosium and Neodymium.[1m for similarity in overall trend and 1m for difference in refinement]ii) With the aid <strong>of</strong> a diagram, account for the above trend.[5m]• Demand-side Factors: Rise in Demand for final goodo growing global affluence à higher demand for consumer durables e.g plasmaTV, hand-phones which use rare earths in their production.o Development <strong>of</strong> new products which requires rare earths: “Global demand forrare earth metals has more than doubled over the past decade, as ever moredevices (e.g hybrid cars, plasma T.V) make use <strong>of</strong> their unique propertiesàDerived dd riseso Price inelastic demand for rare earth à “There is no viable rare earthsubstitute. Other metals have been tested, but they don’t work. Scientistshave had some success in reducing the amount <strong>of</strong> rare earths needed inproducts, but not enough.” (Extract 2, last para)• Supply-side Factors: Fall in Market Supplyo China’s reduction <strong>of</strong> exportsàOverall, prices for all 3 types <strong>of</strong> rare earths riseCandidates must address the higher price <strong>of</strong> dysprosium & terbium and lower price <strong>of</strong>neodymium.‘Some rare earth elements such as … neodymium are more abundant and concentrated’,‘dysprosium and terbium are less plentiful in world’s mining deposits’ (Extract 3, para 2)è Lower SS for Dysprosium and Terbium relative to Neodymium. For any given demand <strong>of</strong>Dysprosium and Terbium, price will higher.Price inelastic supply – more so for Terbium and Dysprosium (‘harder to extract than theothers’)à contributes to higher P than NeodymiumMarks Scheme• Only Demand-side or Supply-side factors w.r.t extracts: Max 3m• Did not explain the ‘difference’ in trend from part (a): Max 3m• No comment on elasticity <strong>of</strong> supply, max 4m


(b) With the help <strong>of</strong> a diagram, analyze the effects <strong>of</strong> China’s increasing control over rareearths mines on the Japanese economy.[6m]- Japan manufactures many <strong>of</strong> the products like wind turbines, solar panels, plasmatelevisions, mobile phones and hybrid car batteries which require rare earths as raw inputs.- is the “world’s biggest importer <strong>of</strong> these metals” (highly dependent on these raw materialsfrom China)Effects <strong>of</strong> China’s increasing control over rare earths minesAS effect (show diagram)- Rising COP with increase in prices <strong>of</strong> rare earth due to cut in China’s exports <strong>of</strong> rareearth- rare earth is essential rare materials in production <strong>of</strong> many goods in Japan- decrease in AS (upward shift in AS curve)- decline in real national income à decrease in productionà decrease in employmentas labour is derived demand, especially in the related industries.- Higher GPL as result <strong>of</strong> decrease in AS à affect export competitiveness <strong>of</strong> exportsAD effect (show effect on the above diagram)- Outflow <strong>of</strong> Chinese FDI from Japan (extract 1: “…. Chinese monopoly could shift thehigh-tech manufacturing industry bases from Japan to China”)- Decline in I à decrease in AD- Fall in real national income by more than proportionate via the k effect- Rise in unemployment as domestic workers working in the affected high-techmanufacturing industries lose their jobs given the shift from JapanMarks Scheme:Identification and explanation effect on AD – 2mIdentification and explanation <strong>of</strong> effect on AS – 2mAccurate diagram with sound explanation – 2mLack depth <strong>of</strong> explanation – lifting <strong>of</strong> points without explanation – Max 3mFocus only on AD or AS effect with accurate diagram and sound explanation – Max 3mNo reference to case material – Max 3m


(c) To what extent would China’s reduction in the export <strong>of</strong> rare earths pose a ‘challenge tothe West’s march toward a greener future’?[8m]China’s reduction in export <strong>of</strong> rare earths àdecrease in global supply <strong>of</strong> rare earth metals àincrease in prices <strong>of</strong> rare earthsà impede the development <strong>of</strong> green technology that othercountries are currently into.Thesis: ChallengeCOP for energy-saving devices eg solar panel, wind turbine riseCOP for hybrid cars riseè Translate into higher Ps è Less incentive for people to purchase these greendevicesð Pollution from use <strong>of</strong> fossil fuel resulting in negative externalities would not bereduced to optimal level.o Use <strong>of</strong> more pollutive methods <strong>of</strong> production, power generation as well as(normal) cars would not be reduced to socially desired level.External costs still remain => Still consume above the socially optimal level <strong>of</strong> output.(Apply MSC=MSB framework)• Eg: With the export cuts in rare earths causing a rise in price <strong>of</strong> hybrid cars, MPB (DD)for ‘conventional’ cars will not be able to fall sufficiently to reduce consumption <strong>of</strong> suchcars to socially optimal in the market for ‘conventional’ cars.• Alternatively, students can explain with respect to the usage <strong>of</strong> hybrid cars, greentechnology such as solar panels and relate to positive externality from using suchdevicesWith a rise in cost <strong>of</strong> production <strong>of</strong> green technology such as hybrid cars, MPC would notfall/ MPC would increase due to increased price <strong>of</strong> rare earths to sufficiently reach Qse inthe market for hybrid cars.Antithesis: May not be a challengev West may consider importing green tech from China.Evidence:o ‘China could emerge as the unquestioned global leader in clean energyproduction … by exporting these clean-energy technologies to the world’(Extract 3, last para)o ‘Rare Earth metals are needed for the manufacturing <strong>of</strong> wind turbines, solarpanels…hybrid car batteries meaning the Chinese monopoly could shift thehigh-tech manufacturing industry bases from Japan and Korea to China.’Extract 1 para 3With possession <strong>of</strong> most <strong>of</strong> the world’s Rare Earth, China gained CA in production <strong>of</strong>Rare earth and the goods made from Rare Earth over other countries. China alsogained CA in the extraction <strong>of</strong> rare earth being the “leader in the field <strong>of</strong> extraction”thus far. Hence China might be able to manufacture green tech at a lower cost thanother countriesà can “export these clean-energy technologies to the world” at a relativelylow prices given the lower opportunity cost in production


Hence as long as the West is willing to import from China, the costs may not escalateto a large extent.West may be prompted to devote more expenditure into R&D to find more sources <strong>of</strong> rareearth, to develop extraction and refining tech so as to cut dependence on ChinaHowever, efforts to find new mines- take long time:In SR - PES


Costs/BenefitsMSCMPCabMECcMPB= MSB0Q*QpQuantitySince rare earth extraction factories pursue their self-interest and seek to maximise pr<strong>of</strong>its,they only take into account private costs and private benefits and produce to the point whereMPC = MPB at Qp. The socially optimum level, however, occurs at Q* where MSB = MSC.Thus, the free market leads to an over-extraction <strong>of</strong> rare earths by the amount Qp - Q*. Atthe private output Qp, MSC>MSB and society will be better <strong>of</strong>f if one less unit is produced.For all the additional units produced in excess <strong>of</strong> Q*, the welfare loss is represented bytriangle abc. Since society’s welfare is not maximised, there is market failure, defined by thefree market failing to allocate resources optimally and efficiently, hence need for govtinterventionExplain how taxes work to “internalise the negative externality”:Taxation allows the government to charge a tax equivalent to the value <strong>of</strong> the MEC on therare earth extraction factories such that they take into account <strong>of</strong> the negative spillovereffects as result <strong>of</strong> their extractions. By increasing the MPC to MSC, the producers willinternalize the negative externality such that the socially optimal level <strong>of</strong> emission isachieved at Q*. In addition, the tax also provides revenues which may be used to counteractthe negative effects <strong>of</strong> the rare earth extractions.A key problem with the tax is that <strong>of</strong> determining the MEC and then charging the amount <strong>of</strong>tax equivalent to it. An over-valuation <strong>of</strong> external cost means that output is below socialoptimal and the society’s welfare is reduced. An under-valuation <strong>of</strong> external cost implies thatalthough output is lowered by the tax, it is not enough to bring output to the socially optimallevel. With the lack <strong>of</strong> precision, society’s welfare cannot be maximized.Evaluation:- Tax policy necessary but may not be sufficient.- Suggest alternative policy/policies to complement tax policy


Marks Scheme:L3( 7 – 8m)L2(4 – 6m)L1(1 -3m)Evaluation(1m)Evaluation(2m)- Clear use <strong>of</strong> economics framework- Sufficient depth/ scopeo Clear explanation <strong>of</strong> the type <strong>of</strong> market failureo Clear explanation <strong>of</strong> the tax policyo Evaluation <strong>of</strong> the tax policy- Apt reference to case evidence- Clear explanation <strong>of</strong> how the tax policy works and itseffectiveness- Insufficient depth/scope- Some application to context/case- Max 5m without explanation <strong>of</strong> the type <strong>of</strong> market failure- Lack economics framework in analysis especially in explanation<strong>of</strong> the type <strong>of</strong> market failure and the working <strong>of</strong> the tax policy- No application to context/case- Unexplained assessment made, merely stating synthesis- Evaluative assessment supported by economic analysis, i.e.suggestion <strong>of</strong> alternative policies and the need for them.


2010 NYJC Prelims H1 Paper CSQ2: Economic Recovery in Spain and Germany(ai) What happened to the general price level in Spain in 2009? [1]The GPL in Spain was decreasing in 2009 (Deflation).(aii) Using Tables 3 and 4, explain which country performed the worst in 2009. [3]Spain [1m]Unemployment rate was the highest, [1m]Current account deficit as a percentage <strong>of</strong> GDP is the highest amongst thecountries. (higher than France whereas Germany had a surplus.) [1m](bi) Compare the trends in labour productivity for Spain and Germany between 2007and 2009. [2]• Labour productivity --Rising ,(Spain ), Falling (Germany) [1m]• Difference in the rate <strong>of</strong> change more pronounced for Germany(decreased by a large percentage from a positive 1% to - 5%.) compared toSpain (increased from around 0.2% to 3 %) [1m](bii)How far does the data in Figure 3 help to explain the changes in the current accountbalance for Spain from 2007 to 2009?[4]YES --- The current account balance <strong>of</strong> Spain has improved between2007-2009, (the deficit is decreasing). Figure 3 shows that labour productivity<strong>of</strong> Spain is rising during this period, this implies a positive relationshipbetween current account betterment & labour productivity. This may bebecause with better labour productivity the cost <strong>of</strong> production decreases,thus helping Spain to make her exports more price competitive & improvingher current account position. [2m]NO —limited information in Fig 3 which only shows overall labour productivitynot the productivity in different sectors (especially export sector).-- Data also shows Current account figures as a percentage <strong>of</strong> GDP, so


conclusion about absolute value <strong>of</strong> current account cannot be made.-- Other reasons that affect current account, e.g. exchange ratemovements,Income changes in other countries, etc. [2m](c)Explain how Europe will be affected by the rising oil and food prices faced by theemerging economies. [4]The rising oil & food prices à higher import expenditure for the emergingeconomies. (Pm increases, PEDm


• If EU is using Chinese raw materials then retaliation can increase their cost <strong>of</strong>production inviting inflationary situation & also harm growth <strong>of</strong> the country.• Trade restrictions also have high opportunity costs; -it gives rise toinefficiency in resource allocation and hence increases welfare loss for thesociety.• Poorer innovation and quality can result in exports itself losingcompetitiveness not from price alone and worsen trade deficit. [2m]Do not agree: arguments in favour <strong>of</strong> protectionism:• to protect EU’s goods against dumping,• protecting domestic consumers from counterfeit, low quality goods,• increasing domestic employment to aid certain industries in times <strong>of</strong> economicslowdown,• actions taken as a retaliatory measure against China’s policy <strong>of</strong> followingprotectionist measures like obstacles faced when doing business in China.• for improving the balance <strong>of</strong> payments deficit. [2m]EvaluationAcceptable economic justifications for applying trade restrictions as a temporarymeasure but not a long term measure because cost <strong>of</strong> restrictive trade practicesmay be too high. [1m](e)Discuss the view that Spain should follow the measures taken by Germany toaddress its macroeconomic problems.[10]Problems faced by Spain• Increasing UN as seen from Table 3 (18% in 2009 & 19.5 % in Feb 2010)• Reduced training incentives by employers because <strong>of</strong> temporary workcontracts holding back productivity (Extract 6)• Rigid & complex system for setting wages• Wage growth not keeping up with productivity growth• Losing competitiveness• Consumers being weighed down by huge mortgage debtsAGREE: Follow Germany’s Policy• Germany’s UN rate is falling over the years (7.4% in 2009)-policy followed wasshort time working scheme & flexible hours which helped the workers whosehours were cut to quickly get back into full time work,--however may work inthe SR but may not work in the LR.


• Germany’s revival has been built largely on exports & also maintaining goodquality products. (Extract 7) & also diversification <strong>of</strong> export markets to newerregions like Asia & Latin America however stiff competition from other EUcountries may make it difficult .• Germany’s CA in engineering goods helped to cater to the changingrequirement’s <strong>of</strong> the world market –however such reforms only take effect inthe LR & depend on the other countries’ willingness.DISAGREE: Besides following Germany to solve some <strong>of</strong> their problems,Spain must• Use expansionary FP (for more job creation) and & LR SS side policy (toundertake economic restructuring to keep up with international market trends,move into areas with potential CA, engage in retraining & upgrading <strong>of</strong> labourskills to increase productivity etc). ---however undertaking FP will cause morestrain to Spain’s Govt budget & SS side policy changes takes a long time tobecome successful.• Spain must also focus on consolidating the domestic market by increasingGovt exp for e.g. by direct financial assistance to help the people deal withmortgage debts —added strain on the Govt budget, may prove to be a highburden to the society in the LR.Evaluation: Spain can follow the measures taken by Germany to overcome some <strong>of</strong>its macroeconomic problems, but which policies should be followed and the extent <strong>of</strong>the success <strong>of</strong> the policies depend on the problems faced by the countries and on thenature <strong>of</strong> the economy (value <strong>of</strong> the multiplier, mindset <strong>of</strong> the people, tradingcountries co-operation etc)So, Spain should cater to its most significant problems by simultaneously usingdifferent policies.LevelMark DescriptorE 1-2 Students have to come to a reasoned conclusion/judgment.L3 7-8 Discusses the advantages and disadvantages to Spain whenit follows Germany’s measure with alternative solutions.L2 4-6 Discusses the advantages and disadvantages to Spain whenit follows measures taken by Germany with reference to theproblems faced by Spain.L1 1-3 Sketchy answer that just lists or briefly explains themeasures without evaluation or without linking to theproblems faced by Spain.


2010 TJC Prelims H1 Paper CSQ2: China and India Trade Growth(a) (i) Compare the change in productivity between China and India. [2]As shown in Figure 1, productivity in both China and India have increasedoverall. However, China was achieving a higher growth rate than Indiathroughout the given period.1 mark for a similarity.1 mark for a difference.(ii) Suggest one policy to raise labour productivity. [2]Labour productivity is usually measured by the amount <strong>of</strong> output per manhour.Labour productivity could be raised through higher education and life-longtraining. Hence, the government could help by subsidizing education andtraining.orLabour productivity could be raised through having more and/or bettercapital equipment to work with. Hence, the government could encourageinvestment by giving tax incentives.Full 2 mark only when candidates state the role <strong>of</strong> government.(b)Using the theory <strong>of</strong> comparative advantage, account for the change inChina’s exports.A country has a comparative advantage over another in the production <strong>of</strong> agood if it can produce it at a lower opportunity cost. Since having acomparative advantage would allow the country to sell the good at a lowerprice, left to the market forces alone and assuming no market imperfections,a country should find itself exporting goods in which it has a comparativeadvantage and importing those in which it has a comparative disadvantage.Comparative advantage may shift over time when cost conditions changeand/or when pattern <strong>of</strong> demand changes. Extract 4 stated that there is achange in China’s exports from mainly apparel and oil products which aremore labour-intensive to electronics and information technology productswhich are more capital intensive. This suggests that while China facessome erosion <strong>of</strong> comparative advantage in the former, it is becomingrelatively more efficient or gaining a comparative advantage in the latter.This is due to market forces at work and could be analyzed in terms <strong>of</strong> thepush and the pull factors. With rising demand for labour, wages are fast[4]


ising making labour relatively more expensive than capital, coupled withincreasingly well-trained and well-educated labour force and rapid capitalaccumulation, the economy is pushed towards becoming a major exporter<strong>of</strong> electronics and information technology products. The pull factor would behigher value-add and hence returns associated with more capital-intensiveproduction.1 mark for meaning <strong>of</strong> comparative advantage1 mark for linking comparative advantage to exportsFinal 2 marks for applying to the case <strong>of</strong> China here.(c) Assess the impact <strong>of</strong> China’s growth on ASEAN economies. [6]China’s growth has both positive and negative impact on ASEANeconomies as raised in Extract 5.China’s growth is likely to bring economic prosperity to ASEANeconomies. China’s rising demand for exports from ASEAN economiescould help stimulate the economies with rising employment and promoteeconomic growth in the long run. This is further supported by the foreignexchange earnings through exports which allow ASEAN to importrelevant capital goods for economic development and growth.However, with a comparative advantage mainly in its cheap labour andvast land, more than half <strong>of</strong> China’s exports come from the processingand assembly <strong>of</strong> imported parts and components. This has driven upmany key commodities prices and created worrying inflationarypressures world-wide. While ASEAN members who are net importers <strong>of</strong>key inputs such as Brunei in oil and Malaysia and Indonesia in palm oilbenefit, others face risk <strong>of</strong> stagflation.China started <strong>of</strong>f as a major competitor for foreign direct investment withmany ASEAN members and it has clearly outperformed ASEAN with itsabundant labour and land, to the detriment <strong>of</strong> ASEAN’s economicdevelopment plan. However, China in recent years, especially with itsvast foreign reserves, has started looking outwards including ASEAN toinvest which would help promote ASEAN’s economic growth.Notwithstanding many concerns over the fast rising China economy,international trade based on true comparative advantage should not bea zero sum game. E.g. Singapore though experienced loss <strong>of</strong>comparative advantage to China in labour-intensive productions hasbeen successful in exporting its expertise and services to China. WhileChina may appear to benefit more given its faster economic growth,availability <strong>of</strong> the relatively low cost products from China has also gonea long way to help lift ASEAN’s living standard and raise its productivecapacity. A more pressing issue, perhaps, would be on China


government’s co-operation and ability to contain its pollution problemsespecially when these problems are trans-boundary.L3A reasoned explanation in concluding the likely effectwith respect to the case materials.5-6L2 For a balanced answer with respect to themacroeconomic objectives.L1 For a one-sided discussion with respect to themacroeconomic objectives.3-41-2Maximum 1 mark for an incomplete analysis that doesnot relate to the macroeconomic objectives.P.S. ASEAN members consist <strong>of</strong> Singapore, Indonesia, Malaysia,Thailand and Philippines, and subsequently Brunei, Cambodia, Laos,Myanmar and Vietnam.(d) (i) Explain real effective exchange rate. [3]An exchange rate is simply the price <strong>of</strong> one currency in terms <strong>of</strong> anothercurrency. E.g. US$1=S$1.38. This is called the nominal rate <strong>of</strong>exchange.An effective exchange rate is a trade weighted exchange rate, ameasurement in index form <strong>of</strong> changes in the value <strong>of</strong> a country’scurrency against a basket <strong>of</strong> other currencies weighted according to therelative importance in trading terms for the country <strong>of</strong> each <strong>of</strong> the othercurrencies.The real effective exchange rate is the rate at which a person can tradegoods and services <strong>of</strong> one country for the goods and services <strong>of</strong> othercountries. It is the effective exchange rate index adjusted for rate <strong>of</strong>inflation for the country’s exports and the weighted average inflation rate<strong>of</strong> imports it buys from other countries.1 mark only for definition <strong>of</strong> nominal exchange rate.Up to 2 mark for meaning <strong>of</strong> effective exchange rate.The last 1 mark for meaning <strong>of</strong> real effective exchange rate.


(ii)With reference to the data, assess whether the rising rupee poses athreat to India’s exports.Thesis - theoretical analysis <strong>of</strong> likely fall in Xs given a stronger currencyCeteris paribus, a rising rupee would lead to a higher foreign currencyprice <strong>of</strong> India’s exports which would dampen the demand for India’sexports. This concern about India’s export competitiveness was raisedin Extract 6 ‘…pressures on competitiveness likely to arise from anyemergent currency appreciation pressures.’Anthesis – however, Xs did not fallShown in Fig. 2• As shown in Fig.3 and also stated in Extract 6, overall the rupeeappreciated since 1992 (mildly by about 3%) while India’s share inworld goods exports has roughly doubled. Moreover, the trendpersisted over a long 15-year period with a higher rate <strong>of</strong>improvement in India’s share <strong>of</strong> world goods exports.India’s non-price competitiveness or rising rupee due to better exportperformance• In addition to the exchange rate, there are likely other factorsinfluencing India’s export competitiveness.• While a stronger rupee may erode India’s price competitiveness, Indiacould have improved its non-price competitiveness in terms <strong>of</strong> qualityor product image. India’s comparative advantage in highly-skilledlabour could also be moving up in value-addedness towards high-techindustries in information technology. In other words, India’s exportperformance could well be the reason behind the rising rupee asIndia’s exports become more competitive and enjoy rising demand.• This is implied in Extract 6 stating ‘the need for ‘continued’ efforts tomaintain strong productivity growth to cope with any pressures oncompetitiveness likely to arise from rising rupee’. To the extent thatIndia is able to maintain its strong productivity growth to support therising rupee, erosion <strong>of</strong> its exports competitiveness from a risingrupee is limited.Difference between nominal and real effective exchange rate• In Extract 6, the concerns raised about competitiveness due tosignificant appreciation <strong>of</strong> rupee during 2007 ‘In nominal terms… particularly notable… ‘ was clearly referring to theappreciation in nominal terms.• Hence, the comment that export performance remainedfavourable despite the significant rupee appreciation could bebecause <strong>of</strong> a lower inflation rate <strong>of</strong> India’s exports relative to theweighted average inflation rate <strong>of</strong> imports it buys from othercountries. In other words, India’s real exchange rate <strong>of</strong> rupeecould have fallen.[5]


L2L1A balanced analysis <strong>of</strong> likely factors other than a risingrupee affecting India’s export. Application to the casematerial is expected for the full 5 marks.For a one-sided explanation <strong>of</strong> how India’s exports mayfall from a rising rupee.No mark for mere definition <strong>of</strong> exchange rate andexport. (Exports should have been defined in (b) whileexchange rate in d(i).)3-51-2(e)Comment on India’s imposition <strong>of</strong> tariffs on China’s export <strong>of</strong>power generators.Tariff is a tax on imports. It is usually considered an act <strong>of</strong> protectionismas the main objective is to protect domestic industry and employment bymaking imports more expensive and the domestic substitutes relativelycheaper. There are various reasons for protectionism. While some arejustified, others are not.Extract 7 has shown a clear case for the imposition <strong>of</strong> tariff on China’sexport <strong>of</strong> power equipment. The ASSOCHAM has claimed that it was acase <strong>of</strong> unfair competition from China as China exempts the productsfrom internal duties while India continues to subject their products toduties and taxes ‘to an extent <strong>of</strong> nearly 6% <strong>of</strong> the equipment cost’. Thisgives China an artificial comparative advantage, making India’s powerplants relatively more expensive and unable to compete with importsfrom China, leading to loss <strong>of</strong> employment. However, there may be anacceptable reason for the tariff imposition provided the tariff rate doesnot exceed 6%. It may be argued that a more appropriate measurewould be for India to also exempt their power plants from their internalduties and taxes. Unfortunately, China has also been widely condemnedfor keeping its currency undervalued to lend support to its exporters.This is mainly based on its relatively stable exchange rate despite itslarge and persistent balance <strong>of</strong> payments surplus and the associatedfast accumulating reserves. While this is giving China an unfairadvantage in its exports, the accusation has come more mainly from thedeveloped world <strong>of</strong> the USA and Europe.China could well be having a true comparative advantage in theproducts as its higher productivity growth as shown in Figure 2 couldmean it has become more productive than India and able to attractbuyers in India against India’s domestic products with lower pricesand/or better quality. However, given that strategic importance <strong>of</strong> powerplants to an economy, the tariff imposition may be justified.[8]


L3L2L1EA balanced discussion <strong>of</strong> the justification for India’stariff imposition on China’s export.For a one-sided discussion on whether the tariff isjustified or not.For a theoretical explanation <strong>of</strong> the objective and effect<strong>of</strong> tariff. Maximum 1 mark for a mere definition <strong>of</strong> tariffand export.A well developed evaluative judgment on India’simposition <strong>of</strong> tariff on China’s exports with respect tothe case materials.5-63-41-21-2


2010 TPJC Prelims H1 Paper CSQ2: Subprime Crisis in ChinaCase Study Q2 <strong>Suggested</strong> <strong>Answers</strong>(a) (i)With reference to Table 2, summarize the trend in China’s Balance<strong>of</strong> Trade for the given period.China’s BOT surplus [1] has increased [1] throughout the given period.Additional point to consider: By the end <strong>of</strong> the period, it has increasedby 682.3%.No mark if students wrote “BOT increased” without stating that thesurplus has increased.[2](ii) Describe what happened to the Chinese yuan for the given period. [1]Yuan appreciated against the US$ for the given period.(iii)To what extent does the data support the theoretical relationshipbetween changes in the exchange rate and its impact on thebalance <strong>of</strong> trade?In theory when the exchange rate appreciates (depreciates), exportswill become more (less) expensive while imports will become cheaper(dearer) and if Marshall-Lerner condition holds, BOT will worsen(improve). [1]From Table 2, we observe that when yuan appreciates against US$,BOT surplus increases. As a result, the data does not support thetheoretical relationship between changes in the exchange rate and itsimpact on the BOT. [1]However, (1) the exchange rate data provided is incomplete as we onlyhave exchange rate <strong>of</strong> yuan against US dollar and there is noinformation on the exchange rates <strong>of</strong> yuan against other currencies. OR(2) the Marshall-Lerner condition may not hold. Therefore the data doesnot support the theoretical relationship to a limited extent. [1][3]


(b)Explain how a rise in interest rate and the bursting <strong>of</strong> the UShousing bubble resulted in the economic crisis for US.The economic crisis in US is caused by subprime mortgage crisis thatresulted in lower national income in US. The subprime mortgage crisisitself is caused by higher interest rates and the bursting <strong>of</strong> the UShousing bubble as stated in Extract 4.As mentioned in Extract 4, the US central bank has increased theinterest rates from 1% to 5.25%. A rise in the interest rate will increasethe cost <strong>of</strong> borrowing as the interest cost will increase for a givenamount borrowed. Therefore consumption and investment will fall asthey are negatively related to interest rate and AD will fall resulting inlower national income in US. [1]Bursting <strong>of</strong> the US housing bubble caused prices <strong>of</strong> housing to fall.Furthermore, the increasing defaults and foreclosures have led to areduced supply <strong>of</strong> new loan as US housing mortgage companies areless willing and able to lend. Extract 4 pointed out that many housingmortgage companies are forced to shutdown. These have two effectson the US economy. First, with a fall in housing prices the wealth <strong>of</strong>house owners will decrease and hence lowering autonomousconsumption. Second, there will be less investment on housing as lessnew loans are <strong>of</strong>fered. AD will fall due to a fall in C and I henceresulting in lower national income in US. [2]Cap at [2] if there is no supporting evidence from the case orexplanation is provided only for 1 factor.[3](c)With the aid <strong>of</strong> a diagram, explain how the subprime crisis mayaffect China’s economy in the long-term.The subprime crisis would affect China’s economy in the long-termthrough its effect on trade.As stated in Extract 4, if the subprime crisis exacerbates and results insevere economic downturn in US and EU, consumption spending willfall and hence imports from China will fall. This will lead to a fall in Xand hence decreases the AD <strong>of</strong> China in the longer term. Inventorieswill increase and lead to a fall in production, output, employment andnational income through the multiplier. This can be seen in the AD-ASdiagram below.[5]


General Price levelASAD 1 AD 0P 0P 1Real national incomeY 1 Y 0Real national income will fall from Y 0 to Y 1 and with the fall in realoutput, unemployment rate will also increase.The subprime crisis may also affect China’s BOT and BOP. A fall in X,ceteris paribus, will lead to a lower (X – M) and hence worseningChina’s BOT. FDI into China may also decrease if US and EU go intoeconomic downturn as firm are less willing (majority <strong>of</strong> FDI will go intoventures that produce goods for export) and able (due to lowering <strong>of</strong>firms’ pr<strong>of</strong>its and ability to get loans from banks), worsening the capitalaccount <strong>of</strong> BOP. Hence BOT and BOP <strong>of</strong> China may be affectedadversely, with the surplus being reduced.(The fall in FDI may also lead to a fall in the LRAS but this is unlikely asthe subprime crisis is not a long term crisis that will decrease theattractiveness <strong>of</strong> investing in China for the foreseeable future. Morelikely for LRAS to increase at a slower pace. [Cap at L1 if students onlyprovide this as answer.])L1L2Able to draw the correct AD-AS diagram / Able toprovide brief theoretical explanation <strong>of</strong> possibleimpacts on China’s economy.Able to provide detailed explanation on both internaland external impacts on China’s economy withcorrectly drawn diagram.Cap at [4] if diagram is not drawn or not correctlydrawn.Cap at [4] if both internal and external impacts notanalyzed.1 – 23 – 5


(d)Consider whether the evidence in Table 2 would lead you tobelieve that China has a healthy economy in 2008.A healthy economy is an economy with high growth, lowunemployment, stable prices, balanced BOP and stable exchange rate.There is high actual growth as can be seen from the GDP and CPI dataand there is also high level <strong>of</strong> investment, indicating that firms ingeneral have a positive outlook for the Chinese economy. Potentialgrowth should be high as LRAS would be increased significantly due tothe high level <strong>of</strong> investment.Inflation is also very low, indicating that in spite <strong>of</strong> the high growthChina’s economy is actually not over-heating. The stability in prices canencourage investments, improving the health <strong>of</strong> the economy.Registered unemployment rate in urban areas is also relatively low andstable, an indication <strong>of</strong> a healthy economy.There is also persistent and increasing surplus in the BOT. Thisindicates strong and increasing export competitiveness <strong>of</strong> industries.China’s industries are gaining CA and this would further encourageFDI, promoting economic growth.Exchange rate has strengthened compared with the previous year. Asthe appreciation is due to excess demand for yuan due to the strongexport performance <strong>of</strong> China, the economy is healthy.[8]But there are inadequacies in the data provided in Table 2 for acomplete assessment <strong>of</strong> the economy. First, GDP data does not informus about the distribution <strong>of</strong> income. Income distribution can be unevenand inequitable with urban cities getting a disproportionate large share<strong>of</strong> national income. Data should include the Gini coefficient for thewhole <strong>of</strong> China. Furthermore, we are not given any data on the size <strong>of</strong>the population. A better indicator <strong>of</strong> a healthy economy is GDP percapita. Second, we do not have the overall unemployment rate inChina. This can hide a high unemployment rate in the rural areas aspeople that are unemployed could return to their home in the ruralareas. Third, we are also not provided with BOP figures. Performancein the financial and capital account is not known.Hence data in Table 2 clearly indicates that China has a strong andhealthy economy in 2008 and she is not suffering from the US subprimecrisis. While ideally we need more complete information such as Ginicoefficient, GDP per capita and BOP for a more complete assessment,the given data is sufficient for us to conclude that China’s economy ishealthy. This is because China being a socialist country has programs


that look into redistributing income and in providing social safety nets.Population growth is also very unlikely to overpower real GDP growth tothe extent <strong>of</strong> not providing China with a high real growth in GDP percapita. BOP should also be healthy as China is accumulating foreignreserves and its currency is appreciating.L1L2E1Able to list the 5 indications <strong>of</strong> a healthy economybut with little ability to explain why they indicate ahealthy economy.Able to explain why China’s economy is healthyusing data from Table 2 & challenge adequacy <strong>of</strong>data provided.Able to come to a justified conclusion on the health<strong>of</strong> China’s economy using data from Table 2 and itslimitations.1 – 34 – 61 – 2(e)Discuss the effectiveness <strong>of</strong> the 4 trillion-yuan stimulus plan insustaining economic growth and improving the living standards inChina.The 4 trillion-yuan stimulus plan is a spending plan that can be easilyclassified into two investment categories: infrastructure and people’slivelihoods.Thesis: The stimulus plan is effective in sustaining EG and LSStandard <strong>of</strong> living (or living standards) refers to the level <strong>of</strong> material aswell as non-material well-being <strong>of</strong> an individual. The living standards <strong>of</strong>people (material and non-material wellbeing) will be improved throughhigher spending on housing, public healthcare, education andinfrastructure that improve living condition and etc. Material well-beingwill improve as the income/consumption level <strong>of</strong> the people increasedfollowing the 4 trillion-yuan stimulus plan. Non material well-being willimprove as health <strong>of</strong> the people and the living condition improved. Asthe living standards <strong>of</strong> many people in China are not high, the stimulusplan will be effective in improving the living standards in China. Thiscan be seen in Table 3 as the government is spending substantialamount <strong>of</strong> 370b and 350b yuan respectively in improving rurallivelihoods and infrastructure and environmental protection for instance.[8]


The higher spending, translating into higher G in AD, can lead to higheroutput and national income. (Illustrate using AD-AS diagram and brieflyexplain how inventories, production, output, employment and nationalincome would be affected via the multiplier effect.) This will result inhigher actual growth as the increased spending is <strong>of</strong> significant amount(Extract 5 states that this sum represents about 16 percent <strong>of</strong> China’seconomic output in 2007 and is roughly equal to the total <strong>of</strong> all centraland local government spending in 2006.)General Price levelASAD 0 AD 1P 1P 0Real national incomeY 0 Y 1The spending can also promote sustained growth as it is mainly oninfrastructure investment (Table 3 lists the proposed spending oninvestment projects such as transportation network, power grids,technological innovation and education.) that can increase theproductive capacity <strong>of</strong> the country. LRAS will increase and together withan increase in AD, result in sustained growth.Anti-thesis: The stimulus plan is ineffective in sustaining EG and LSIn the short run, the spending is mainly focusing on infrastructureconstruction and relatively little is spent on improving living standardsas stated in the last paragraph <strong>of</strong> Extract 5. Hence it will not be effectivein raising living standards as China is large and populous and moreneeds to be spent.In the long run, sound infrastructure will definitely contribute to China’spotential growth, and is also necessary to improve people’s livingstandards. However in the short run, in the case <strong>of</strong> China, infrastructureconstruction would only play a limited role in boosting domesticconsumption and hence has limited effect on material well-being <strong>of</strong> thepeople. While large scale infrastructure construction could generate


employment to some degree and could create some demand forconstruction materials, equipments, services etc., it would notcontribute much to boost domestic consumption. The little impact ondomestic consumption has serious implication on sustained growth aswages is bound to increase in China with her development into adeveloped country reducing its comparative advantage in productionand exports <strong>of</strong> labour intensive goods. This, together with the fact thatsaving rates will most likely fall in the future as there is already“excessive savings”, would dampen exports and its growth. Exports willbecome less <strong>of</strong> a driver <strong>of</strong> economic growth and if domesticconsumption is not increasing enough to compensate that the long termgrowth <strong>of</strong> China would be adversely affected.Furthermore, as stated in Extract 5, local governments and stateenterprises are backed by powerful vested interests strongly preferringto invest in pr<strong>of</strong>itable infrastructure projects. These pr<strong>of</strong>itable projectsmay not be the best projects in improving the infrastructure quality <strong>of</strong>China for the long term and as such, represent a waste <strong>of</strong> scarceresources at its worst or an inefficient use <strong>of</strong> resources at the least.ConclusionAs China’s main driver <strong>of</strong> economic growth is investment (Its highsaving rate provided the necessary goods and resources forinvestment. Another indicator <strong>of</strong> high savings is the large BOTsurplus.), a stimulus plan that focuses on investment will not beeffective in improving living standards and sustaining economic growth.The 4 trillion yuan stimulus will achieve much better results ifpreference was given to projects related to securing people’slivelihoods instead <strong>of</strong> infrastructure construction. In addition, China hasalso implemented expansionary fiscal and monetary policies to helpbolster the economy and to sustain growth. Hence it can be seen thatthe 4 trillion yuan stimulus package is not sufficient.L1L2EAble to define living standards, sustained economicgrowth and state how the 4 trillion spending plan canimprove these.Able to explain how living standards and sustainedeconomic growth can be achieved through higherinvestment and govt spending in theory with attemptin applying to context.Able to come to an explained and justified conclusionwith regards the effectiveness <strong>of</strong> the spending plan.1 – 34 – 61 – 2


2010 RI (JC) Prelims H1 Paper CSQ2: Foreign Investment(a) (i) Using Figure 1, compare the trend <strong>of</strong> Foreign Direct Investment inSingapore and Singapore’s Direct Investment abroad between 2004 and2008 [2]Both increase between 2004 and 2008. However, FDI in Spore is always higher thanSpore’s direct investment abroad.(ii)With reference to Figure 1 and Figure 2, assess the likely BOP position<strong>of</strong> Singapore between 2004 and 2008.[6][6]BOP comprises current account and financial account.From Fig 1, Spore’s BOT is likely to be a surplus since value <strong>of</strong> exports is greater thanvalue <strong>of</strong> imports. This is likely to lead to a current account surplus. Also from Fig 2,Financial account is likely to be in surplus because financial inflow due to FDI is greaterthan financial outflow due to Spore’s direct investment overseas.Therefore BOP is likely to be in surplus.However, there are other components in the current account (income flow and moneytransfer) and financial flow account (hot money flow) which are not provided in case study.Therefore, we cannot assess BOP position because <strong>of</strong> insufficient data.((b)Discuss whether China’s protectionist measures in Extract 5 can be justified interms <strong>of</strong> economic theory.[6]Thesis:Based on the theory <strong>of</strong> comparative advantage, USA currently has greater endowment <strong>of</strong>technology and high quality labour force, therefore USA has CA in knowledge intensiveindustries like advanced technology and R&D. However, China has potential CA in theseindustries too and therefore, has engaged in protectionist measures to help its firmsdevelop at their infancy stages. This is justified but only in the short term.Anti-thesis:From the case study, it seems that the measures are not meant to help new start-ups in theshort term. Some <strong>of</strong> the measures require American firms to create Chinese brands andtransfer R&D <strong>of</strong> new products to China in order to have access to Chinese markets, eg theregulation for foreign firms to have joint venture with Chinese firms in the automobileindustry. This clearly is unfair trade practice against foreign firms.Synthesis:China’s protectionist measures do not seem justified and as seen in the extract, this islikely to invite retaliation from her trading partners. When this happens, all parties will sufferin terms <strong>of</strong> reduced trade and reduced investment in the world.


Max 3 marks for one sided answer.Max 5 marks if no judgement/ synthesis is given.(c)Compare how the different attitudes <strong>of</strong> Singapore and China towardsforeign investment may have an impact on their economic growth, usingboth the case study and your own relevant knowledge.[6]Singapore welcomes FDI because this will help to expand its actual economic growth andpotential growth. When foreign firms set up production and purchase more capital goodsand services, for example, when Soitec set up a $700m facility to produce wafer (Extract4), more factors <strong>of</strong> production are employed, including workers. As household incomeincreases, more consumer goods and services are demanded. Through the multipliereffect, GDP increases by a multiple. Potential economic growth also increases in the longterm as these firms may bring in greater technology too. Soitec brings the latest advancetechnology and over time, this is likely to be transferred to the Singapore industries andworkers.On the other hand, China, with its protectionist antagonist measures against foreign firms(pick any example from Extract 5), is likely to discourage foreign firms with from setting upbusinesses there. This leads to reduced demand for capital goods and services in Chinaand transfer <strong>of</strong> technology or business expertise from foreigners may also be reduced.Thus, both actual and potential growth may suffer.However, it should be noted that China may not be worse <strong>of</strong>f in reality, therefore theantagonist attitude. This is because China has sufficient demand from its domestic sectorand does not need foreign firms’ spending to boost its Aggregate Demand, unlike Sporewhich has such a small domestic market. Foreign firms set up business in China because<strong>of</strong> the large Chinese market. Further, China probably has confidence in its ability toinnovate and harness technology because <strong>of</strong> its relatively large number <strong>of</strong> talent, unlikeSpore, which has a very small pool <strong>of</strong> talent.(d)Assess whether attempts by the government to “fire up householdconsumption” is desirable and effective for China to achieve a higher currentand future standard <strong>of</strong> living.[10]• SOL refers to both the quantitative and qualitative aspect <strong>of</strong> the welfare <strong>of</strong> the people.• Identify the strategies used by the government to increase household consumption:Thesis:Taxation reform- Reducing income tax, increase inheritance, property tax:This will increase the disposable income <strong>of</strong> households and lead to increase inconsumption <strong>of</strong> goods and services and therefore a higher material SOL now. This isespecially beneficial to the lower income group and will improve the SOL <strong>of</strong> the averageChinese.Social reforms -Provide for old age, healthcare and education


This will increase SOL <strong>of</strong> Chinese as they will have greater access to affordable healthcareand education. Further, this will reduce the precautionary savings <strong>of</strong> household andencourage other kinds <strong>of</strong> consumer spending and this leads to greater quantitative aspect<strong>of</strong> SOL. The lower income group is likely to benefit most because healthcare andeducation take up a large portion <strong>of</strong> their income.Promote competition in the service industriesExpand service industries and introduce competition by removing barriers so that prices willfall and quality <strong>of</strong> services will improve, leading to higher SOL.Anti-thesis:Tax reform, like increasing inheritance tax, increases taxation on the wealthy anddiscourages savings from this group. This will lead to a fall in loanable funds and may stifleinvestment. It may also encourage high net worth individuals to move assets away fromChina and may add on to the detrimental effect on the economic growth and therefore thefuture SOL.An increase in financial burden on the govt as more needs to be spent on healthcare andeducation and the future generation may have to pay for these. These policies may alsosend a wrong signal to households and lead to an over reliance on govt for welfare. Sothough they are effective, they are not desirable.The qualitative aspects like stress level, level <strong>of</strong> pollution, spiritual well-being may notimprove with these policies to fire up consumption. It may actually lead to a society whichputs too much emphasis on materialism.Conclusion:These policies are likely to be effective in improving the current quantitative aspect <strong>of</strong> SOL,especially the lower income group, but they may lead to a lower SOL in the future.Nevertheless, it is necessary for the Chinese to have a larger domestic consumption sothat China can reduce its reliance on the external sector. But this should not be overdoneand a balance must be strike between household current consumption and investment forthe future.L3 (7-8)L2 (4-6)L1 (1-3)E (1-2)Student examines desirability and effectiveness <strong>of</strong> measures found in theextract to increase current and future SOLStudent does not fully address the question/ does not use measures found inextract/ one-sided answerStudent merely lists the measures without linking to SOLStudent provides value judgment based on economic analysis


2011 MJC Prelims H1 Paper CSQ2: The Road to Recoverya (i) Summarize the 2010 forecast growth rates in GDP for selected countries in Table2 [2m]All selected countries are forecasted to experience increase in output/GDP.However, all selected Asian countries are forecasted to grow faster than all the selected G7countries.a (ii) With reference to data, explain one possible reason why 2010 forecast growthrates differ from country to country [2m]Differences in investment levels.Observed from Figure 3: Investments as a % <strong>of</strong> GDP à rising trend for the Asian countriesbut decreasing trend for G7.Increased investments within China, India and Singapore àimplies firms expectations <strong>of</strong> higher demand for domestic output in the future à increase Cand I à increase in AD à hence explains why China, India and Singapore are forecasted tohave relatively higher growth rates than UK, US, Germany and Japan.a (iii) Is there enough evidence to support the view that there has been animprovement in the economic situation <strong>of</strong> Asian countries?Largely insufficient evidence. Data in Table 2 only representative <strong>of</strong> selected Asiancountries. Extract 4 suggests that Asian economies are recovering faster from the globaldownturnà qualitative nature <strong>of</strong> data à insufficiency. Quantitative indicator such as theforecast GDP growth rates given in table 2 is insufficient as they are forecasted growth rates“China’s households have been saving a larger slice <strong>of</strong> their income” (Extract 5)(b) Explain the impact <strong>of</strong> the above change on China’s circular flow <strong>of</strong> income [4m]Saving a large slice <strong>of</strong> their income à with every additional dollar earned, a greaterproportion goes to savings à with every round <strong>of</strong> spending a higher level <strong>of</strong> withdrawal fromthe circular flow <strong>of</strong> income à lower level <strong>of</strong> income retained within the inner flow à reducesthe size <strong>of</strong> the national income multiplier for China.(c)To what extent does the data suggest that the Chinese government should allowthe currency to appreciate in 2010? [6]Advantages <strong>of</strong> appreciation:1. Extract 5 : an appreciation <strong>of</strong> its currency will help to “lift consumers’ real purchasingpower”à Chinese consumers will be able to buy more imported goods and services


2. Appreciation is also needed to discourage excessive investments into manufacturingà loss-made export firms will exit the industry à more resources channelled to otherindustries, for instance, service industries as quoted in extract 7à which will help toincrease employment <strong>of</strong> workers3. Appreciation can also help to “reduce the trade deficit further, alleviate the risk <strong>of</strong> aprotectionist backlash abroad” àin fig 5, trade surplus was accumulating from 2005to 2008 with the highest trade surplus recorded in mid 2008Disadvantage <strong>of</strong> appreciation:• Extract 6 evidenced that China’s exports as a percentage <strong>of</strong> GDP are 32%à stillrelatively reliant on exports earnings as a source <strong>of</strong> national income à figure 4 showedthat when yuan appreciated in June 2008, trade surplus started to fall significantly asshowed in figure 5Synthesis/conclusionAppreciation <strong>of</strong> Yuan recommended.è Important to improve equity à excessive investment into capital-intensive growth hascaused “the share <strong>of</strong> national income going to households (as wages and investmentincome) to fall, while the share <strong>of</strong> pr<strong>of</strong>its has risen”Fall in export revenue is likely to be compensated by the rise in domestic C and overall ADwill still increase à actual growth maintained. (Extract 4)(d) “Excessive levels <strong>of</strong> investment are not a recipe for sustained rapid growth”Comment. [6m]àInvestment inferred in extract 5 à public investment and private investment “Excessivelevels <strong>of</strong> investment” à implies economic imbalance in China à Government stimulusfocuses on promoting investment à increases I à at the expense <strong>of</strong> C falling or growing ata slower rate.à Private investment à Loosening <strong>of</strong> credit à reduce interest rates àincrease ininvestment for firms à firms channel investments to capital intensive industries àproduction <strong>of</strong> output relies more capital goods rather labour à hence limited income flows tohouseholds à households reduce lower wages à C falls è Increase in I is <strong>of</strong>fset by C àAD increases at a slower extent à Real output increases at a slower rates à henceexcessive levels <strong>of</strong> investment are not a recipe for sustained growthPublic investment à “However, the pace <strong>of</strong> investment is unsustainable” à excessive levels<strong>of</strong> investment under taken by government à unsustainable because part <strong>of</strong> the investmentunder taken by government à lead to strain on budget position for China à may lead to abudget deficit for China. Draw AD-AS diagram to link to growth rates


Evaluate:• Public investment à However “government needs to prop up demand in the shortterm” à government spending towards capital goods output is necessary à a means<strong>of</strong> lifting China economy out <strong>of</strong> the recession in 2009 à expansionary fiscal policy inthe short term to increase real output in order to increase consumer confidence àtowards longer term rapid growth• Public investment à “China as a poor country needs infrastructure” à otherobjectives also have to be considered à pursue <strong>of</strong> material and non-material wellbeingRecommendationInvestment is still an important determinant <strong>of</strong> growth for China à however, production <strong>of</strong>capital goods à intended for production <strong>of</strong> consumption good à China should achievebalance between C and I as the driving components <strong>of</strong> domestic demand for China.(e) In light <strong>of</strong> the issues raised in the extracts, discuss the relative desirability <strong>of</strong>globalization across the world economies.[8m]Define globalizationAdvantages and disadvantages <strong>of</strong> globalisationDesirableIncreased mobility <strong>of</strong> goodsacross economies à greaterintegration <strong>of</strong> economies• Globalization à Enablescountries to produce goodsaccording to their comparativeadvantage in production àenable country to consumeoutside the PPC àIllustrate withPPC diagram• Evidence: Extract 4 – Asia- China– US/Rich country traderelationship – is an example <strong>of</strong>each country producing a part <strong>of</strong>the whole process <strong>of</strong> productionaccording to CA.• In times <strong>of</strong> strong demand forgoods due to higher GDP growthacross the world → higher worldoutput, higher domesticeconomic growth, generategreater employment.• Analyze with AD-AS analysis:Apply to countries in Extract 4:Not desirableGlobalization –volatility <strong>of</strong> macrovariables• Evidence: Extract 4 “Hardest hit”,“export plunged’, 60% <strong>of</strong> exportsend up in the rich world” impliesthat the integration <strong>of</strong> economylead to dependence on externalperformance <strong>of</strong> the economy todetermine the achievement <strong>of</strong>macro aims within a domesticeconomy.• Evidence Extract 6: Export Led –growth economies will experiencegreater degree <strong>of</strong> volatility• Evidence extract 5: Chinabuilding its domestic engine àneed to decrease dependence onexternal economies à showsless desirability for globalisationTo analyze with AD-AS analysis


Increase Real output for Asiaproducing intermediate output,Increase Real output for Chinaproducing final goods assembly.Evaluation: Beggar thy neighboreffect à many countries can gain• Increased mobility <strong>of</strong>capital ( gross fixed capitale.g. machinery, rawmaterials )• Evidence: Extract 7: one <strong>of</strong>the factors that determinelonger productivity growth isopenness <strong>of</strong> an economy toFDI• Use AD-AS framework toanalyse.• Globalization – negativeimplications on governmentbudget positionEvidence: “biggest fiscalstimulus” – in the context <strong>of</strong>extract 2 – the example <strong>of</strong> therecent financial crisis required alarger magnitude <strong>of</strong> increased inG expenditure in order toincrease AD <strong>of</strong> the economy.• Increased mobility <strong>of</strong> laborand other factors <strong>of</strong>productions acrosscountries-­‐ FOP can now be easilyovercome with inflow <strong>of</strong>labour and capitalresourceWhilst globalization increases theinternational factor mobilityacross economies - it aggregatesthe negative impact <strong>of</strong> factorimmobility within the domesticeconomy. The inflow <strong>of</strong> betterquality labour and capital mayrender some skills redundant andhence local workers becomeunemployed – the mismatch <strong>of</strong>skills unresolved will lead tostructural unemployment withinthe country.à Ext 7 “rural to urban areas”movement <strong>of</strong> workers is notcurrently taking place in China àcould be due to available inflow<strong>of</strong> foreign labour à hence slowsdown need to tap on the Chineselabour force à thus slowing downthe job opportunities available toChinese households in the ruralareas à reduces their ability toearn wages and hence increasepurchasing power and materialwelfare.


Synthesis: Justify whether Globalization on a whole is desirable or notStill desirable as analysed above à Extent <strong>of</strong> benefits outweighs the costsdue to volatility in trade cycles especially for smaller Asian economies (Ext7) all the more there is greater dependence on external demand à hencemore desirable for small economiesL3 (5-6)L2 (3-4)L1 (1-2)E (1-2)Students are able to evaluate the desirability <strong>of</strong> globalisation through at least2 areasStudents provide one-sided answer, or two-sided argument on one areaStudents listing the benefits <strong>of</strong> globalisationStudent give valued judgment based on economic analysis


Section B: EssaysMicroeconomics:1. 2011 HCI Prelim H1 Paper Q3: CPE, Demand / Supply & Elasticities2. 2011 PJC Prelim H1 Paper Q3: Demand / Supply & Price controls3. 2011 JJC Prelim H1 Paper Q3: Demand / Supply & Elasticities4. 2011 DHS Prelim H2 Paper Q1: Elasticities & Government Policies5. 2011 RI Prelim H1 Paper Q3: Market Failure6. 2011 NJC Prelim H1 Paper Q3: Market Failure7. 2011 IJC Prelim H2 Paper Q3: Market FailureMacroeconomics:8. 2010 RVHS Prelim H2 Paper Q4: Circular Flow <strong>of</strong> Income9. 2011 VJC Prelim H2 Paper Q4: KEI/Standard <strong>of</strong> Living10. 2011 RI Prelim H2 Paper Q4: KEI and Policies11. 2011 HCI Prelim H1 Paper Q4: Macroeconomic Problems and Policies12. 2011 DHS Prelim H1 Paper Q4: Unemployment & Demand management policy13. 2011 JJC Prelim H1 Paper Q4: Macroeconomic Aims & Policies14. 2011 RVHS Prelim H2 Paper Q5: Macroeconomic Aims & Policies15. 2011 MI Prelim H1 Paper Q4: Inflation & Exchange Rate16. 2011 M1 Prelim H2 Paper 2 Q4: Monetary Policies17. 2010 ACJC Prelim 2010 H2 Paper Q6: International Trade18. 2011 DHS Prelim H2 Paper Q1: International Trade19. 2010 IJC Prelim 2010 H2 Paper Q6: International Trade


(1) 2011 HCI Prelim H1 Paper Q3(a) Car ownership necessitates using land to build a network <strong>of</strong> roads. Using the concept <strong>of</strong>scarcity, choice and opportunity cost, explain the problems the government faces in allocatingresources in the case <strong>of</strong> car ownership. [10]IntroductionWhat isResourceAllocationResource allocation refers to the choices that have to be made in terms <strong>of</strong> “what” and“how much to produce” from society’s limited resources. Given the problem <strong>of</strong> scarcity,the aim <strong>of</strong> resource allocation is to optimize the use <strong>of</strong> scarce resources so as to benefitsocietyI shall explain the problems the government face in allocating the use <strong>of</strong> scarce land tobuild roads for use by car owners.BodyUsing an SCO framework :ScarcityIn allocating resources to allow for car ownership the government faces the problem <strong>of</strong>scarcity. The government has to decide how to use the scarce land optimally to benefitsociety.Roads are meant for use by car owners. Thus, car ownership necessitates building anetwork <strong>of</strong> roads. Road building is the responsibility <strong>of</strong> the government given that it is apublic good. However, the demand or desire for car ownership is unlimited, whilst theavailability <strong>of</strong> resources to build roads for the use <strong>of</strong> cars owners is limited. In landscarce SG; this is all the more pressing problem. Government has got to find the optimalmix for land use.ChoiceTo achieve an optimal mix <strong>of</strong> land use, the government has to consider the variousalternative uses for it. Given the government’s role in providing public goods and meritgoods, these alternative uses include building schools, hospitals, parks and publichousing <strong>of</strong> benefit to society. Thus, the government has to decide “What” and “Howmuch” to produce <strong>of</strong> both roads and merit goods using state land.Figure 1:


Roads for Cars A X B Other infrastructures e.g. schools, parks Figure 1 illustrates the choices facing the government in using the scarce resource land tobuild either roads or other public infrastructures like schools and parks. Given thatresources are scarce, it is not possible for the government to allocate resources such thateveryone is satisfied. Given scarcity, building more roads would mean less land isavailable for other infrastructure and vice versa. In other words, the government has tochoose between a combination like A and B. Such choices would invariably involveopportunity costs or trade-<strong>of</strong>fs. However, a combination like Point X represents a choicethat might satisfy everyone in society but is unattainable.OpportunitycostIn making optimal choices the government has to weigh benefits against the opportunitycosts <strong>of</strong> each alternative use <strong>of</strong> land.Example:If the government chooses to build more roads it has to consider the benefits against theopportunity costs. The benefits would include improved mobility <strong>of</strong> goods and people i.e.better transportation and communication which could enhance economic activity (i.e.increase GDP) and improve standard <strong>of</strong> living for its citizens (e.g. higher incomes; shortercommuting times etc). However, the trade-<strong>of</strong>fs or opportunity costs <strong>of</strong> land use for roadbuilding would have to include social costs such as externalities generated from carownership e.g. road congestion and vehicle pollution. In practice, it is difficult to measurethe costs <strong>of</strong> such negative externalities accurately, since these are spillover costs that arenot captured by the price mechanism.ConclusionThus as explained and illustrated above, in allocating land use <strong>of</strong> road building thegovernment faces the problem <strong>of</strong> scarcity which necessitates making optimal choices andweighing opportunity costs.


Level Marks DescriptorsL3 9-10 Excellent analysis, coherent structure and good exemplification7- 8 Answer is contextual and has both adequate scope and depth <strong>of</strong>analysis and no major conceptual flaws/errors.L2 6 Answer is contextual and has depth but is lop-sided e.g. did notexplore the use <strong>of</strong> the 3 concepts adequately.5 Answer is contextual and has adequate scope or breadth but lackingin depth <strong>of</strong> analysis.L1 4 Answer is contextual but is too superficial; sketchy and lacking incoherence. Have glaring conceptual errors/flaws.3 Too descriptive or generic ( i.e. non-contextualized)0- 2 Mostly irrelevant answer(b) COEs are quotas fixed by the government to control car ownership. Thegovernment intends to introduce a new measure to reduce the number <strong>of</strong> COEs. Inview <strong>of</strong> weakening US recovery, analysts are unsure <strong>of</strong> its impact.Using demand and supply analysis assess the impact <strong>of</strong> these events on the marketfor COEs and COE revenue collected by the government. [15]<strong>Suggested</strong> Answer:Demand <strong>of</strong> COEs is a derived demand for cars. This means that the demand <strong>of</strong> COEs is aresult <strong>of</strong> the demand for cars.The weakening US recovery will have a negative impact on Singapore’s economy. A weakereconomic performance will bring about both less wage increment and lower level <strong>of</strong>employment. As a result, consumers maybe more cautious in making purchases like cars.This will lead to a negative impact on the demand for normal goods such as cars. This willcause the demand for cars decrease and thus a decrease in the demand for COEs.Price S 1 P 1 A P 2 B D 1 D 2 O Q s Figure 2: the market for COE Qty


With reference to Figure 2 above, (assuming that ss remain constant), the supply for COEs isperfectly inelastic at Qs, as the quantity is determined by the government. Hence, the price <strong>of</strong>the COEs is determined by the level <strong>of</strong> demand. Given the decrease in demand in the currentquarter due to reasons given above, the price <strong>of</strong> COE will decrease from P 1 to P 2 while thequantity remains at Qs. As the PED <strong>of</strong> cars is elastic (since car is not a necessity and it takesup a large portion <strong>of</strong> income), the demand for COE is likely to be elastic too.The lower demand will be accompanied by the actual cut in the supply for COEs in light <strong>of</strong> thenew quota system..Price S 2 S1 P 2 P 1 D 1 O Qs’ Q s Figure 3: the market for COE Qty


With reference to Figure 3 above, with the sizeable cut in the supply <strong>of</strong> COEs, the price inthe subsequent quarters will increase further.Putting the 2 impacts together, the effects together, the changes in the COE markets dependon the relative changes in the magnitude.Scenario 1: DD decrease > SS decrease: Prices <strong>of</strong> COEs will decrease and governmentrevenue will also drop.Scenario 2: DD decrease < SS decrease: Prices <strong>of</strong> COEs will increase and governmentrevenue will also drop (assuming PED is elastic). This is a more likely scenario.However, this cannot be inferred directly from the information given. For example, the recentcutback <strong>of</strong> COE was drastic (i.e. Government aimed for zero % growth in vehicle population.On top <strong>of</strong> that fall in the no <strong>of</strong> de-registered cars also affected the supply <strong>of</strong> new COEs).


Conclusion:The future impact in the COE market will depend on the relative changes in magnitude <strong>of</strong> DDand SS. Empirically, the price rose after the supply <strong>of</strong> COE was cutback. In fact, it even hit$70-$80K at one point. This was because the cutback in supply <strong>of</strong> COE outstrips the fall indemand for COE as the impact <strong>of</strong> the weak US recovery did not have any significant impacton the SG economy.Knowledge, Application, Understanding and AnalysisL3L2L1EvaluationFor a developed and balanced analysis on the impact on the COEand government revenue using relevant economic concepts includingPED and PES.For an answer that has depth <strong>of</strong> analysis but is lop-sided e.g. no Antithesisi.e. too narrow in scopeFor an answer that has adequate breadth but is lacking in depth <strong>of</strong>analysis i.e. too superficialBroadly on track, but analysis is lacking in terms <strong>of</strong> both breadth anddepth i.e. too superficial, sketchyMostly descriptive answers ( smattering <strong>of</strong> econ analysis)Mostly irrelevant answers i.e. <strong>of</strong>f-tracked9 - 117– 8653-40-2E2 For an evaluative discussion based on economic analysis 3 – 4E1For an unexplained judgment, or one that is not supported byeconomic analysis1 – 2


(2) 2011 PJC Prelim H1 Paper Q3(a) Using demand and supply analysis, explain the use <strong>of</strong> price controls toprevent excessive price changes.[10]Interpret the QuestionCue Word(what are the skillsrequired for thisquestion?)ExplainCausal explanation <strong>of</strong>- how price ceilingcan curb priceincreases- price floors can curbexcessive price fallsConcepts(what are the conceptsrequired to answer thisquestion?)- Price Controls- Demand, Supply &Market EquilibriumSchematic MapContext(what is the context for thisquestion?)General“Excessive”Key wordIntroduction-Explain circumstances where price changescan be considered excessive- explain causes <strong>of</strong> excessive price changes- introduce dfferent types <strong>of</strong> price controlsPrice Ceiling- Define and explain why it isused to prevent prices fromrising beyond desired level- Diagram- Explain impact <strong>of</strong> priceceiling ie. shortage arisesPrice Floor- Define and explain why it isused to prevent prices fromfalling beyond desired level- Diagram- Explain impact <strong>of</strong> priceceiling ie. surplus arises


ConclusionSummarise and link to part (b)Thinking processIntroductionIntroduction- Explaincircumstanceswhere pricechanges can beconsideredexcessive.In the free market system, prices are determined by the interaction <strong>of</strong>demand and supply. However, in some cases excessive pricechanges can cause macroeconomic problems such as inflation orworsen the distribution <strong>of</strong> income in the economy. Thus thegovernment may opt to curb price increases through the use <strong>of</strong> aprice ceiling and excessive price decreases in the form <strong>of</strong> a pricefloor.


Thinking processExcessiveincrease in price- Explain causes <strong>of</strong>an increase in priceBodyPrice increases can be caused by a rise in demand or a fall in thesupply <strong>of</strong> a good. Assuming that the market is initially in equilibrium, atprice P and quantity Q, an increase in the demand from D to D 1 willlead to a shortage at the current price. Upward pressure on priceresults in price rising to P 1 .Price Figure 2S 1SCauses <strong>of</strong> an Increasein PriceP 2PP 1QD 1DQuantityQ 1Similarly, for a market at an initial equilibrium at price P 1 and quantityQ 1 , a fall in the supply <strong>of</strong> the good from S to S 1 will lead to upwardpressure on prices, causing price to rise to P 2 .Thus, an increase in price can be brought about by an increase indemand, a fall in supply or a combination <strong>of</strong> both. If the situation isprolonged, an excessive increase in price results.On the other hand, a fall in demand would lead to a surplus <strong>of</strong> the goodat current prices, leading to fall in its market price. A rise in supply willalso lead to a fall in price. Thus, a fall in the price <strong>of</strong> a good can becaused by a fall in demand, a rise in supply or a combination <strong>of</strong> both. Ifthe situation is prolonged, an excessive decrease in price may result.The situation is worsened if the demand and supply <strong>of</strong> a good isrelatively inelastic. In such cases, any change in demand and supplycan lead to sharp changes in price. This is especially true in primarymarkets for raw materials and agricultural products. Such excessivechanges in price can worsen economic problems such as inflation orthe distribution <strong>of</strong> income. Thus the government attempt to curb priceincreases through the use <strong>of</strong> price controls.Define price ceilingExplain why thegovernmentimposes them,giving examples.Price ceilings are maximum prices set below the market equilibriumprice so as to lower prices and to prevent further price increases Thiscan be done to achieve a lower rate <strong>of</strong> inflation by keeping the prices <strong>of</strong>necessities and raw materials low; or, to improve income distribution tokeep the price <strong>of</strong> necessities affordable, e.g. rent controls to keep theprice <strong>of</strong> rental housing affordable.Referring to Figure 1, assume that the market for rice is initially inequilibrium at price P and output Q. If the demand for rice rises to D 1 ,


Explain causes <strong>of</strong>an increase in priceExplain how a priceceiling prevents aprice increaseequilibrium price will increase to P E if the government does notintervene.However, as rice is a staple food, the government may wish to keepprices low so as to ensure it remains affordable the governmentimposes a price ceiling at price P MAX , preventing further price increases.At price P MAX , the quantity demanded <strong>of</strong> rice is Q units. But firms wouldonly be willing to supply quantity Q s <strong>of</strong> rice. Thus, a shortage <strong>of</strong> QQ s iscreated at P MAX .PriceS 1Figure 1Price CeilingP EP MAX =PMaximum PriceQShortage DQ EQ DD 1QuantityLink back toquestion.Define price floorExplain why thegovernmentimposes them,giving examples.Hence, the price ceiling prevents price from increasing beyond the levelset by the government. Any further increase in demand and or a fall insupply will not raise prices but will worsen the shortage instead.However, the shortages created by this policy would need to beaddressed by the government, either through an efficient rationingprocess, or by subsidizing producers to raise output.Price floor is the minimum price set by the government above themarket equilibrium price, which is deemed too low and to prevent anyfurther decline in price. Price floors are seen as a common tool for thegovernment to achieve a more equitable (fair) distribution <strong>of</strong> income.For example, to protect farmers from the loss <strong>of</strong> revenue when themarket prices for their agricultural products decline sharply, thegovernment may set a price floor on the agricultural products; similarly,minimum wage laws can be implemented help unskilled workers.Taking the example <strong>of</strong> the market for c<strong>of</strong>fee beans. Referring to Figure2, the market is initially in equilibrium at price P and output Q. If thesupply <strong>of</strong> c<strong>of</strong>fee falls to S 1 , equilibrium price will fall to P E if thegovernment does not intervene.


Explain how a priceceiling prevents aprice increasePriceSFigure 2P MIN =PSurplusS 1Price FloorMinimum PriceP EQQ EQ SDQuantityHowever, to protect c<strong>of</strong>fee farmers from a loss in revenue, thegovernment may decide to implement a price floor, P MIN . At price P MIN ,the quantity demanded <strong>of</strong> c<strong>of</strong>fee is Q units but firms are willing tosupply quantity Q S . Thus, a surplus <strong>of</strong> QQ s .Link back toquestion.Thus, a price floor would prevent prices from declining excessively.A fall in demand and/or a rise in supply would no longer lead to adecline in price but lead to a larger surplus instead. However, thegovernment would nees to remove the surpluses that result fromthis policy by buy ing up the excess supply and selling it during apoor harvest or sell it abroad to other markets. Thus, a highexpenditure is incurred by to subsidise this scheme.ConclusionHence, the government can limit price changes through the use <strong>of</strong> aprice controls. However, the implementation <strong>of</strong> ceiling or price floorresults in shortages and surpluses, respectively, which would need tobe addressed through further policy measures.L1 § <strong>Answers</strong> that explain only one type <strong>of</strong> price controls.§ Theoretical answers that explain price floor and price ceiling w/ogiving e.gsL3 §<strong>Answers</strong> that use lack demand theoretical and framework, supply concepts lack content to show or clearly containhow concept price errors floors and price ceilings prevent excessive pricechanges. Appropriate examples are provided§ Clear explanation <strong>of</strong> how price floors and ceilings prevent pricechanges using examples. Answer may not explain clearly whyprice changes or why these would be considered “excessive”L2 § Use <strong>of</strong> demand and supply concepts to explain how both pricefloor and price ceilings affect prices. <strong>Answers</strong> may contain gaps,fail to address issue <strong>of</strong> price changes directly or may lackexamples.] 3 - 4]9 - 101 – 27 - 85 – 6


(b) Discuss the view that price control is the most effective way to reduce inflation in an economy.[15]Interpret the QuestionCue Word(what are the skills required for thisquestion?)Discuss- A balanced answer wherebycircumstances <strong>of</strong> when priceceiling is most effective andwhen it is not is expected- The thesis/anti-thesis approachis recommendedConcepts(what are the conceptsrequired to answer thisquestion?)- To convert “pricecontrol” to mean“price ceiling”Context(what is the context for thisquestion?)InflationKey word“Most effective” implies a needto demonstrate comparison with2 or more policies.Schematic Map“reduce” means a fall in GPLwith/without solving inflationIntro :Define inflation/policies which can be usedBody : Thesis: When is price ceiling most effectiveAnti-Thesis : When is price ceiling less effective/use <strong>of</strong> otherPoliciesConclusion :Take a stand to the viewThinkingprocessDefineterms andscope theanswerThesis :Benefits <strong>of</strong>priceceilingIntroductionInflation is the sustained increase in the general price level <strong>of</strong> an economy. Thereare many policy tools that can be used to reduce inflation in an economy. In 3(a), aprice ceiling is one <strong>of</strong> the ways in which excessive price increases can beprevented. Others include demand management policies such as contractionarymonetary and fiscal policy, supply side policies, as well as exchange rate policy <strong>of</strong>currency appreciation.BodyPrice ceilings is the most effective to reduce inflation1. If inflation is high and unanticipated, then the consequences on the economycan be very damaging. Hence there is a need to put an immediate stop to theescalating prices. There are two ways to bring about an immediate stop to priceincreases, namely currency appreciation or price ceilings.2. For open economies, currency appreciation is used as its impact is immediatelytranslated in the form <strong>of</strong> lowered import prices and when such economies areimport reliant for its sources <strong>of</strong> raw materials, intermediate inputs and finished


Reason 2:If govtsaim toreduce &solveinflationWhile price ceilings reduces inflation, it does not tackle the root cause <strong>of</strong> inflation asit is a form <strong>of</strong> direct control on prices. For this purpose, other policies are required.Demand management policies are used to deal with inflation arising from thedemand side <strong>of</strong> the economy.Contractionary MP can be used. This occurs when money supply is reduced toincrease interest rates, so that consumption and investment spending will bereduced and hence AD will fall.Contractionary FP is another demand management policy where taxes areincreased to bring about a fall in consumption and investment spending. Togetherwith the fall in government spending, AD will fall. The fall in AD via both MP and FPwill bring about a fall in the GPL thus reducing demand-pull inflation.Supply side policiesThese are microeconomic policies aimed at increasing the quality and quantity <strong>of</strong>factors <strong>of</strong> production in order to raise the productive capacity <strong>of</strong> the economy.Policies which target at reducing the economy’s cost <strong>of</strong> production will help to bringdown cost-push inflation. These can be in the form <strong>of</strong> tax breaks to companies, R& D grants, fall in CPF contribution rates, reduction <strong>of</strong> trade union powers, moreinfrastructure spending, skills training programmes and so forth. When theeconomy’s capacity to produce is increased, price stability in the economy will berestored.However the above policies are not without its limitations:Both FP and MP are subjected to the unpredictable effects on AD. The rise in T andinterest rates may not cause a fall in C and I if the economy is booming as outlookand confidence is good. Hence AD may not fall to bring about the desired effect onthe GPL.Only the fall in G has a predictable effect on GPL but such a move is highlyunpopular although effective. Hence there might be a longer time lag for theimplementation <strong>of</strong> a fall in G as governments need to carefully consider itsundesirable social impact.Similarly most supply-side policies take a long time for its results on price stability tobe seen and to ensure such results, continuous funding is essential and this mayact as a constraint for governments who may already be running budget deficits.If the economy is concurrently beset with the problems <strong>of</strong> inflation and recession,then if contractionary MP and FP are used, they will be reducing inflation at theexpense <strong>of</strong> worsened unemployment rates. In such cases, if the government wereto apply contractionary FP and MP, the more effective it is, the worse will theimpact on jobs be. Hence when FP and MP is effective at reducing inflation, it mayamplify the problem <strong>of</strong> unemployment. This means the government may need toadopt expansionary MP and FP to tackle the jobs problems while resorting to eitherprice ceilings or currency appreciation to stem the price increases in the short run.


Take astandConclusionPrice ceiling provide a most effective way to reduce inflation without solving it. Toreduce inflation with the aim <strong>of</strong> solving it requires other policies despite theirlimitations. There is no one effective way. Very <strong>of</strong>ten governments resort to eitherprice ceilings or appreciation to dampen price increases in the short run but need apolicy package <strong>of</strong> supply side, MP and FP policies to maintain long-term pricestability as these policies tend to tackle the root cause <strong>of</strong> the inflation.MARK SCHEMEL1 (1-4) Sketchy answer with concept errorsL2 (5-7) Answer shows price ceiling is not the most effective due to limitations & explains 2or 3 other policies with limitations. Concludes that price ceiling is not the mosteffective.L3 (8-11)E1(1-2)E2(3-4)Answer does little to explore when price ceiling is most effectiveBalanced answers clearly explaining when the price ceiling is most effective andwhen it is notTakes a stand with little economic justificationTakes a stand with sound economic justification


(3) 2011 JJC Prelim H1 Paper Q3Taxi fares are going up at an additional 50 cents per ride due to an increase in fuel prices.Land Transport Authority <strong>of</strong> Singapore has increased the number <strong>of</strong> taxi licenses granted inthe midst <strong>of</strong> a stronger economy. In the same year, the car COE prices hit a 14-year high.Adapted from The Straits Times, 2011(a) Account for the fare increases in the taxi market. [10](b)Explain the concepts <strong>of</strong> price elasticity <strong>of</strong> demand and supply, and discuss therelevance <strong>of</strong> price elasticity <strong>of</strong> demand <strong>of</strong> Premier Taxis Company in maximising itspr<strong>of</strong>its. [15]<strong>Suggested</strong> answer (a)The taxi fare is determined by the interaction <strong>of</strong> demand and supply <strong>of</strong> taxis. Demand is thewillingness and the ability <strong>of</strong> the buyer to pay a particular price for a particular quantity <strong>of</strong> agood while supply is the willingness and ability <strong>of</strong> the seller to <strong>of</strong>fer different quantities <strong>of</strong> aparticular good at different prices.Possible non-price determinants <strong>of</strong> increased supplyThe Land Transport Authority <strong>of</strong> Singapore (LTA) has increased grants <strong>of</strong> taxi licenses to theall the taxi firms in Singapore, to increase the number <strong>of</strong> taxis on roads. This causes arightward shift <strong>of</strong> the supply curve to the right.Fuel prices hit a two and a half year high as a result <strong>of</strong> spiraling global crude oil prices spiral.As fuel is a factor input to operate the taxis. This increase in price <strong>of</strong> petrol leads to anincrease in cost <strong>of</strong> production. Taxi drivers will incur more cost when they increase theirnumber <strong>of</strong> travelling trips on the road. Due to the higher cost, the supply <strong>of</strong> taxis on the roadwill decrease quite significantly, causing a bigger decrease in supply leftwards.Net change in supplyWith the increase in fuel prices, taxi drivers may be more conscious <strong>of</strong> how much fuel they useand how efficiently they use fuel, they may be more selective over their destinations, chosenroutes, and times when they choose to drive, so as to avoid consuming more fuel thannecessary; but few taxi-drivers may consider stopping taxi-driving altogether, since it is afterall a livelihood for them; especially when the rise in fuel prices continues to increasesignificantly or rises sharply.Hence, the decrease in supply may be outweighed by the increase in supply <strong>of</strong> taxis, andoverall the supply <strong>of</strong> taxis will decrease, albeit by a small extent.


Possible non-price determinants <strong>of</strong> increased demandA rise in income due to economic boom as mentioned in the preamble that the GDPincreased 11.2%. Singaporeans have higher disposable income. The consumers will havehigher purchasing power and higher ability to spend. This causes an increase in demandfor normal goods such as taxis, causing a rightward shift <strong>of</strong> demand curve.As stated in the preamble, COE prices <strong>of</strong> cars have seen a sharp increase in 2011. Theimplication is that the prices <strong>of</strong> the cars have increased and have become relatively moreexpensive. When price <strong>of</strong> cars increase, this will lead to a fall in quantity demanded forcars. Consumers, given a stronger economy, switch to taking taxis as a substitute for carsand public transportation such as buses and MRT. There will be an increase in demand fortaxis, causing the demand curve to shift right even further.Synthesis (Diagram)Price <strong>of</strong> taxi (taxi fares)S20S1P2P1D2D10Figure 1: Market for taxisQuantity <strong>of</strong> taxis


The increase in demand is shown by a rightward shift in demand curve from D1 to D2 andthe decrease in supply is shown by a leftward shift in supply curve from S1 to S2, resulting ina price increase from 0P1 to 0P2.Comment: Students can write an increase in supply with explanations however they need toexplain that the increase in demand dominates the increase in supply to explain an increasein taxi fares for top L3 marksLevel Description Marks3 Well developed explanation <strong>of</strong> both Demand and Supply factors(4 in total) with clear link to evidence from the pre-amble.7-10Top marks for the synthesis <strong>of</strong> the effects (net effect on supply)2 Well developed explanation <strong>of</strong> either determinants <strong>of</strong> Demand orSupply5-6ORUndeveloped explanation <strong>of</strong> both Demand and Supply factors1 Only explains either demand or supply determinants withconceptual errors1-4


<strong>Suggested</strong> answer (b)Price elasticity <strong>of</strong> demandThe price elasticity <strong>of</strong> demand (PED) measures the responsiveness <strong>of</strong> a change in thequantity demanded <strong>of</strong> a good to a given change in the price <strong>of</strong> the good itself, ceterisparibus.The numerical value <strong>of</strong> the price elasticity <strong>of</strong> demand is always negative, which merelymeans that the price and quantity demanded <strong>of</strong> a good have an inverse relationship. Wetend to ignore the negative sign when interpreting the PED value, i.e. we take the absolutevalue.If the numerical value <strong>of</strong> price elasticity <strong>of</strong> demand is greater than one, i.e. PED >1, then thedemand for a good is said to be price elastic. If PED < 1, the demand for a good is saidto be price inelastic.Price elasticity <strong>of</strong> SupplyThe price elasticity <strong>of</strong> supply measures the responsiveness <strong>of</strong> a change in the quantity supplied<strong>of</strong> a good to a given change in the price <strong>of</strong> the good itself, ceteris paribus.The numerical value <strong>of</strong> price elasticity <strong>of</strong> supply is always positive, which merely means thatprice and quantity supplied for a good have a direct relationshipIf the numerical value <strong>of</strong> price elasticity <strong>of</strong> supply is greater than one, i.e., PES >1, then thesupply <strong>of</strong> the good is said to be price elastic. This means that a change in the price <strong>of</strong> a goodwill cause a more than proportionate change in quantity supplied, ceteris paribus.If PES < 1, the supply is said to be price inelastic. This means that a change in the price <strong>of</strong> agood will cause a less than proportionate change in quantity supplied, ceteris paribus.Relevance <strong>of</strong> PED to Premier TaxisPremier Taxis is a pr<strong>of</strong>it maximizing producer. Pr<strong>of</strong>its are the difference between total revenue(price x quantity) and total cost. Pr<strong>of</strong>its will increase if total revenue increases while total costremains constant or while total revenue increases more than the increase in total cost. Since aPremier Taxis can usually control the price <strong>of</strong> its good, knowledge <strong>of</strong> the price elasticity <strong>of</strong>demand would help Premier Taxis in its pricing strategies in order to increase total revenue andhence pr<strong>of</strong>its.


The producer can use the knowledge <strong>of</strong> PED for its good to reduce or increase the priceso as to increase its total revenue.Premier Taxis would be likely to have many competitors such as ComfortDelgro, CitiCab andSMRT who are close subsititutes. Hence, the demand for Premier taxis is price elastic.As such, the producer should lower its taxi fare. This will cause a more than proportionateincrease in quantity demanded, ceteris paribus. Thus total revenue increases assuming totalcost remains constant or increase in cost to a smaller extent, then its pr<strong>of</strong>its would rise.However, when Premier Taxis lower their price, its competitors would very likely follow suit andmay even lower their prices more. This would trigger a price war which could result in allproducers being worse <strong>of</strong>f and only benefiting the consumers. Thus, price cuts may not alwaysbe a good strategy.In view <strong>of</strong> rising costs <strong>of</strong> fuel, costs <strong>of</strong> production have risen. Hence, for Premier Taxi, it not onlyneeds to consider strategies that increase its total revenue, there is also a need to considerstrategies to reduce costs as well. For example, the Premier Taxi can negotiate bulk discountfor entire taxi fleet for servicing, maintenance and fuel costs. However, the firm must increasetaxi fleet so as to increase EOS enjoyed.Perhaps, a better strategy is to increase its demand and/or to make the demand for hisproduct price inelastic by product differentiation and product development.Product differentiation includes using strategies like advertising, product branding, promotionslike giving free gifts etc. Product development involves using research and development andinnovation to improve on product design <strong>of</strong> taxis, either to improve comfort or facilities <strong>of</strong>fered inthe cab; or to improve fuel efficiency <strong>of</strong> the taxi, hence lowering costs and consequently prices;or improving the quality <strong>of</strong> the services provided, such added convenience for customers:nets/credit card payment modes, to improve customer service <strong>of</strong> call service personnel andtaxi drivers through training like phone service etiquette, familiarity to routes, improvelanguage ability and promoting places <strong>of</strong> interests to foreign visitors, advertising/branding <strong>of</strong>service standards on fleet <strong>of</strong> well-maintain taxis, promptness <strong>of</strong> call service centre(hardware) and experience <strong>of</strong> taxi drivers (s<strong>of</strong>tware) and to increase the source <strong>of</strong> revenueby selling advertisement spaces on taxis.


Limitations <strong>of</strong> non-pricing strategiesThe implementation <strong>of</strong> these strategies may be costly and these high costs may <strong>of</strong>fsetrevenue earned. In addition, other taxi companies may also follow suit, hence defeating thepurpose <strong>of</strong> implementing the strategy in the first place, which is to differentiate product andmaking demand inelasticLimitations <strong>of</strong> PEDElasticity are based on Elasticity are based on past data - taste and expenditure changesover time, making the data less useful. With the rise <strong>of</strong> the internet, the elasticity figures willchange quickly. Hence the firm would need to collect updated empirical data and evidence.Based on the assumptions <strong>of</strong> ceteris paribus - there are various factors that affect thedemand for a product. The three concepts <strong>of</strong> elasticity only considers price, price <strong>of</strong> relatedgood and income level. The taxi firm needs to consider ways to reduce costs. He will need tomonitor the cost structure to ensure that his firm is cost effective.Knowledge, Application, Understanding and AnalysisL3L2L1For an answer that has a well-developed analysis <strong>of</strong> PED, PES and howPremier Taxis can make use <strong>of</strong> the price elasticity concept in devising price,non-price and output strategies, with application to the given context <strong>of</strong> taxisFor Top L3 marks, students must include answers <strong>of</strong> PED


(4) 2011 DHS Prelim H2 Paper Q1Households today pay on average just half the amount they were paying in early 2008 fortheir broadband internet connection. On the other hand, food prices have risen by 18% overthe same period.a. Explain the likely effects <strong>of</strong> the above price changes on households’ expenditure.[10]b. To stabilise food prices, several governments have released grains and frozen meatfrom their stockpiles. To what extent does the success <strong>of</strong> this policy depend on theprice elasticities <strong>of</strong> demand and supply?[15]<strong>Suggested</strong> AnswerPart (a)Recognisethedifferencesin PEDbetween the2 gdsEffect <strong>of</strong> thepricechange onhhs’expenditure(economicintuition)(graphs)Internet broadband connectionDemand for internet broadband connectionis likely to be price elastic.Broadband internet connection is a nonnecessity,part <strong>of</strong> their discretionaryspending after they have satisfied theirbasic needs àQd is likely to be responsiveto price changes(Students may also argue that PEDdepends on the country / type <strong>of</strong> householdin question. Demand for bandwidth may beprice inelastic for high Y countries andhouseholds that enjoy online gaming,where it is seen as a necessity à totalexpenditure may fall with falling prices)Give price elastic demand, a reduction inthe price brings about an increase inexpenditure.A reduction in its price, ceteris paribus,brings about a larger than proportionateincrease in Qdàreduction in expenditurefrom lower per unit spending(P 1 P 0 ab)more than compensated by increase inspending from increased units bought(Q 0 bc Q 1 ) à increase total expenditureFoodDemand for food is likely to be priceinelastic.Food as a necessity is required forsurvival, not something that individualscan do without even as its price risesàQd is likely to be unresponsive to pricechangesGive price inelastic demand, an increasein the price brings about an increase inexpenditure.An increase in its price, ceteris paribus,brings about a smaller than proportionatereduction in Qd à increase inexpenditure from higher spending per unit(P 2 P 3 hg) more than <strong>of</strong>fsets the decreasein expenditure from from cutback in unitsbought (Q 3 gfQ 2 ) àincrease totalexpenditure


Internet broadband connectionFoodEffect <strong>of</strong>the pricechange onhhs’expenditurePriceP 0P 1bacS 0S 1P 3P 2PriceghfS 1S 0DDOQ 0 Q 1QuantityOQ 3 Q 2QuantityAssumptions:i. The price changes are driven by ss changes ii. No substitution between the 2 goods


Part (b)1. Explain how the policy is intended to work to help stabilise / moderate increase in foodprices.The policy to release grains and frozen meat from the state stockpiles increases themarket supply, pushing prices down.Release supply from the government’s stockpile à surplus forms at current price àdownward pressure on prices as firms cut prices to rid the surplus à as price falls,quantity demanded rises while quantity supplied falls à price will continue to fall untilthe market eventually clears where quantity demanded exactly balances quantitysuppliedGraph + accompanying explanation2. Thesis: success <strong>of</strong> policy depends on PED and PESa. Relevance <strong>of</strong> PEDFor any given increase in ss, the extent by which prices fall depends on PED.Demand for food items tend to be price inelastic (more so for grains which are seen asstaple than meat). With demand for grains being more price inelastic, prices will haveto fall more to induce a large enough increase in quantity demanded to clear themarket <strong>of</strong> the surplus created by the release <strong>of</strong> supply from the government’sstockpile. The converse is true <strong>of</strong> meat.Graph + accompanying explanationb. Relevance <strong>of</strong> PESFor any given demand curve and increase in supply, the extent by which prices falldepends on PES.Supply <strong>of</strong> meat is likely to be more price inelastic compared to supply <strong>of</strong> grains. Whenthe government releases frozen meat from its stockpile, a surplus <strong>of</strong> meat (frozen andfresh) is formed at the original price. For the farmers, the need for refrigeration makesit costly for them to store meat. To continue to hold on to their stocks <strong>of</strong> animalsinvolves costs on a daily basis. For these reasons, farmers are more prepared toaccept lower prices for meat products in the face <strong>of</strong> a surplus. In contrast, it is cheaperto store grains. When there is a surplus, farmers would more readily store grainshoping to sell at higher prices in the future than to accept lower prices at the currentpoint in time. Quantity supplied <strong>of</strong> grains will therefore fall by a larger extent inresponse to any given fall in price as compared to meat.Graph + accompanying explanation


3. Antithesis: success <strong>of</strong> policy does not just depend on PED / PESAbove analysis assumes other factors constant.In reality, the policy is likely to result in unintended consequences and second-roundeffects which will affect its success in stabilising prices.a. Holding down the price <strong>of</strong> the good in the country relative to other countries may attract buyers from other (neighbouring) countries Without any policy to restrict foreigners from buying up the supplies in the domestic market or to stop domestic firms from exporting the grains and frozen meat to foreign markets to take advantage <strong>of</strong> the price differential à arbitrage will reduce the domestic supply, counter-­balancing the govt’s efforts to increase domestic supply by releasing grains and frozen meat from its stockpile à reduce effectiveness <strong>of</strong> policy (Graph + accompanying explanation) b. An important determinant <strong>of</strong> success: expectations Releasing <strong>of</strong> supply from stockpile à may help to calm fears <strong>of</strong> further price increasesà reduce hoarding à fall in dd, increase in ss by private sellers à help to furthermoderate price increase(Graph + accompanying explanation)4. EvaluationSuccess <strong>of</strong> policy <strong>of</strong> releasing ss from stockpile depends very much ona. how much the government is able to release from its stockpile. The larger the size <strong>of</strong> the stockpile, the stronger the ammunition the government has to fight the price increase. For example, even when the PED <strong>of</strong> meat is more elastic, the policy can still be effective in bringing down prices if the government can release enough frozen meat from its stockpile, i.e. if the supply increases by a large enough amount, there can still be a substantial reduction in prices. (Graph + accompanying explanation) b. whether the government is able to stem arbitrage e.g. effective enforcement <strong>of</strong> export bans


Mark SchemePart (a)Knowledge, Application, Understanding and AnalysisL37-10L24-6L11-3For an answer which• displays depth <strong>of</strong> economic analysis for most part <strong>of</strong> the essay • applies accurately to the given context • analyses the effects on both markets For an answer which• contains some economic analysis though not consistently applied throughout • is largely theoretical with limited application to the given context OR• applies sound economic analysis but lacks scope – analysed only one <strong>of</strong> the goods [cap at 5m] For an answer which• is largely irrelevant with no indication that the meaning <strong>of</strong> the question has been properly grasp e.g. no application <strong>of</strong> the PED concept or no links to consumer expenditure • is largely descriptive and/or contains gross conceptual mistakes Part (b)Knowledge, Application, Understanding and AnalysisL39-11L26-8L11-5For an answer which• displays depth <strong>of</strong> economic analysis for most part <strong>of</strong> the essay • makes effective use <strong>of</strong> diagrams and other tools <strong>of</strong> economic analysis • applies accurately to the given context • analyses both thesis and antithesis arguments • (at the lower end <strong>of</strong> the mark range) may contain some inaccuracies e.g. explains correctly the relevance <strong>of</strong> PED but not PES For an answer which• presents both sides <strong>of</strong> the arguments but • is not consistently analytical throughout, limited in use <strong>of</strong> the tools <strong>of</strong> economic analysis such as diagrams • is largely theoretical with limited application to the given context OR• applies sound economic analysis but lacks balance For an answer which• is largely irrelevant with no indication that the meaning <strong>of</strong> the question has been properly grasp


EvaluationE2For a reasoned judgement3-4E1For an unexplained judgement1-2(5) 2011 RI Prelim H1 Paper Q3As Indian economist Amartya Sen has said, “The invisible hand <strong>of</strong> the market has <strong>of</strong>tenrelied heavily on the visible hand <strong>of</strong> government.”(a) Explain how the invisible hand works in the allocation <strong>of</strong> resources. [10](b) Evaluate the view that a government such as that <strong>of</strong> Singapore should subsidize entry tomuseums and tickets to watch arts performances. [15]<strong>Suggested</strong> Answer Outline(a) Explain how the invisible hand works in the allocation <strong>of</strong> resources. [10]IntroductionDefine invisible hand– price mechanismooPlays an allocative function as it allocates resources to the production <strong>of</strong>different types <strong>of</strong> goods.It acts as a rationing mechanism by synchronizing decisions by buyers andsellers.As resources are scarce relative to the insatiable demands <strong>of</strong> human wants, economies areconcerned with basic questions <strong>of</strong> allocation:i. What and how much to produce?ii. How to produce?iii. For whom to produce?Development:i. What and how much to produce?Insufficient resources to produce enough goods and services to satisfy human wants


è economy must make a choice on the types <strong>of</strong> goods and services that it wants tomake available to the country.è For example, an economy has to decide on the different quantities <strong>of</strong> consumergoods and capital goods.o determined jointly by firms and consumers through the signaling role <strong>of</strong> pricesand the pr<strong>of</strong>it motive.§ price <strong>of</strong> a good reflects the value placed on it by consumers.§ Consumers indicate their tastes and preferences to firms by the pricethey are willing to pay for the goods.§ Producers would only produce goods which consumers demandbecause they want to maximize pr<strong>of</strong>its.§ The higher the price <strong>of</strong> a good, the more the producers will supply thatgoodè In this way, the price acts as a signal telling the producers what to produce and howmuch <strong>of</strong> the good to produceo Thus determines the allocation <strong>of</strong> resources among various goods.ii. How to produce?What method <strong>of</strong> production should the scarce resources be used to produce the desiredcombination <strong>of</strong> goods/services as efficiently as possible.è For example, a manufactured good can either be produced by capital intensivemethods (where there is little use <strong>of</strong> labour and greater use <strong>of</strong> machines) or labourintensive methods (where greater use is made <strong>of</strong> labour).o involves the organization <strong>of</strong> production.è Main aim: to achieve the least cost combination guided by relative factor prices.iii. For whom to produce?This problem concerns the distribution <strong>of</strong> a country's national income.è resources are scarce, no society can satisfy all the wants <strong>of</strong> its people.è finished goods/services have to be distributed to households, firms and thegovernment.o How the limited supply <strong>of</strong> final goods/services produced is allocated amongthe members <strong>of</strong> society?o price acts as a rationing mechanism in a market economy and distributes theoutput only to people who are able and willing to pay for the good.§ This in turn depends on the purchasing power and the value thatpeople place on the good.Brief explanation <strong>of</strong> demand and supply analysis with the help <strong>of</strong> a diagram- Determination <strong>of</strong> price and quantity by the interaction <strong>of</strong> demand and supply - Any market disequilibrium à market adjustment until equilibrium price and quantity achieved à satisfaction <strong>of</strong> both buyers and sellers maximized -


LevelsL3(7m – 10m)L2(5m – 6m)L1(1m - 4m)Descriptors- Clear explanation <strong>of</strong> all 3 basic questions <strong>of</strong> allocation anddemand-supply analysis- Sufficient depth/ scope- Insufficient depth/scope in addressing the basic questions <strong>of</strong>allocation.- Some attempt in addressing the demand-supply analysis- Some errors in explanation- Poor and erroneous explanation with little understanding <strong>of</strong> howthe price mechanism allocates resources.(b) Evaluate the view that a government such as that <strong>of</strong> Singapore should subsidize entry tomuseums and tickets to watch arts performances. [15]Introduction- the shortcomings <strong>of</strong> the price system as a mechanism to allocate resources explains the economic rationale for government intervention in a market economy. - presence <strong>of</strong> merit goods and positive externalities in the case <strong>of</strong> museums and arts performances Thesis: Govt should subsidize- explanation <strong>of</strong> the type <strong>of</strong> market failure for museums and arts performancesà positive externality & merit goodsPositive externalities- occurs when third parties in the society benefit from the consumption <strong>of</strong> a commodityor service by private individuals- For museums and arts performances à there exist external benefits such as morecultured and refined society, inculcate ability to appreciate arts, encouraged localartists showcase their work, boost to tourism and hospitality industry.o Divergence between MSB and MPB (MSB>MPB)o Left to the free market, underconsumption as individuals (producers andconsumers) disregard the external benefitso Therefore, they will choose to equate their marginal private costs andmarginal private benefits, ignoring the marginal external benefit generated.This is shown by point E 2 on Figure 1.o allocative efficiency is not achieved at the free-market equilibrium.§ Society seeks to maximise the welfare <strong>of</strong> the society and would thusprefer to consume/produce where MSB=MSC, as illustrated by point Eon both diagrams, illustrating shaded area as deadweight losses


Cost/BenefitMPC = MSCEE 2MSBMPB0Q 1Q sFigure 1QuantityMerit Goods- Individuals may not act in their own best interests because <strong>of</strong> imperfect information about the benefits that can be derived from the consumption <strong>of</strong> merit good like museums and art performances. - Merit goods are those goods and services that the government thinks are “beneficial” for the individual consuming them as individuals are likely to underestimate their personal benefits in consuming merit goods. - Left wholly to the private sector, merit goods will be under-­‐consumed because individuals do not understand or appreciate the good effects (that can be seen only in the long term) The above shows allocative inefficiency under an unregulated free mkt system. Marketfailure results. Thus, govt intervention may be necessary to reduce the allocativeinefficiency.Explanation <strong>of</strong> how subsidy works to correct the market failureA subsidy is a payment made either to a firm or to a consumer when the firm produces orwhen the consumer buys a good or service.- Can be used to correct the problem <strong>of</strong> positive externalities and merit goods.


Subsidies to ConsumersWith government intervention, a subsidy <strong>of</strong> AB per unit to consumers will shift the demandcurve from MPB to MSB as consumers are ‘encouraged’ to increase consumption, resultingin the socially optimal level, Q 2 where MSC = MSB .Cost/benefitMSC = MPCD• BC• AMSB = MPB + MEB0Q 1 Q 2MPBQuantityFigure 2Subsidies to ProducersSubsidies given to producers reduce the cost <strong>of</strong> supplying products. This is shown in Figure3. The equilibrium without government intervention is at output Q 1 where MPC = MPB or D 1= S 1 . In this case, marginal external benefit is added to the MPB curve to give the MSB ormarginal social benefit curve.If the government subsidises the production <strong>of</strong> arts performances, the supply curve moves tothe right from S 1 to S 2 , which equals MPC minus the subsidy. The marginal cost <strong>of</strong> supplyingthe good is reduced by the amount <strong>of</strong> subsidy and the vertical distance GH is equal to thevalue <strong>of</strong> the subsidy provided. Producers will be able to sell output Q 2 at a price <strong>of</strong> P 3 whichis where D 1 crosses S 2 . The production and consumption level would be socially optimal.


Cost/benefit S 1 S 2 P 2 P 1 P 3 F HG = subsidy MPC H Figure 3: Governmentsubsidies to producersD 1 = MPB MSB Q Quantity In both cases, the under-allocation <strong>of</strong> resources would be corrected as positive externality issaid to have been ‘internalised’.The net gain to society from the subsidy or the elimination <strong>of</strong> deadweight loss is shown bythe triangle DBC in figure 2 and triangle GHF in figure 3.- Desirable as it is flexible and market based solution In the case <strong>of</strong> Singapore (National Arts Council)- Fundings to the performing arts was provided through a comprehensive grants framework. - Under the 2-­‐year Major Grant Scheme, ten <strong>of</strong> Singapore’s top performing arts companies received a total <strong>of</strong> $2.8 million in FY 2009. These were Singapore Dance Theatre, T’ang Quartet, Singapore Lyric Opera, The Finger Players, The Necessary Stage, Drama Box, TheatreWorks, The Theatre Practice, Singapore Repertory Theatre and Wild Rice. - The grants enabled these groups to o create and present quality programmes, o develop artistic practice and pr<strong>of</strong>essional expertise in their fields o facilitate their artistic growth and resource development Anti-thesis: Govt should not subsidizeLimitations <strong>of</strong> subsidy• The valuation <strong>of</strong> the external benefit generated at the social optimal output level is inpractice a difficult task. An overestimation will lead to over consumption <strong>of</strong> the good. Anunderestimation will lead to less than social optimal consumption but at this level it is atleast more than the market equilibrium level.• High government expenditure is required to provide for the subsidy required.


ooEspecially when demand is price inelasticStrain on govt finances especially with ageing population anddiversion <strong>of</strong> resources away from other areas like education,national defence, healthcare (need to examine the trade<strong>of</strong>fs/opportunitycosts incurred)- Reason why govt shdn’t subsidize- If perceived by government to generate zero or negligible positive externality à shouldbe left to the market mechanismConclusion: - Need for govt intervention if there are positive external effects from entry to museums and watching arts performances. - Other than subsidies, other solutions could possibly include campaigns and direct provisions. - If positive benefits are perceived to be low or negligible, then it is best left to the price mechanism to do the allocation <strong>of</strong> resources. LevelsL3(9m - 11m)L2(6m - 8m)L1(1m - 5m)E2(3m - 4m)Descriptors• Use <strong>of</strong> appropriate framework/concepts in analysiso MSB/MSC framework• In-depth discussion <strong>of</strong> questiono Appropriate and well structured thesis/antithesiso Other reason/s for not intervening in the market formuseum and arts performances• Good depth <strong>of</strong> analysiso Well-elaborated cause-effect relationship• Use <strong>of</strong> appropriate framework/concepts in analysis- MSB/MSC framework• Sufficient scope <strong>of</strong> coverage- Thesis/antithesis structure• Sufficient depth <strong>of</strong> analysis- primary and important cause-effect relationships are explained• Some errors in analysis•• Lacks use <strong>of</strong> economics framework/concepts in analysis• Lacks scope <strong>of</strong> coverage- Thesis/antithesis structure• Irrelevant answer• Glaring conceptual errors• No explanation <strong>of</strong> the type <strong>of</strong> market failure• For an insightful and explained assessment based on economicanalysisE1(1m - 2m)• For an unexplained assessment <strong>of</strong> whether Singapore governmentshould subsidise entry to museums and arts performances


(6) 2011 NJC Prelim H1 Paper Q3(a) Explain how the presence <strong>of</strong> externalities results in inefficiency in the markets for airtravel and education.[10](b) Discuss measures the government can adopt to address the inefficiency in themarket for air travel.[15]<strong>Suggested</strong> <strong>Answers</strong>:(a) INTRODUCTIONThe free market fails to take into account the fact that there are externalities in theproduction and consumption <strong>of</strong> certain goods. In this case, when left to the free market, it willresult in a market failure situation where there is an overconsumption <strong>of</strong> air travel due to thepresence <strong>of</strong> positive externalities and an under-consumption <strong>of</strong> education due to thepresence <strong>of</strong> positive externalities, both <strong>of</strong> which are not taken into account by the privateconsumers.BODYIn the education market, consumers only take into account <strong>of</strong> their own private benefits <strong>of</strong>educating themselves (we assume that there is no externalities in the production <strong>of</strong>education, where Marginal Social Cost (MSC) = Marginal Private Cost (MPC) ). Theseprivate benefits include the ability to earn higher earnings currently and in future, the abilityto enrich oneself with the power <strong>of</strong> knowledge etc.However, these consumers fail to take into account that when they educate themselves, theycan benefit society as well through spill over effects known as external benefits, whichinclude a society that becomes more productive as a whole with a more educated workforcethat can help the country to generate economic growth via foreign and local investments, aswell as reducing the level <strong>of</strong> social problems such as crime rates as a country becomesmore educated.Therefore we see that Marginal Social Benefit (MSB) = Marginal Private Benefit (MPB) +Marginal External Benefit (MEB). Because individuals do not take into account the MEB, wesee a divergence between the MSB and MPB. Looking at Figure 1, individuals only equateMPB and MPC, and consume up to a level Q 0 . However the socially optimal point is at Q * ,where the society’s consumer welfare will be maximise, therefore the under-consumption <strong>of</strong>education results in a deadweight loss (ABC) to society that is captured by no one.


Therefore the market is said to be inefficient when the socially optimal amount <strong>of</strong> educationQ * is not achieved due to an under-consumption <strong>of</strong> education from society’s point <strong>of</strong> view,and this results in market failure.Costs/Benefits MPC = MSC MEB A B C MSB MPB Q p Q * Qty (Education) In the air travel market, consumers only take into account <strong>of</strong> their own private benefits <strong>of</strong>flying. These private benefits can be in terms <strong>of</strong> the satisfaction from flying, and the feelgood factor from going on a holiday, or even to businessmen where the benefits <strong>of</strong> flying isimmense, it can make a difference between closing a multi-billion dollar deal overseas.Again we assume that there are no externalities in the production <strong>of</strong> air travel.However these consumers fail to take into account the external costs <strong>of</strong> flying, such as thepollution and noise generated to the local and global community. In other words, theyoverestimate the benefits <strong>of</strong> flying. In this case we see that MSB = MPB – MEC. Again thereis a divergence between the MSB and MPB. Looking at Figure 2, individuals only equateMPB and MPC, and consume up to a level <strong>of</strong> Q p, where obviously the socially optimal level islesser at Q * if the external costs are taken into account. The over-consumption <strong>of</strong> flyingresults in a deadweight loss (ABC) to society. The market is inefficient as the overconsumption <strong>of</strong> flying results in wastage <strong>of</strong> resources, and this results in market failure.


Costs/Benefits C MPC = MSC A B MEC MPB MSB Q * Q p Qty (Air travel) CONCLUSIONTherefore we see that the presence <strong>of</strong> externalities (in this case positive and negativeexternalities in consumption) can result in misallocation <strong>of</strong> resources if left to the freemarket. The government needs to step in to correct these market failures so thatsociety’s welfare is maximised.LevelsL3(7m – 10m)L2(5m – 6m)L1(1m - 4m)Descriptors- Clear explanation <strong>of</strong> and identification <strong>of</strong> the externality problemin both the education and air travel market- Clear graphical analysis.- Sufficient depth/ scope in explaining how market failure occurs.- Insufficient depth/scope in addressing the basic questionsexternalities- Some attempt in identifying at least one correct externality in thecontext <strong>of</strong> the question for education and air travel- Some errors in explanation and diagram showing the externalityproblem- Poor and erroneous explanation with little understanding <strong>of</strong> howexternalities causes market failure\


(b) INTRODUCTIONTo address the problem <strong>of</strong> negative externalities in <strong>of</strong> air travel which results in overconsumption,the government can adopt a few measures to try to achieve the sociallyoptimal level. While this may be possible in theory , in practice these measures havetheir limitations as well.BODYFirstly, the government can impose taxes. In many countries these are flight taxesimpose and these taxes are included when an air-ticket is purchased. The flight taxesincreases the cost <strong>of</strong> production for airlines, forcing them to take into account <strong>of</strong> theexternal costs <strong>of</strong> flying. In Figure 3, this will cause a divergence in the MSC and MPCline. MSC will shift leftwards. If implemented correctly and accurately, the tax should beequivalent to external costs <strong>of</strong> flying. In this way, we will reach a socially optimal level <strong>of</strong>flying. The external costs (MEC) which will cause the MSC curve to shift upwards will hitthe point where the socially optimal level <strong>of</strong> output is achieved again at Q * . This is thepoint where the socially optimal level is achieved.Costs/Benefits MSC MPC Tax MPB MSB Q * Q p Qty (Air travel)


Furthermore, if we assume that the demand for flights is price elastic, all we need is justa small increase in tax, and it can result in a more than proportionate fall in quantity <strong>of</strong>flights. This will render the tax policy even more effective.However, there are limitations. It is difficult to exactly pinpoint how much <strong>of</strong> the flight taxis to be charged. Any under-estimation <strong>of</strong> the tax will result in the tax policy ineffective asthe socially optimal level has not been reach yet, or it can result in a different problemwhen over-estimating: under-consumption. It also takes time to pass the legislation toapprove <strong>of</strong> the new tax policy, which may face fierce resistance. The difficulty <strong>of</strong>estimating the exact tax amount is enough to put policy-makers <strong>of</strong>f.An example can be seen when the European Union intended to impose a flying taxbased on the distance an airline travelled from its point <strong>of</strong> departure. Critics claim that itis unfair and penalises those airlines which have to travel longer distances from Asia orAustralia.Another solution could be in the form <strong>of</strong> direct regulation. Governments could pass lawsthat mandate airlines to limit the amount <strong>of</strong> pollution that they can generate via tradablepermits. Tradable permits works in such a way: If an airline exceeds its stipulatedamount <strong>of</strong> pollution it generates, it can purchase excess permits at market price, whichcan be very expensive, from other airlines which manage to keep its pollution levelsbelow the stipulated level. The whole point about the tradable permits is to provide theairlines with a strong incentive to invest in better airplane technology that uses fuel moreefficiently and generate less air and noise pollution.However, the problem with tradable permits is that it is difficult to enforce. Governmentshave to think <strong>of</strong> how many permits are they going to give to each airline to ensurefairness. Do larger airlines get more permits? How do we define the size <strong>of</strong> the airlines?Furthermore, it is difficult to monitor and enforce the tradable permits system, becausethe commercial planes <strong>of</strong> airlines are not permanently based in one country, so how cangovernments keep track <strong>of</strong> where the planes are and how much are they polluting?Furthermore, some airlines would rather pay the fines associated with exceeding theamount <strong>of</strong> pollution permitted by the tradable permits, since the benefits <strong>of</strong> flying moreand longer will earn them more revenue, which can outweigh the costs <strong>of</strong> paying thefines.The third solution could be in the form <strong>of</strong> a total ban. Governments could perhaps banflights below a certain distance between two places. This is possible because planesflying between two nearby places always fly at a lower altitude than long distance flights,causing much more pollution. In this way, the outright ban will ensure no pollution takesplace within the area where the ban is enforced.


However, a total ban does not makes sense too because there may be almost nosubstitutes to flying between two places, unless the land or even sea transport systembetween these two places is excellent that it almost takes the same time to reach theother destination as it would be if one was taking a plane. In some countries, the poorland transport system means that flying may be inevitable. Furthermore, a total banwould result in a total loss <strong>of</strong> the possible social benefits <strong>of</strong> flying to society. In Figure 4,we see that a total ban will result in a greater welfare loss <strong>of</strong> Area B than if nothing wasdone to regulate flying at Q P .Costs/BenefitsYMSC = MPC + MECMPCBXAZ0 Q SMPB = MSBOutput levelQ PFigure 4To sum it up, out <strong>of</strong> the three solutions proposed, the tax system seems more reasonableand practical to implement. In this globalised world, flying becomes more and moreindispensible and the demand for flying is price inelastic. Therefore governments canimplement the tax system, collect more tax revenue since the increase in cost <strong>of</strong> flyingresults in a less than proportionate fall in the amount <strong>of</strong> flying, and at the same time allowsthe market to move towards the socially optimal amount <strong>of</strong> flying. The extra tax revenuecould then be used to subsidise plane manufacturers through research and developmentsuch as Boeing and Airbus to develop new commercial jets that are environmentally friendlyand cost efficient.The other two solutions are harder to implement because it becomes a nightmare to even tryto predict the amount <strong>of</strong> permits deemed desirable, and enforcement is almost impossible.Lastly, banning <strong>of</strong> flights is almost impossible, because in the free market, market forcesshould be dictating whether people decide to fly or not, not governments.


LevelsL3(9m - 11m)L2(6m - 8m)L1(1m - 5m)E2(3m - 4m)Descriptors• Use <strong>of</strong> appropriate framework/concepts in analysiso MSB/MSC framework• In-depth discussion <strong>of</strong> questiono Appropriate and well structured thesis/antithesiso Logical reasons to support why these solutions must beused with caution to correct inefficiency in air travelmarket.• Good depth <strong>of</strong> analysiso Well-elaborated cause-effect relationship with diagrams• Use <strong>of</strong> appropriate framework/concepts in analysis- MSB/MSC framework• Sufficient scope <strong>of</strong> coverage- Thesis/antithesis structure• Sufficient depth <strong>of</strong> analysis- primary and important cause-effect relationships are explained• Some errors in analysis•• Lacks use <strong>of</strong> economics framework/concepts in analysis• Lacks scope <strong>of</strong> coverage- Thesis/antithesis structure• Irrelevant answer• Glaring conceptual errors• No explanation <strong>of</strong> the how the solutions can solve market failure• For an insightful and explained assessment based on economicanalysisE1(1m - 2m)• For an unexplained assessment on why one <strong>of</strong> the proposedsolutions is the best


(7) 2011 IJC Prelim H2 Paper Q3


(8) 2010 RVHS Prelim H2 Paper Q4a) Explain how a change in the U.S interest rate can lead to a lower circular flow <strong>of</strong> incomeand expenditure for Singapore. [12]b) Examine whether a lower circular flow <strong>of</strong> income and expenditure necessarily mean alower standard <strong>of</strong> living. [13](a)Interest rate is the price charged for the use <strong>of</strong> money. It represents the cost <strong>of</strong> borrowing aswell as the opportunity cost <strong>of</strong> consumption. On the same note, the circular flow <strong>of</strong> incomeand expenditure depicts the expenditure on goods and services by the consumers, firms,government and the foreigners to the firms which in turn determine the income payments tothe factors <strong>of</strong> production, namely land, labour, capital and entrepreneurship. An increase ininterest rate can reduce the aggregate expenditure via its components and can reduce thecircular flow <strong>of</strong> income and expenditure.As Singapore is a small and open economy, whenever U.S interest rate increases, there willbe a hike in the Singapore’s interest rate given the huge capital flows and excessivechanges to the exchange rate <strong>of</strong> Singapore.Consumption is spending by households on goods and services, with the exception <strong>of</strong>purchases <strong>of</strong> new housing. With higher cost <strong>of</strong> borrowing due to the increase in interest rate,consumers postpone spending on consumer durables like cars, furniture and consumerelectronics. In addition, those with mortgage payments, having to fork out more to pay for theinterest, will have less disposable income to spend. Moreover, with the increase in interestrates, there is more incentive to save given the higher opportunity cost <strong>of</strong> consumption.Higher interest rate will also have negative impact on asset prices, hence households will bediscouraged to spend as they feel less wealthy. These factors will cause consumption todecrease.Investment is spending on capital equipment, inventories and structures, includinghousehold purchases <strong>of</strong> new housing. Firms <strong>of</strong>ten finance their purchases <strong>of</strong> equipment,inventories and structures through loans. With reference to Fig 1, when interest rateincreases from OR 1 to OR 2 , projects I 1 I 2 which are viable in the past becomes no longerviable, hence investment falls. In addition, the higher cost <strong>of</strong> borrowing also discourages thebuying <strong>of</strong> property, hence reduces the level <strong>of</strong> investment.


Besides the above, an increase in the interest rate can also cause government expenditureto fall. In addition, the excessive strengthening <strong>of</strong> the exchange rate due to the huge capitalinflow can also cause net exports to fall.MEC / interest rate R 2 Fig. 1: Impact <strong>of</strong> an increase in interest rate on investment R 1 MEI O I 2 I 1 investment With reference to Figure 2, given a fall in the components <strong>of</strong> AE due to an increase ininterest rate, the AE function shifts down from AE 0 to AE 1. At this level <strong>of</strong> output Y 0, the level<strong>of</strong> output produced is more than AE. Hence, there is a piling up <strong>of</strong> inventories and will signalto the producers to lower production. This lower production will mean less factors <strong>of</strong>production will be used, hence leading to a lower circular flow <strong>of</strong> income and expenditure.AE AE 0 AE 1 Y 1 Y 0 NY Figure 2: Impact <strong>of</strong> a fall in AE


In conclusion, a change in the U.S interest rate, specifically a hike in interest rate, can leadto a lower circular flow <strong>of</strong> income and expenditure.Level 3For a developed answer that shows the effect <strong>of</strong> an increase ininterest rate on Singapore’s circular flow <strong>of</strong> income and expenditure.10-12mAnswer should address relationship between U.S interest rate andinterest rate <strong>of</strong> Singapore. Answer should also consist anexplanation <strong>of</strong> how a fall in AE will lead to a fall in NY.Level 2For an undeveloped answer that shows the effect <strong>of</strong> an increase ininterest rate on Singapore’s circular flow <strong>of</strong> income and expenditure5-9mLevel 1 A smattering <strong>of</strong> valid points 1-4m(b)Standard <strong>of</strong> living in a society is a broad concept that refers to the well being <strong>of</strong> thepopulation. It encompasses both the quantitative and qualitative aspects <strong>of</strong> life. Thequantitative aspect focuses on the material well being whereas the qualitative aspectfocuses on the non-material well being.A lower circular flow <strong>of</strong> income and expenditure can imply deterioration in the quantitativeaspect <strong>of</strong> standard <strong>of</strong> living since it indicates a lower amount <strong>of</strong> goods and services peoplecan enjoy. However, it does not conclusively mean that the quantitative aspect <strong>of</strong> standard <strong>of</strong>living is lower. In fact, it is possible for SOL to be higher or remained at the same leveldespite the lower circular flow <strong>of</strong> income and expenditure, given a more accurate datacollection, a lower population, a lower price level, and a more equitable distribution <strong>of</strong>income or an improvement in the qualitative aspect <strong>of</strong> life.First, given that NY per capita measures the level <strong>of</strong> material well-being <strong>of</strong> the averagecitizen, it can be a more comprehensive indicator to measure SOL. Hence, another possibleargument that SOL may not have fallen even though the NY is falling is because thepopulation could have fallen at a more rapid rate, culminating in an increasing NY per capita.Population in a country could have fallen because <strong>of</strong> some successful family planningprogram being introduced. This will subsequently lead to a higher level <strong>of</strong> quantity <strong>of</strong> goodsand services available to each person on average and thus, does not necessarily mean a


lower standard <strong>of</strong> living. This is especially true for developed economies where the birthrates seem to be at record low. However, on the other hand, it seems unlikely thatpopulation will decline at a rapid rate given the modern developments in healthcareservices, as evident by a low infant mortality rate worldwide.Second, prices could have fallen more than the fall in nominal income, resulting in a rise inreal income. This is especially applicable to the situation <strong>of</strong> a recession when the economymay face periods <strong>of</strong> deflation as well. It does not necessarily mean that SOL hasdeteriorated when real NY has fallen because real income <strong>of</strong> the people could haveincreased and they can buy more goods and services than before.In addition to the above quantitative aspects, improvements made in the qualitative aspectscan also bring about higher SOL. These include better quality <strong>of</strong> goods and services, shorterworking hours, better working conditions, a more politically and socially stable society andhigher quality <strong>of</strong> lifestyle, which cannot be inferred from the NY figure.The quality <strong>of</strong> goods and services consumed may have improved even though the NY hasdecreased. Such improvements in products and customer services help bring about a higherlevel <strong>of</strong> satisfaction which contributes to the well being <strong>of</strong> the society. For example, althoughNY <strong>of</strong> a particular country may have decreased, the consumers may still get to enjoy a widerrange and better quality <strong>of</strong> the goods like hand-phones and laptops etc as the producerscontinue to innovate in the hope <strong>of</strong> stimulating the economy. This is highly probable giventhe level <strong>of</strong> high competition in the market. When the level <strong>of</strong> consumption isdeclining, producers may seek to develop their products instead so that they canretain their market share.Standard <strong>of</strong> living can also improve if shorter working hours are implemented. For instance,5- day workweek for the civil service recommended by the Singapore government someyears ago has certainly help to improve the quality <strong>of</strong> life. In addition, quality <strong>of</strong> life can alsoimprove if the relevant authorities <strong>of</strong> the country ensure better working and living conditionsin terms <strong>of</strong> safety and cleanliness.Another aspect that can cause SOL to improve despite a lower NY is a moreenvironmentally friendly community and greener society. This can come about by a higherliteracy rate and society being more aware about green issues. In addition, with a betterenvironment, it helps increase life expectancy, bringing about higher quality <strong>of</strong> life.Regulation like clamping down on motorists emitting large amounts <strong>of</strong> fumes can alsohelp bring about a better environment.To conclude, a lower circular flow <strong>of</strong> income and expenditure does not necessarily mean alower SOL. This is because the figures fall short <strong>of</strong> being good indicator in measuring thequantitative aspect. In addition, the figures have neglected the qualitative aspect. Other


indicators like the Gini ratio, pollution index and length <strong>of</strong> working hours should besupplemented to better interpret the statistics especially for standard <strong>of</strong> living.Alternatively, other indicators like the human development index and measure <strong>of</strong>economic welfare can be used instead.Level 3For a developed answer that shows a lower NY does notnecessarily mean a lower SOL. Answer should include bothquantitative and qualitative aspects.7-9Level 2For an undeveloped answer that shows a lower NY does notnecessarily mean a lower SOL. Answer should include bothquantitative and qualitative aspects.4-6For a developed answer that shows a lower NY does notnecessarily mean a lower SOL, but the answer only covers oneaspect <strong>of</strong> SOLLevel 1 Smattering <strong>of</strong> valid points. 1-3E2 For an evaluative assessment based on economic analysis 3-4E1For an unexplained assessment or one that is not supported byanalysis1-2


(9) 2011 VJC Prelim H2 Paper Q4Singapore’s real GDP growth was 13.9% and current account surplus growth was 32.8% in2010.a) Explain why real GDP and current account balance are indicators <strong>of</strong> living standards in Singapore. [8] b) Discuss the most appropriate policies that a government could adopt to achieve sustained economic growth. [17] Approach Define real GDP, current account balance and standard <strong>of</strong> living. Explain the use <strong>of</strong> real GDP per head as the key indicator <strong>of</strong> standard <strong>of</strong> living. Explain the great importance <strong>of</strong> trade to the Singapore economy. Since GDP = C+I+G+(X-­‐M) the size <strong>of</strong> NX as a percentage <strong>of</strong> GDP is <strong>of</strong> significance for Singapore. However, both are indicators only <strong>of</strong> the material welfare <strong>of</strong> the people. To have an indication <strong>of</strong> the quality <strong>of</strong> life, there must be other indicators. Brief Outline Define standard <strong>of</strong> living -­‐-­‐ the material and non-­‐material welfare <strong>of</strong> an individual or household. Real GDP Define Real GDP -­‐-­‐ the value <strong>of</strong> final goods and services produced within the country during a period <strong>of</strong> time usually a year after discounting for inflation. It is a measure <strong>of</strong> Singapore’s economic activity or output performance. It can indicate material welfare <strong>of</strong> an individual. Real GDP divided by the appropriate population gives real GDP per capita. Real GDP per capita is a key measure <strong>of</strong> the average living standards in Singapore. The strong growth in Real GDP means Singapore is producing more output domestically. It indicates higher income per head, the people can consume more final goods and services and thus improved (material) standard <strong>of</strong> living. When the economic pie gets bigger, it will facilitate redistribution <strong>of</strong> incomes. Whereas a contraction <strong>of</strong> real GDP figure indicates that the Singapore economy has shrunk (e.g.2009-­‐Singapore went into a recession) and the people are worse <strong>of</strong>f, having lower per capita income. Current account balance The current account balance is divided into 2 main items: visible balance and invisible balance. Singapore is a small and open economy which is strategically located. The nature <strong>of</strong> the economy compels it to be trade driven. Total trade makes up more than 300 percent <strong>of</strong> GDP and net exports about 30% <strong>of</strong> GDP. GDP=C+I+G+(X-­‐M). Thus, any changes in the current account balance have great bearing on the country’s real GDP.


It is important for Singapore to continue having a current account surplus as export is the largest component <strong>of</strong> GDP. When Singapore records a strong current account, its GDP expands boosted by export revenue. (given that GDP=C+I+G+X-­‐M).There will be more income for the people, more consumption and a higher living standards. This is also an indication that the country is not spending beyond its means. However, there should be other indicators on nonmaterial welfare. Some examples are UN’s Human Development Index and MEW to have a more accurate measure <strong>of</strong> living standards. Mark SchemeL1L2L3A sketchy explanation <strong>of</strong> Real GDP and current account balance asindicators <strong>of</strong> living standards in SingaporeAn adequate explanation <strong>of</strong> both Real GDP and current account balanceas indicators <strong>of</strong> living standards in SingaporeA good explanation <strong>of</strong> either Real GDP or current account balance asindicators <strong>of</strong> living standards in SingaporeA good explanation <strong>of</strong> both Real GDP and current account balance asindicators <strong>of</strong> living standards in Singapore1-23-56-8b) Discuss the most appropriate policies that a government could adopt to achieve sustained economic growth. (17) ApproachThe focus <strong>of</strong> the question is the choice <strong>of</strong> the most appropriate policies on achieving themacroeconomic goal <strong>of</strong> sustained economic growth. Discussion on appropriateness willinclude that <strong>of</strong> effectiveness and suitability. For actual growth to be sustained in the long-run,there must also be increase in potential output. Discuss the mechanism, suitability and theeffectiveness <strong>of</strong> demand-side and supply-side policies in achieving actual and potentialgrowth. Discuss how well each policy works and their limitations given the different natureand the current state <strong>of</strong> the economies. Use country examples to illustrate.Brief OutlineDefine actual and potential growth. To have sustained economic growth, there must bepotential growth.


Explain how policies bring about actual growth work.1) Expansionary fiscal policy to raise ADInvolves raising government expenditure and/or reducing taxes to increase AD and leadingto a multiple rise in NY.Consider the appropriateness <strong>of</strong> the policy.Less effective for small and open economies like Singapore where G, domestic C and I aresmall relative to X. The extent <strong>of</strong> change in AD is therefore smaller. Also, there is a weakmultiplier effect due to high leakages because <strong>of</strong> high MPS and/or MPM. Whereas for largeeconomies like the United States fiscal policy is much more effective in stimulating economicgrowth as it has a large proportion <strong>of</strong> G and domestic C, I relative to X and a large multiplier.As such, Singapore uses fiscal policy to deliver supply-side initiatives to promote potentialgrowth. For example during the 2009 Financial Crisis.In a recession, fiscal policy is more effective than monetary policy in stimulating growth asgovernment spending is a direct injection into the economy and it can be targeted. But thenagain there are other limitations like time-lag and crowding out effect. Also in a recession,large tax cuts may not bring about huge private investment if business expectations arepoor.2) Expansionary monetary policy to raise ADInvolves increasing money supply or lowering interest rate to encourage C and I thusincreasing AD.Consider the appropriateness <strong>of</strong> the policySuch a policy is ineffective for a small and open economy like Singapore which is an interestrate taker. Whereas for large economies, monetary policy is more appropriate given a largedomestic economy.In a recession, due to low consumer and business confidence, C and I may not increase withfall in interest rate. Thus, fiscal policy is more effective. But a decreased interest rate willallow the government to borrow to finance a deficit budget. Thus, expansionary monetaryand fiscal policies in a recession can well complement one another.


3. Exchange Rate policyDepreciation will cause exports to be cheaper in foreign currency and imports moreexpensive in domestic currency. Assuming Marshall-Lerner Condition holds, (X-M) riseleading to actual growth.Consider the appropriateness <strong>of</strong> the policyDuring weak economic climate, the exchange rate can be weakened to stimulate exports.But it has its limitations especially for small, open and export-driven economies as it causescost-push inflation and may encourage competitive depreciation, leading to negativefeedback effects. Thus, the use <strong>of</strong> exchange rate policy to stimulate actual growth is limitedin such economies. Preventing the currency from strengthening to promote exports has beensuccessful in China. If there is a global recession, such a policy is ineffective as globaldemand is low.4. Trade PolicyThe support <strong>of</strong> freer trade will expand export markets allowing for production on a largescale and goods becoming more price competitive. E.g. signing <strong>of</strong> FTAsThis is <strong>of</strong> importance for economies with small domestic markets and which are resourcescarce, for example Singapore. These economies are dependent on both imports andexports. The increase in the current account surplus will boost actual growth. However,some countries may also turn to protectionism during weak economic climate which makes itdifficult to reap the benefits from free trade policy.Supply-side policies to boost potential growth (rightward shift <strong>of</strong> LRAS)Once the economy attains full employment, further increase in real output can only occur ifthere is a rise in productive capacity in the economy (potential growth). To have sustainedeconomic growth, there must be potential growth which is promoted with supply-sidepolicies. They can be market-oriented or direct interventionist. The range <strong>of</strong> supply-sidepolicies are privatization, deregulation, liberalisation, weaken trade unions, reduction <strong>of</strong> taxrates, government providing or subsidising training programmes, government fund/subsidiesR&D, reduction <strong>of</strong> personal tax rates, building <strong>of</strong> infrastructure etc. But such policies mayhave long gestation period and may be a financial burden on the government.Consider the appropriateness <strong>of</strong> the policy.Supply-side policies are paramount and ongoing for all economies aspiring for sustainedgrowth. But supply-side policies may be more effective in bringing about sustained growthfor small and open economies given the limitations <strong>of</strong> demand management policies


ConclusionTo have sustained actual growth there must be potential growth. Once an economy reachesfull employment, the productive capacity has to be increased to allow for further actualoutput to increase. Supply-side policies targeting the increase in potential output is ongoingfor all economies. Demand side policies targets actual growth. Which policies work best willvery much depend on the current state <strong>of</strong> the economy and the nature <strong>of</strong> the economy. Forsmall and open economies, demand-side policies like the conventional monetary and fiscalpolicies may be inappropriate or less effective. Such economies use supply-side policiesmore actively to achieve economic growth. In a recession, fiscal policy can be moreappropriate than expansionary monetary policy. The latter works well in an inflationarysituation. To achieve sustained growth, countries will usually use a mix <strong>of</strong> policies.L1L2Definitions <strong>of</strong> actual and potential growth. Identify some AD-side and ASsidepoliciesExplanation <strong>of</strong> how dd-management policies can boost AD and SS-sidepolicies boost AS with economic analysis but some gaps in explanationE.g. <strong>of</strong> gaps- considered only actual growth but not potential growth (vice-versa)- considered only demand management policies but not supply side policies(vice-versa)- explained theories but with little or no examples to illustrate- largely 1–sided without consideration <strong>of</strong> demerits/limitations1-56-9L3A good analysis <strong>of</strong> the effectiveness and suitability <strong>of</strong> these policies (at least2 demand-side and supply-side policies) with good use <strong>of</strong> examples toillustrate.10-13E1 Mainly unexplained judgement 1-2E2 Judgement based on analysis <strong>of</strong> the nature and state <strong>of</strong> the economy 3-4


(10) 2011 RI Prelim H2 Paper Q4The Economic Strategies Committee was set up by the government in May 2009 tochart Singapore’s future growth directions. The Committee’s key recommendation is toraise Singapore’s annual productivity growth from an average <strong>of</strong> 1 percent in the pastdecade to 2 to 3 percent over the next decade. This is a major shift from thegovernment’s past strategy <strong>of</strong> heavy reliance on labour inputs to generate economicgrowth.(a) Explain the key macroeconomic objectives <strong>of</strong> a government. [8] (b) Discuss whether the shift towards productivity-­‐driven growth will help Singapore achieve its macroeconomic objectives? [17] (a)Introduction:§Identify the 4 main macroeconomic goals <strong>of</strong> a government – sustained economic growth, low unemployment, low inflation and BOP equilibrium Body: Explain the meaning <strong>of</strong> each macro goal and the key reason/s for each goal, i.e. the benefits arising from achieving the macro goals. §Sustained economic growth o Sustain economic growth is a sustained increase in actual output as measured by higher Real GDP and increase in potential output/ productive capacity. o Actual growth is desirable as it leads to higher standard <strong>of</strong> living if population growth is slower as more goods and services are available for consumption. It also results in easier redistribution <strong>of</strong> income which can dampen the effects <strong>of</strong> income inequality in a country. o Potential growth is desirable as it promotes price stability and leads to non-­inflationary actual growth in the LR if AD increases. It also leads to sustained rise in SOL. Sustained economic growth also lowers cyclical unemployment. §Low unemployment rate o Unemployment Rate = % <strong>of</strong> unemployed workers in the labour force. The unemployed are people <strong>of</strong> legal working age who are actively looking but unable to find work. o Low unemployment or full-­‐employment is desirable as the economy is producing the maximum output possible with all its resources, thereby maximizing the country’s standard <strong>of</strong> living. Low unemployment will also lead to less social costs and hysteresis (loss <strong>of</strong> skills) due to long periods <strong>of</strong> unemployment; which may dampen a country’s economic growth. It will also result in less burden on the government budget due to lower welfare benefits and higher tax revenue. Apart from achieving


full employment, a country also strives to lower its natural rate <strong>of</strong> unemployment so as to increase its potential output which will lead to rising SOL. §Low and stable inflation rate o Inflation rate = annual % change in the GPL, measured by the CPI or GDP deflator. o Low inflation or an acceptable rate <strong>of</strong> general price rise promotes efficiency and certainty. Price signals transmit revenue/cost information accurately and resources are efficiently allocated by the price mechanism. Rates <strong>of</strong> return on potential projects can be accurately estimated, thereby encouraging investment. Low inflation and the consequent low COP enhances a country’s export competitiveness, boost exports & FDI, thereby strengthening its BOP. The increased I, X & FDI stimulate AD, resulting in both actual & potential growth, higher employment & rising SOL. §Healthy Balance <strong>of</strong> Payments (BOP) o BOP is an annual account <strong>of</strong> a country’s transactions in trade & investment with the rest <strong>of</strong> the world. Over the LR, a country strives to achieve BOP equilibrium. o A persistent BOP deficit will lead to drain <strong>of</strong> foreign reserves and debt which dampens a country’s economic growth and/or a depreciation <strong>of</strong> the ER which may lead to loss <strong>of</strong> confidence in the economy and imported inflation. o A persistent BOP surplus may result in the dutch disease caused by appreciation <strong>of</strong> the ER, a lower SOL then was possible, and possible retaliation from trading partners. Mark Scheme: Knowledge, Application, Understanding, AnalysisL1 -­‐ Merely state the goals without explaining the goals nor thereasons for the goals-­‐ Weak in conceptsL2 -­‐ Clearly explain the meaning and the reasons for at least 2 <strong>of</strong> the4 macro goals Or explain the meaning and reasons for all 4macro goals but lacks depth.L3 -­‐ Explain the meaning and reasons for all 4 macro goals, showingstrong theoretical framework-­‐ Or explain meaning & reasons for 3 macro goals in great depthand somewhat superficial on the 4 th macro goal1 – 34 - 67 - 8(b)Introduction:§ “Productivity-driven growth” is achieved by improvment in the quality <strong>of</strong> resources,specifically improvements in productivity <strong>of</strong> resources.§ Define productivity: Productivity is measured by the output per worker per period <strong>of</strong>time or output per man-hour.


§ Policies/Measures to increase productivity: Subsidize education and training toequip workers with new knowledge/skills, tax incentives /subsidies, subsidize firms’R&D initiatives etcBody:Thesis: A productivity-driven growth will help Singapore achieve its macro goals <strong>of</strong> LRnon-inflationary economic growth, as well as improves its BOP.§ Students are expected to explain how the measures to improve productivity willachieve macroeconomic objectives in Singapore.§ Impact on potential growthèIf the above measures adopted by the government are successful, the higherproductivity will increase the economy’s productive capacity in the LR, as more outputcan be produced with the given resources. In addition, COP/ATC also falls becauseevery unit <strong>of</strong> output now requires less time and hence less labour cost to produce, i.e.higher productivity leads to lower AVC and hence lower ATC. This LR effect on COP &potential growth leads to both a downward and rightward shift <strong>of</strong> the AS.GPLAS 0 AS 1P 1P 3P 0AD 0AD 1Y 0Y 1 Y 2Real NY§ Impact on actual growth, employment, GPL & BOPè Interventionist measures discussed above will lead to increased AD due tohigher I and G. As Real NY rises, firms’ demand for labour increases andcyclical unemployment falls. This will bring the economy closer to its potentialoutput depending on the increase in I and G. This SR actual growth leads tohigher GPL as productivity remains unchanged.è In the LR, when productivity is successfully raised, the lower COP leads tolower GPL.è With falling COP & GPL, the quantity <strong>of</strong> output demanded by the 4 sectors <strong>of</strong>the economy increases, including export demand & FDI by the foreign sector.This impact is, represented by a movement along the new AD curve, raisingReal NY further from Y 1 to Y 2 .


è The boost to exports & FDI improve both the current and financial accounts <strong>of</strong>Singapore’s BOP.Anti-thesis: A productivity-driven growth cannot guarantee the achievement <strong>of</strong> FE inSingapore. In addition, raising productivity growth is not without problems anduncertainties.§ Impact on full-employmentè Achieving full employment level <strong>of</strong> national income requires that AD issufficiently high to buy the potential output. However, the level <strong>of</strong> AD dependson many factors, including business and consumer confidence. A policyfocusing on raising productivity cannot effectively counter such fluctuations inAD because such a policy is a LT supply-side policy aimed primarily atshifting the AS.è However, during a recession, the Singapore government does step up itsspending on productivity to “kill two birds with one stone”, i.e. stimulate somedemand and alleviate cyclical unemployment while boosting future potentialgrowth. But such a policy alone is insufficient due to its limited impact on AD.Hence the government needs to adopt additional stimulus to boost AD.§ Issues on unemployment rateè If growth <strong>of</strong> AD does not keep pace with potential growth, unemployment ratewill rise.§ Issues in increasing productivityè While increasing productivity growth is a desirable and necessary growthdirection for Singapore in view <strong>of</strong> intensified global competition, there aremany problems in raising productivity. It not only takes time, it is also a costlyinvestment and a heavy drain on govt finances. The outcome is uncertaindepending not only on the workers receptiveness but also the quality <strong>of</strong>training.Conclusion & Overall Evaluation§ Make a judgement on SR & LR impact <strong>of</strong> policy on macro goals—LR impact largelyfavourable, but the policy cannot resolve Singapore’s underlying vulnerability toexternal shocks and FE in Singapore still depends on external conditions.§Make a judgement on the difference between the policy <strong>of</strong> focusing on productivityincrease versus the past strategy <strong>of</strong> heavy reliance on foreign labour.Mark Scheme: Knowledge, Application, Understanding, AnalysisL1 -­‐ General remarks on the impact <strong>of</strong> the policy with or withoutreference to the macro goals-­‐ No evidence <strong>of</strong> theoretical framework-­‐ Good analysis on the thesis, but no evidence <strong>of</strong> any attempt toprovide an anti-thesis.L2 -­‐ Clearly discuss the impact <strong>of</strong> the policy on at least 2 <strong>of</strong> the 4macro goals, using a clear theoretical framework1 – 45 - 9


-­‐ Or discuss the impact on all 4 macro goals but weak intheoretical framework-­‐ A good answer with theoretical framework but no reference atall to measures adopted by Singapore government to raiseproductivity-­‐ largely one-sided answer, either weak thesis or weak antithesis.L3 -­‐ Discuss impact on all 4 macro goals with a clear theoreticalframework, showing a balanced analysis with scope & depth10 -13EvaluationE1 An unexplained judgement. 1 – 2E2Evaluative assessment supported by economic analysis. 3 - 4


(11) 2011 HCI Prelim H1 Paper Q4Inflationary pressures in Singapore are rising due to higher property and COE prices.Singapore reduced its growth forecast for 2011 as a faltering US economy and theEuropean debt crisis heightened the risks to global expansion.(a) Explain how inflationary pressures are a threat to sustained economic growth andimprovement in living standards. [10](b) Assess the effectiveness <strong>of</strong> the policy options available to deal with both economicthreats facing the Singapore economy. [15]<strong>Suggested</strong> answer for 4 (a)IntroductionInflation is the persistent rise in the GPL as measured by the % increase in CPI over time. Unless,inflation is brought under control (i.e. low and stable) it could pose a threat to a country’s economicgrowth and living standards.BodyThesis (1)- Impact on AD and growthFigure 1:Price Level AS AD1 AD Real GDP Y Yf With reference to figure 1, economic growth can be adversely affected if inflationary pressures result inan inward shift <strong>of</strong> the AD. Assuming the economy is currently operating at the Yf level <strong>of</strong> output. Ifinflation spins out <strong>of</strong> control ( i.e. prices soar to a high and unstable levels), AD might be fall due to:


(1) Loss in export competitiveness e.g. soaring wage rates and other business costs like rentals;utility and transportation costs pushed up the cost <strong>of</strong> production for exporters. Foreigners mightshun such expensive exports causing demand to slump and export earnings to fall. Ceterisparibus, AD would fall, if the outcome is to cause M >X or NX to become negative.(2) Loss <strong>of</strong> investors’ confidence. In an economy that is plagued by soaring inflation e.g.hyperinflation, it is difficult for businesses to predict the future returns from their investments. Forexample, when prices and costs are spiraling out <strong>of</strong> control, it is difficult for businesses to assessthe risks and benefits <strong>of</strong> investing in new production capacity e.g. buy new machines; tools andequipments or build new factories. Due to uncertainties and risks businesses are likely to holdback investments. Hence the I component <strong>of</strong> AD falls.In short, high and unstable inflation is likely to hit X and I causing AD to fall and economic growth toslow-down or even contract from Yf to Y. (Refer to figure 1).Thesis (2) - Impact on ASFigure 2:Price level LRAS SRAS1 SRAS AD Y Yf Real Y With reference to figure 2:Cost-push inflationary pressures can also derail growth by causing the AS curve to shift leftwards.How?Higher rental and transport costs => Increases cost <strong>of</strong> production to businesses/producers. Businessespassed on the higher costs to consumers by raising the prices <strong>of</strong> their output/product. AS curve shiftsupwards…. Real GDP slows or even falls from Yf to Y as shown in figure 2.


Thesis (3) -Threat to improvement in SOLSOL or living standard is measured in terms <strong>of</strong> real GDP per capita.If inflationary pressures result in a fall in real Y or output (refer to figures 1 and 2) it might cause SOL orliving standards to deteriorate. Fall in real income <strong>of</strong> households => fall in purchasing power =>deterioration in material SOL.ConclusionAs explained above, inflationary pressures if left unchecked could derail growth by dampening spending(AD) or pushing up business costs ( AS) thus leading to a slowdown/contraction in growth and apossible deterioration in material SOLLevel Marks DescriptorsL3 9-10 Excellent analysis, coherent structure and good exemplification7- 8 Answer is contextual and has both adequate scope and depth <strong>of</strong> analysis and nomajor conceptual flaws/errors.L2 6 Answer is contextual and has depth but is lop-sided e.g. did not explore the use <strong>of</strong>the 3 concepts adequately.5 Answer is contextual and has adequate scope or breadth but lacking in depth <strong>of</strong>analysis.L1 4 Answer is contextual but is too superficial; sketchy and lacking in coherence. Hasglaring conceptual errors/flaws.3 Too descriptive or generic ( i.e. non-contextualized)0- 2 Mostly irrelevant answer<strong>Suggested</strong> answer for 4 (b)IntroductionIn the above scenario, Singapore faced the twin threat <strong>of</strong> rising inflationary pressures as well as apossible recession. To deal with these two major economic threats, the government could resort to usinga range <strong>of</strong> policy weapons such as fiscal and monetary policy, Supply side policies and trade policy. Ishall assess the effectiveness <strong>of</strong> these policy options.Body


The essay discussion can be divided into 2 major parts:(A) Generally ineffective policies e.g. fiscal and monetary policies(B) Generally effective policies e.g. SSP and Trade policies(C)Part A : Fiscal and Monetary Policies(Basically Ineffective Policies in the SG contexts)Thesis (1) – Use <strong>of</strong> Fiscal Policy – Keynesian style e.g. pump-priming or fiscal tighteningRecession Threat:To deal with possible recession arising from weakness <strong>of</strong> the US and EU economies, the SG govt couldresort to ‘pump-prime’ the economy to head <strong>of</strong>f a recession e.g. building infrastructure like roads; givingout shopping vouchers or unemployment benefits etc. The aim is to generate multiplier effects to boostspending in order to cushion <strong>of</strong>f adverse impact on AD due to the slump in exports to these major tradepartners.Anti-ThesisIn the context <strong>of</strong> a small and open economy like Singapore, this measure has very limited usefulnessbecause <strong>of</strong> huge leakages arising from high propensity to import and save. The huge leakages haveresulted in a very weak K effect (i.e. K value/size is less than 1). In fact, given a k value <strong>of</strong> 0.5, anyincrease in G spending by say $1 will generate an expansion <strong>of</strong> the national income by only 0.50 cents.Inflation ThreatTo dampen inflationary pressures, the government could resort to fiscal tightening measures e.g. cut Gspending and raise Taxes.Anti-ThesisHowever, using fiscal tightening measures to dampen inflationary pressures is likely to be ineffectivebecause the root cause <strong>of</strong> inflation, as described in the above scenario is not excessive spending. It isdue to rising property prices and fuel costs. These are largely cost-push inflationary pressures and notdemand-pull. Thus, tightening fiscal spending would only worsen recessionary pressures without havingmuch impact on dampening inflationary pressures.Thesis 2 - Use monetary Policy


2 different approach to conduct monetary policy:Interest Rate versus Exchange Rate Centered monetary policy2.1 Conventional method <strong>of</strong> controlling interest rates is not used.The MAS or central bank does not control interest rate in order regulate liquidity or volume <strong>of</strong> credit ormoney supply. As SG is trade-dependent economy with a small and open capital market, the governmenthave chosen to manage the Sing$ exchange rate instead <strong>of</strong> controlling interest rates.FYI onlyTo control rising property prices the MAS used regulations e.g. restrict loans for buying property to controlinflationary pressures due to rising property prices.Evaluation:Cooling measures e.g. restricting home loans targeted at keeping property prices from soaringMay not be very effective if there is a influx <strong>of</strong> capital from foreigners who are cash rich and looking forbetter returns from their investments. Foreign buyers <strong>of</strong> property contribute to high property prices.[ Students wont be penalized if they don’t refer to cooling measures]2.2 Exchange rate centered monetary policyThis is <strong>of</strong>ten used as SG key anti-inflationary tool to counter imported cost-push inflationary pressures [refer to MAS policy <strong>of</strong> “gradual and modest appreciation” or “Grama Policy” ]Effectiveness(1) Might not be useful in fighting inflation…Strengthening the currency might not be effective in dealing with soaring property prices/asset bubbles orrising COE prices originating from domestic ( as opposed to imported) sources.According to some economists, the GRAMA policy has unintentionally caused soaring property prices andasset bubbles. Foreigners are taking advantage <strong>of</strong> the GRAMA policy to park their money/funds in SGand to invest in property


2) Might not be useful in fighting recession…oooPart B :Weakening the Sing$ to deal with the threat <strong>of</strong> recession, might be counter-productive. This isbecause SG’s exports have a high import content e.g. SG imports crude oil to be processed intorefined oil for exports. So, a weaker Sing$ implies that cost <strong>of</strong> refining oil would be pushed upmaking our oil exports less competitive.Moreover, in the face <strong>of</strong> threat <strong>of</strong> global recession, weakening the currency might not be adequateto <strong>of</strong>fset the slump in export demand due to the lack <strong>of</strong> purchasing power/falling incomes <strong>of</strong> USand EU consumers <strong>of</strong> SG exports.Supply Side Policies (SSPs) and Trade PoliciesSSPs – 2 types short and long run.To deal with inflation ThreatSR - cost cutting measures….Government can subsidized business costs e.g. cut property tax or subsidized rentals/housing costs andother business costs e.g. fuel costs.Evaluation – This policy seems to deal with both the threat <strong>of</strong> inflation and recession.By reducing costs ( transport and housing)LR - expand productive capacity:(1) Boost productivity e.g. skills upgrading(2) R + D and Innovation and improvement in technology(3) Influx <strong>of</strong> FDI and foreign/ migrant workers to supplement the indigenous aging workforce.Generally useful in sharpening resiliency and the economy’s competitiveness in dealing with majoreconomic problems like threat <strong>of</strong> inflation or recession.Benefit ExportersBoth cost cutting and expansion in productive capacity via productivity gains and product innovation andimprovement should also benefit our exporters facing the threat <strong>of</strong> a global recession. It should help tosharpen our export competitiveness – both price and non-price, if these supply side measures result in theability <strong>of</strong> our exporters to <strong>of</strong>fer better and cheaper products to ourTrade partners.However(1) Effect can only be felt in the long run e.g. productivity gains.(2) R +D, innovation etc – no guaranteed success.


Trade PoliciesIn the light <strong>of</strong> an impending threat <strong>of</strong> global recession, SG should continue to pursue the policy <strong>of</strong> tradeliberalization e.g. forging a network <strong>of</strong> FTAs with both traditional (e.g. EU) and non-traditional tradepartners(Russia, Middle East). By diversifying export markets and FDIs flows via FTAs such as establishingmore trade and investment ties with emerging economies like China and India, which are neither“faltering “ economies nor are they affected by “Sovereign Debt Problem” , SG can cushioned itselfagainst external shocks. Unlike countries like US etc, SG should not resort to protectionist measures toshelter domestic firms or workers from foreign competition.ConclusionSince SG faces the twin problems <strong>of</strong> an inflation-cum-recession threat ( i.e. stagflation), the mostappropriate measures are SSP policies aim at sharpening the economy’s competitiveness by reducingcosts and improving the quality and appeal <strong>of</strong> its exports in the world market. In addition, the governmentmay also have to build more resiliency e.g. reserves which are useful to tide the economy over a difficultperiod <strong>of</strong> external turbulence.Domestically, the government may need to specifically target policies at controlling property prices fromsoaring or collapsing. Moreover, moderating COE prices might conflict with other objectives such ascontrolling road congestion/pollution arising from car ownership.Knowledge, Application, Understanding and AnalysisFor an excellent answer that has good analysis, coherent structure and application to thecontext.10-11For an answer that has adequate breadth and depth and no major conceptual flaws.9For an answer that has depth <strong>of</strong> analysis but is lop-sided e.g. no Anti-thesis i.e. too narrow inscope7–86For an answer that has adequate breadth but is lacking in depth <strong>of</strong> analysis i.e. too superficialBroadly on track, but analysis is lacking in terms <strong>of</strong> both breadth and depth i.e. too superficial,


sketchy 5Mostly descriptive answers ( smattering <strong>of</strong> econ analysis)Mostly irrelevant answers i.e. <strong>of</strong>f-tracked3-40-2EvaluationFor an evaluative discussion based on economic analysis 3 – 4For an unexplained judgment, or one that is not supported by economic analysis 1 – 2(12) 2011 DHS Prelim H1 Paper Q4In the recent Global Financial Crisis, Singapore faced the sharpest recession and highestunemployment among major Asia-Pacific economies, given its high dependence on externaldemand.(a) Explain the possible causes <strong>of</strong> high unemployment in Singapore.[10](b)Discuss whether demand-management policies can best address the issue <strong>of</strong> highunemployment in Singapore.[15]Part (a): A high rate <strong>of</strong> unemployment can be caused by:• Cyclical (demand-deficit) unemploymento Falling national incomes due to GFC à significant fall in purchasingpower à sharp fall in Singapore’s export demand, assuming highYED for Singapore’s exportso Large share <strong>of</strong> X in AD à fall in X leads to decrease in AD à firmsface large unplanned investments, thus reducing output and demandfor factors <strong>of</strong> production (including labour). Fall in NY andemployment reinforced by multiplier process, leading to increase incyclical unemployment.• Structural unemploymento Structural change in economy (eg, loss <strong>of</strong> export demand coupled with loss <strong>of</strong> comparative advantage causing sharp decline in manufacturing sector, strong growth in services sector due to opening <strong>of</strong> IRs) à laid-­‐<strong>of</strong>f workers unable to find alternative employment (eg, due to factor immobility) à high structural unemployment


• Real-­‐wage unemployment o Wages may be held above market clearing level and unable to adjust quickly (eg, contractual agreement) à use DD/SS analysis to show surplus labour à high real-­‐wage unemployment Knowledge, Application, Understanding and AnalysisL38-10L25-7For an answer which• has sufficient scope and depth <strong>of</strong> economic analysis • applies accurately to the given context, explaining causes <strong>of</strong> high unemployment • provides evidence <strong>of</strong> exemplification • contains minor conceptual errors For an answer which• provides at least two possible causes <strong>of</strong> high unemployment • contains some economic analysis though not consistently applied • is largely theoretical with limited application to the given context • contains some conceptual errors ORCap at 5m• applies sound economic analysis but lacks scope, i.e. explained only one possible cause <strong>of</strong> high unemployment ORCap at 7mL11-4• applies sound economic analysis with scope but without consideration <strong>of</strong> high unemployment For an answer which• is largely irrelevant with no indication that the meaning <strong>of</strong> the question has been properly grasped • is largely descriptive or lacking in the use <strong>of</strong> economic analysis • contains major conceptual errors Part (b)• ‘Best address’ – discuss (i) speed and (ii) extent <strong>of</strong> resolving high unemployment• Explain how demand management policies can solve demand-deficientunemployment (include limitations and inability to solve structural unemployment)• The need for supply-side policies to resolve structural unemployment (includelimitations <strong>of</strong> supply-side measures)• Provide judgment on the use <strong>of</strong> demand management policies to solveunemployment in general.• Use AD/AS diagrams to reflect the impacts <strong>of</strong> the relevant policies in addressing unemployment


oooJobs Credit Scheme as a form <strong>of</strong> expansionary fiscal policy -­‐ directly subsidise the cost <strong>of</strong> production for firms and retain employment. § Limitations - firms may still have to lay <strong>of</strong>f workers (i) if they cannotstay afloat despite the reduction in the costs <strong>of</strong> production (ii) ifthey face persistent declining demandSPUR (Skills Programme for Upgrading and Resilience) – trainingopportunities for workers who are unemployed in view <strong>of</strong> skills mismatchwith the employment opportunities available. The training serves toreduce domestic structural unemployment in the long run.§ Limitations – Need to convince employers and workers to invest time and effort for a long term benefit when they are facing short term constraints. Increase in government expenditure as a form <strong>of</strong> expansionary fiscal policies (eg, bring forward infrastructural projects, increase provisions in education and healthcare, expand employment in public sector). There are supply side effects from the expansionary fiscal policies since the expenses are on education and healthcare. This has long term effect <strong>of</strong> improving the AS curve to the right. § Limitations – small multiplier – high leakages may dilute theeffectiveness <strong>of</strong> an increase in G (eg, high import spending inSingapore; increased savings due to poor economic outlook)Evaluation:• Nature and magnitude <strong>of</strong> unemployment – combination <strong>of</strong> policies required tobetter address high unemployment• Characteristics <strong>of</strong> Singapore economy – small and open – limited effectiveness <strong>of</strong>government policies in dealing with external economic downturn• Time lags: increase in government spending might take some time to workthrough the economy. If the time lag is too long, the fiscal policy may prove to bedestabilizing, because the economy could have recovered by then. The fiscalpolicy could thus lead to the problem <strong>of</strong> overheating the economy.


Knowledge, Application, Understanding and AnalysisL39-11L26-8• For an answer which• discusses demand management and alternative policies • displays depth <strong>of</strong> economic analysis for most part <strong>of</strong> the essay • makes effective use <strong>of</strong> diagrams • applies accurately to the given context • may contain minor conceptual errors For an answer which• provides sufficient scope, i.e. demand management policies and alternatives • contains some economic analysis though not consistently applied • is largely theoretical with limited application to the given context • contains some conceptual errors ORCap at 8m• provides synthesis <strong>of</strong> various policy options without addressing the issue <strong>of</strong> high unemployment ORCap at 6mL11-5E2• applies economic analysis but lacks balance For an answer which• is largely irrelevant with no indication that the meaning <strong>of</strong> the question has been properly grasped • is largely descriptive or lacking in the use <strong>of</strong> economic analysis • contains major conceptual errors For a reasoned judgement3 – 4E1For an unexplained judgement1 – 2


(13) 2011 JJC Prelim H1 Paper Q4Firms are more cautious and have held back on their hiring as the global economic recoveryremains weak and uncertain. With an increasingly large U.S. federal debt, the governmenthas its hands tied for any further spending to stimulate the economy.a) Explain why the U.S. government prioritizes to solve a balance <strong>of</strong> payment deficit overhigh unemployment from the global recession.[10]b) Discuss the view that fiscal policy is the best way for the U.S. government to achieve ahealthy balance <strong>of</strong> payment and low unemployment.[15]Part AIntroductionThe global recession in 2009 has plunged the world and the U.S. into a deep recession. Thishas resulted in a massive rise in unemployment figures in the U.S as consumption both inU.S. and the rest <strong>of</strong> the world fell sharply. The recession resulted in a dramatic fall ininvestor’s confidence both abroad and domestically. Producers in U.S. faced a fallingdemand for their goods and services hence cut back on production and hiring. As a result,consumption and income will fall in the U.S leading to a fall in AD.Previously, the challenges <strong>of</strong> globalization has already resulted in massive loss <strong>of</strong> jobs in theU.S. as emerging economies like China has overtaken the U.S. in low cost manufacturing,resulting in a persistent and growing balance <strong>of</strong> trade (BOT) deficit. The global recessionmade things worse by aggravating the unemployment problem by increasing the cyclicalunemployment due to a fall in AD. Also, the need for capital inflows to finance the BOTdeficit might not be sufficient.Hence, in the midst <strong>of</strong> a global recession, it is all the more a priority for the US governmentto achieve a healthy balance <strong>of</strong> payments, so as to avoid or minimize the costs/implications<strong>of</strong> a BOP deficit. Therefore the need to weigh the implications between that <strong>of</strong> a balance <strong>of</strong>trade deficit, capital account deficit and high unemployment will have to be done.Internal effects <strong>of</strong> a BOT deficitWith an increasing trade deficit, whether it is due to falling export revenue or rising importexpenditure, it is detrimental to domestic production. A fall in net exports will reduce


aggregate demand in U.S. and decrease production through the multiplier effect. Nationalincome will thus fall by a multiple amount and cyclical unemployment will rise considerably.A country that loses its comparative advantage will also experience a rise in structuralunemployment which could take a longer time to remove. For example, with an increase indemand for low cost imports from China, US firms producing labour intensive manufacturinggoods such as mobile phones and textiles, suffer from the increased competition whichresults in higher unemployment. In addition, the effects <strong>of</strong> outsourcing operations from U.S.to low cost economies like China and Vietnam has worsened the unemployment problem(structural).External effects <strong>of</strong> a BOT deficitThe U.S. government needs to maintain a healthy BOT. A persistent or prolonged BOTdeficit means that the country has been depleting its foreign reserves and/or resorting toexternal borrowing from the organisations such as the International Monetary Fund orthrough the issue <strong>of</strong> government bonds and selling them abroad.The US may run the risk <strong>of</strong> exhausting all its reserves or they may accumulate large amount<strong>of</strong> external debts. This has an adverse effect on its economy due to large debt repayments infuture years which is a burden for future generations. There is also a huge opportunity costas funds used to repay debts could be utilised to improve productive capacity and promoteeconomic growth instead. In addition, in the recent years, foreign countries are less willing tobuy US government bonds to finance their trade deficit. Debtor countries like China andJapan may question the ability <strong>of</strong> the U.S. government in paying back these debts. It mayalso result in higher borrowing costs for the U.S.Explain how external and internal value <strong>of</strong> money is affectedAs export earnings fall, the demand for US dollar will fall while a rise in import expenditurewill increase the supply <strong>of</strong> US dollar in the foreign exchange market. Therefore, US dollar willdepreciate. As the US dollar depreciates, the resultant imported inflation, higher cost <strong>of</strong>living; higher costs <strong>of</strong> production will all act as a deterrent to potential foreign investors to theUS. This will be detrimental to US actual and potential growth.Explain the implications <strong>of</strong> the capital accountForeign Direct Investments (FDI) into U.S. might be falling as other emerging economiescompete for these FDI. Also, the global recession could be weakening the confidence <strong>of</strong>foreign investors in U.S. Instead Asia’s growth might be more optimistic and FDI arechannelled into Asia. Hence, the capital inflow to the capital account might not be sufficientto <strong>of</strong>fset the BOT deficit, triggering the implications <strong>of</strong> a BOT deficit as discussed above.


Explain the impact <strong>of</strong> high unemployment (cyclical) as a result <strong>of</strong> the recessionThus a BOT and capital deficit alone can cause output and employment to fall. It is one <strong>of</strong>the key contributors contribute to structural unemployment. The global recession has madethings worse by raising cyclical unemployment rates significantly. The recession resulted in adrastic rise in cyclical unemployment due to the huge fall in AD from exports as Europesuffers from a drastic decline in income and output. Domestic consumption, a key driver <strong>of</strong>economic growth in the U.S. will fall significantly. These will result in a sharp rise inunemployment, especially cyclical unemployment. High unemployment is harmful to aneconomy as it will result in high fiscal cost to the government. This results in lesser funds forinfrastructural development. In addition, individuals may suffer from a loss <strong>of</strong> self-esteemand morale, which will worsen the longer they stay unemployed, leading to social problemssuch as rising crime and increase divorce rates.Conclusion: Comparing the relative priority <strong>of</strong> a BOP deficit over the unemploymentproblem due to the global recessionBoth the BOP deficit and unemployment will cause serious consequences to the U.S.economy. The U.S. government should prioritize to solve balance <strong>of</strong> trade deficit first as itshould help alleviate the problem <strong>of</strong> the high unemployment from the recession. Appropriatesupply and trade policies would need to be implemented in order to raise U.S. productivityand develop new growth areas in its industries. This will help to reduce the severe cyclicalunemployment and structural unemployment problem. There is an urgency to stimulate ADand start increasing output and production to provide jobs and income for the people and torestore confidence in the U.S. economy. By rejuvenating its export sector, foreign investorswill regain their confidence in the U.S. and increase capital inflows for investment purposesvia the capital account. Hence, by tackling both the BOT and capital account will solve thebalance <strong>of</strong> payment problem and eventually pull the U.S. out <strong>of</strong> the recession.


Marking SchemeL3Well developed explanation <strong>of</strong> the negative consequences <strong>of</strong> bothBOP deficit and high unemployment7-10L2Answer must also substantiate the relative importance <strong>of</strong> ahealthy BOP over unemployment from the global recessionWell developed explanation <strong>of</strong> the negative consequences <strong>of</strong>either the BOP deficitORWell developed explanation <strong>of</strong> the negative consequences <strong>of</strong> highunemploymentORUnder developed explanation <strong>of</strong> the negative consequences <strong>of</strong>BOP deficit and high unemployment5-6L1 A smattering <strong>of</strong> valid points 1-4b) The government can implement demand management policies such as fiscal policy andsupply side policies to tackle the balance <strong>of</strong> payment deficit issues and high unemploymentfrom the global recession. Fiscal policy is not the best way out <strong>of</strong> its economic problems butit can serve as a temporary support to its sluggish economy and improve the balance <strong>of</strong>trade. The U.S. will need supply side policies as a long-term solution to address its BOTdeficit and restore confidence in its economy.Explain how the policy works to address the problem <strong>of</strong> unemployment and recessionFiscal Policy: To support the economy due to the fall in AD from the recessionFiscal policy refers to the deliberate attempt by the government to adjust macroeconomicvariables by adjusting tax rates and government expenditure. The severity <strong>of</strong> the globalrecession meant the U.S. government had to act fast and increase its expenditure to supportthe economy against the recession that followed. The government can reduce both personalincome and corporate tax rates so as to encourage consumption and investment. Areduction in personal income tax would increase disposable income thus stimulatingpurchasing power and hence spending which benefits firms. A cut in corporate tax willincrease after-tax pr<strong>of</strong>its, encourage firms to invest, expand and recruit more workers. TheU.S. government can directly inject public spending into the circular flow <strong>of</strong> income to


increase the aggregate demand. Via the multiplier effect, national income will increase by amultiple and as production increases, this gives rise to higher employment opportunities.Limitations <strong>of</strong> Fiscal PolicyHowever, though pump priming the U.S. economy may have a significant impact on nationalincome, production and hence unemployment. This is because U.S. citizens have in generala high marginal propensity to consume. Any additional increase in income is likely tosignificantly induce additional consumption. Its fiscal multiplier effect tends to be large.However, the persistent and large federal budget deficit and debt that has accumulated willseverely limit the ability <strong>of</strong> government spending to stimulate the economy.Further increasing government spending may have significant consequences on thecredibility <strong>of</strong> the U.S. government’s fiscal health and raise the cost <strong>of</strong> borrowing. Confidence<strong>of</strong> the USD from important trading partners such as China will be eroded and they will not bewilling to lend to the U.S. in future.Monetary Policy: To support the economy during the recessionThe Federal Reserve has lowered interest rates to combat the sluggish economy activity inU.S. A lowering <strong>of</strong> interest rates will lower cost <strong>of</strong> borrowing and allow firms to borrow at acheaper rate to finance their operations and expansions. Also, it will allow marginalinvestment projects to be pr<strong>of</strong>itable and hence increase investments. This will help toincrease AD and output.In addition, though contractionary monetary policy can also attract short term capital inflows,helping the BOP to improve, it is not an appropriate time to do so as the need to stimulatethe economy takes priority.Limitations <strong>of</strong> Monetary PolicyLowering <strong>of</strong> interest rates may not achieve the stimulating effect on AD due to other factorsaffecting investment activities. These factors could be a drastic fall in confidence amongstfirms to engage in any form <strong>of</strong> investments due to the pessimistic economic outlook during theglobal recession. Also expectation for future pr<strong>of</strong>its will be low as a falling AD reduces incomeand consumption. Consumers are unlikely to spend; rather they will tighten their belts.The MEI for investments is inelastic as investments are unresponsive to interest rate changes.Hence, the lowering <strong>of</strong> interest rates will only lead to a less than proportionate increase ininvestment, limiting the effectiveness <strong>of</strong> monetary policy to stimulate AD and national income.This is especially evident during a global downturn as confidence and business expectationsare likely to be more important than interest rates.


Conclusion on demand management policiesDemand management policies such as fiscal policy do not work well to curb the BOP deficitfacing the U.S. especially in the midst <strong>of</strong> a severe recession. However, the fiscal andmonetary policy can cushion the impact <strong>of</strong> the fall in AD from the global recession. However,appropriate supply side polices should be implemented to promote long term economicgrowth, this is especially important when the U.S. eventually emerges from the recession toensure its competitiveness is not compromised. Also, these policies seeks to solve cyclicalunemployment, there is still structural unemployment from the rising global competition thatneeds to be tackled.(Exchange rate policy can be used as well)Supply side Policy: To correct the problems <strong>of</strong> the BOT deficitDevelop new niche/comparative advantage for export growthThrough such schemes, the workforce will be equipped with new skills and knowledgerequired by the high-tech and knowledge-intensive industries. Training schemes should beput in place to not only retrain the already structurally unemployed but also those who maybe unemployed in future as the U.S. economy needs to maintain itself as a knowledgebasedeconomy.Increased in education and training <strong>of</strong> workers has an impact <strong>of</strong> improving the quality <strong>of</strong> ourworkforce which will lead to greater productivity and higher efficiency. With higher productivityand efficiency, the cost <strong>of</strong> production can be lowered making exports more competitive.Investing the necessary resources into developing new niche areas (comparativeadvantage) is important for the U.S. to correct the BOT deficit. This will allow the U.S. togenerate strong export growth and stimulate jobs and output for the economy. The globalrecession has aggravated the unemployment problem in the U.S. It was already facing stiffcompetition in certain industries from emerging China and India resulting in severe structuralunemployment. Hence there is an urgency to reemploy these displaced workers into otherindustries.U.S. has prided itself as the leading economy that recognizes and utilizes foreign humancapital to drive its economy and to encourage investments and innovations. It must continueto open its doors to globalization to encourage trade and human capital movement. This willhelp to maximize the potential from its own human resources and those from abroad in orderto stay ahead <strong>of</strong> its competitors.


Limitations <strong>of</strong> Supply side policiesSupply side policies require huge fiscal commitments from the U.S. government. At a timewhen the U.S. government is facing mounting debts and a budget deficit, its ability to spendfurther will be limited. In addition, supply side policies benefits can only be felt in the longrun, it will not alleviate the stress <strong>of</strong> cushioning the impact <strong>of</strong> the loss <strong>of</strong> income and jobs.Measures to raise productivity and to make U.S. workers more employable might faceresistance from the trade unions and the workers themselves. Workers might not bereceptive to retraining. Trade unions might lose these members to other industries as thesedisplaced workers seek employment in other industries.ConclusionFiscal policy is not the best measure to solve the BOP deficit but it can help in the short termto alleviate the impact <strong>of</strong> the fall in AD due to the recession as fall as the U.S. fiscal balancecan support. Perhaps a more permanent solution to solving its BOP problem is to identify theroot cause <strong>of</strong> the problem such as falling competitiveness and inefficiencies in its labourmarket and implement appropriate supply side policies to tackle them. This will boost theexport sector and spearhead the recovery <strong>of</strong> the U.S. economy.Marking schemeL3Well developed explanation (how the policy works andlimitations) on how fiscal policy and 2 other policies (supply sidepolicy is a must) helped to address the economic problems <strong>of</strong>recession and the rising unemployment rate.9-11L2 Developed explanation on EITHER how fiscal policy OR the 2other policies help to address the economic problem <strong>of</strong>recession and the implementation <strong>of</strong> other supply side policiesthat rectify the root cause <strong>of</strong> the quality and competitiveness <strong>of</strong>U.S. exports.6-8ORL1Undeveloped explanation on how both fiscal policies and otherpolicies help to address the economic problems <strong>of</strong> recession &rising unemploymentA smattering <strong>of</strong> valid points on how fiscal policy and otherpolicies work to solve the problem <strong>of</strong> recession andunemployment1-5


E2 For an evaluative assessment based on economic analysis 3-4E1For an unexplained assessment or one that is not supported byeconomic analysis, e.g. a combination <strong>of</strong> policies is to be used,nature <strong>of</strong> the economy, etc.1-2(14) 2011 RVHS Prelim H2 Paper Q5In 2009, many countries introduced stimulus packages to deal with the impact <strong>of</strong> the globalfinancial crisis.a) Explain the factors that can undermine the impact <strong>of</strong> such stimulus package. [12]b) Assess the effectiveness <strong>of</strong> the policies used in Singapore to mitigate the most pressingissue <strong>of</strong> the global financial crisis. [13]Part (a)Size <strong>of</strong> multiplier• The size <strong>of</strong> the multiplier is calculated by k = 1/ (1-mpc) or 1/mpw, where mpw =mps+mpt+mpm.• The larger the mpc, the larger will be the size <strong>of</strong> the multiplier and hence the larger theeventual increase in national income and vice versa• Thus, increase in government spending and hence the stimulus package will have lesssignificant impact on increasing national income.Crowding out effects• Government may finance its deficit spending by borrowing from the credit market• Higher interest rates, in turn, tend to reduce or ‘crowd out’ consumption and investmentwhich are sensitive to interest rates.• Hence, the effectiveness <strong>of</strong> expansionary fiscal policy in stimulating aggregate demandwill be mitigated.Responses from consumers and investors• May not get the desired response from households and firms due to other reasons suchas poor economic outlook ahead.Size <strong>of</strong> fiscal stimulus


• If the size <strong>of</strong> the fiscal stimulus is <strong>of</strong> an insignificant proportion to GDP <strong>of</strong> the country, theimpact on national income and employment level will also be insignificant as not enoughare given to reach out to the masses.Knowledge, Application, Understanding and AnalysisL3For a developed explanation on factors which mayundermine the impact <strong>of</strong> the fiscal stimulus.9 - 12L2For an undeveloped explanation on the factors whichmay undermine the impact <strong>of</strong> the fiscal stimulus.5 - 8L1 Smattering <strong>of</strong> valid points 1 - 4Part (b): Most pressing issue: unemploymentFiscal policy• # government expenditure/ $ taxes I #AD I #NY I # employment• Examples <strong>of</strong> # government expenditure/ $ taxes: bringing forward <strong>of</strong> infrastructureprojects, reducing corporate tax/ hiring more public servants/• However, the size <strong>of</strong> the multiplier in Singapore tends to be small as compared to othercountries due to high withdrawals I not effectiveExchange rate policy• Depreciation <strong>of</strong> the SGD I # (X-M) I #NY I # employment• However, other countries had also depreciated their currency which in turn meant thatexports from Singapore were not that price competitive after all.Supply-side policyShort-run• Government introduced short run supply side policies to help firms cope with costs <strong>of</strong>production• Examples: jobs credit scheme/ property tax rebates


Long run• Provided higher course fee support for companies and individuals and absentee payrollsfor companies that send their workers for training so as to help Singaporeans to upgradetheir skills so that they can stay employed or seek re-employment.• Training grants were also provided for general education and skills for workers to acquirepost-graduate diploma and even university degrees so as to have a stronger impact onre-employment and retaining productive workers in the labour market.• Example: Skills Programme for Upgrading and Resilience (SPUR)• However, re-training or skills upgrading is costly, which gives rise to funding problems.Knowledge, Application, Understanding and AnalysisL3For a developed discussion on the effectiveness <strong>of</strong> a variety <strong>of</strong>policies to tackle the most pressing issue brought about by theglobal financial crisis7 - 9L2For an undeveloped discussion on the effectiveness <strong>of</strong> a variety<strong>of</strong> policies to tackle the most pressing issue brought about by theglobal financial crisisOrFor a developed explanation on the effectiveness <strong>of</strong> a policy totackle the most pressing issue brought about by the globalfinancial crisis4 - 6L1 Smattering <strong>of</strong> valid points 1 - 3EvaluationE2Limitations <strong>of</strong> interventions are well explained, coupled withfurther evaluation.3 - 4E1 Limitations <strong>of</strong> interventions are superficially explained. 1 - 2


(15) 2011 MI Prelim H1 Paper Q4“Policymakers have said recently that they will use a stronger Singaporean dollar to combatinflation, but don’t want to strengthen the currency too much because that could underminethe city-state’s export competitiveness.”Taipei Time, 15 Apr 2011a) Explain how exchange rate <strong>of</strong> a country is determined.[10]• The external value a currency refers to the price <strong>of</strong> one currency in terms <strong>of</strong> anothercurrency.• The market forces – D & S – helps to determine the exchange rate : Demand factorssuchrising/falling demand for foreign goods & services, higher investment opportunitiesin foreign countries; Supply factors – favourable government policies attracts foreigninvestment into the country or preference for S’pore produced goods & services byforeigners. These factors causes the exchange rates to change eitherfavourable/unfavourable to the country.• Explain that the S$ is weighted against a basket <strong>of</strong> currencies, under a managed floatexchange rate system.• Diagram required.Level 1 Superficial definitions or vague linkages or explanation. 1-3Level 2 Clear explanation <strong>of</strong> how exchange rates are determined. 4-7Level 3 Clear explanation <strong>of</strong> how Singapore determines its exchange rate. 8-10b) Discuss whether this policy alone was adequate to resolve inflation in Singapore.[15]• Define inflation• Recognise that the increase in GPL (inflation) is due to the rise in price <strong>of</strong> essential rawmaterials – oil, food items etc, which accounted for the more expensive imported goods=> cost push inflation.• As Singapore is highly dependent on imports both for local consumption as well as forproducing exports, the S$ cannot be too weak or strong. In this case, what MAS did wasto strengthen the S$, making its currency relatively more expensive against a basket <strong>of</strong>currencies. When the S$ is strengthened, its imports, bought with S$ will now berelatively cheaper. The price <strong>of</strong> imports has fallen. Price <strong>of</strong> exports is now relatively moreexpensive, hence X may fall. Hence (X-M) may fall à fall in AD à lower growth, higherunemployment but keeping prices down and checking inflation.


• Exchange rate policy may not be adequate to resolve inflation.Other policies:1) Contractionary Fiscal Policy to reduce AD domestically2) Supply side policies to increase AS – market oriented and interventionist to reduceinflation.3) Trade policy to diversify buying <strong>of</strong> goods and services sources to reduce dependencyand seek cheaper alternative sources.Conclusion:• Exchange rate policy is not sufficient to solve inflation. This policy helps to maintain lowerimport prices to curb inflation but make exports less competitive which results in BOTdeficit or lower AD – result in conflict with other aims – Favourable BOP or growth etc.Level 1 Superficial or vague explanation provided or just mere listing <strong>of</strong> policy 1-4Level 2 Clear policy explanation and linked to the macroeconomic issue –exchange rate policy option and adequately mention other policies toresolve the inflation.5-8Level 3A balanced approach taken to explain the various policies to resolveinflation within a country. (3 policies)9 - 11E1 Superficial or vague evaluation <strong>of</strong> policies. Attempts to evaluate policies. 1-2E2 Clear evaluation <strong>of</strong> various policies, comparison <strong>of</strong> policies – if applicable. 3-4


(16) 2011 MI Prelim H2 Paper Q4The Monetary Authority <strong>of</strong> Singapore revalued the currency in April 2010 and said inOctober 2010 it would steepen and widen the currency’s trading band while seeking amodest and gradual appreciation.(a) Explain Singapore’s choice <strong>of</strong> using exchange rates rather than interest rates as aninstrument for its monetary policy. [10](b) Assess the impact <strong>of</strong> the revaluation and the changes made to the currency’s tradingband on the Singapore economy. [15]<strong>Suggested</strong> Answer Scheme:(a) Explain Singapore’s choice <strong>of</strong> using exchange rates rather than interest ratesas an instrument for its monetary policy. [10]Exchange rate measures the price <strong>of</strong> domestic currency in terms <strong>of</strong> another foreign currencywhereas interest rate is the cost <strong>of</strong> loans or borrowing.Changes in exchange rate can be used as an instrument for monetary policy by altering theprice <strong>of</strong> exports and imports and hence, causing changes in export revenue and importexpenditure to influence economic growth and unemployment.However, interest rate can be used as an instrument for monetary by changing the cost <strong>of</strong>borrowing. For instance, an increase in interest rate can increase the cost <strong>of</strong> borrowing andin turn, cause returns from investment to decrease and hence, lower investment. This in turninfluences AD to affect economic growth and employment.However, interest rate is ineffective as a choice <strong>of</strong> monetary policy.Singapore’s small and open economy results in a lack <strong>of</strong> monetary control and hence,interest rates would be volatile. Singapore’s vulnerable to capital flows is due to Singapore’sposition as an international financial centre. This can result in an inflow <strong>of</strong> ST capital whichwould negate any increase in interest rates and push interest rates down. Hence, interestrates would not be a useful instrument for its monetary policy in Singapore due to itsopenness to capital flows.Singapore’s GDP is mainly contributed by export growth. Hence, keeping exchange ratescompetitive in the short term would reduce price <strong>of</strong> exports and assuming Ex +Em >1 i.e.Marshall Lerner condition holds, this would result in an intended increase in AD i.e.


economic growth, a fall in demand-deficient unemployment. This would also reducedemand-pull inflation. A gradual appreciation <strong>of</strong> the S$ would keep the price <strong>of</strong> imports lowand this would be important due to Singapore’s importdependence. This would keep cost <strong>of</strong> production <strong>of</strong> producers in Singapore low and hence,this keeps cost push inflation low.Knowledge, Application, Understanding and AnalysisL3L2L1Clearly supports with appropriate reasons why Singapore chooses exchangerate over interest rate as a monetary policy Answer shows depth.Some reasons were given on Singapore’s choice <strong>of</strong> monetary policy but maylack substantiation.Scanty description <strong>of</strong> reasons on Singapore’s choice <strong>of</strong> monetary policy butlacks depth.7 – 105 – 61 – 4(b) Assess the impact <strong>of</strong> the revaluation and the changes made to the currency’strading band on the Singapore economy.[15]A revaluation <strong>of</strong> the S$ refers to the increase in the value <strong>of</strong> the S$ brought about throughintervention by the government i.e. purchase <strong>of</strong> S$. The government sought a gradualappreciation <strong>of</strong> the S$ that would bring about internal and external effects on the Singaporeeconomy. The external effects <strong>of</strong> the revaluation <strong>of</strong> the S$ and the steepening and widening<strong>of</strong> the currency’s trading band include an improvement in the balance <strong>of</strong> trade and in turn,cause the balance <strong>of</strong> payments to improve.Price <strong>of</strong> imports will fall and this translates into lower costs <strong>of</strong> production i.e. lower importexpenditure as quantity demanded <strong>of</strong> imports will increase less than proportionately due toSingapore’s import dependence. Lower costs <strong>of</strong> production translates into lower prices <strong>of</strong>exports that will <strong>of</strong>fset the increase in price <strong>of</strong> exports due to the revaluation and hence,BOT improvesIn addition, the improvement in net exports would result in an increase in aggregate demand(AD) and hence, cause national income to increase via the multiplier effect and result in theincrease in actual economic growth.An additional internal effect <strong>of</strong> the revaluation would be demand-pull inflation. Demand-pullinflation would result when the economy is near full employment and hence, the increase inAD would cause an escalation in prices. Any increase in national income may be purelynominal and hence, there might then be a fall in real national income levels. In addition,demand deficient unemployment would decrease.


The steepening and widening <strong>of</strong> the currency’s trading band would allow Singapore to curbcost-push inflation in the long term to allow for an exchange rate appreciation to alleviate therising price <strong>of</strong> imports. An appreciation <strong>of</strong> exchange rates would allow Singapore torebalance its economy in the longer term due to increased economic challenges from theglobal economy and amidst rising prices <strong>of</strong> inputs.Knowledge, Application, Understanding and AnalysisL3L2L1For an answer that explores positive and negative impacts on the Singaporeeconomy. Answer exhibits depthFor an answer that tends to explore a balanced view <strong>of</strong> the impacts butanswer may show limited depth.For an answer that is limited in its substantiation. Answer tends to be onesided.9 – 116 – 81 – 5Allow up to 4 additional marks for EvaluationE2 For a judgment based on economic analysis / adequately substantiated 3 – 4E1For an unexplained judgment, or one that is not supported by economicanalysis.1 – 2(17) 2010 ACJC Prelim 2010 H2 Paper Q6 (only part a)<strong>Suggested</strong> answera. Explain the economic basis for specialization and free trade. [10]1. Why do countries trade?2. Economic concepts: Comparative Advantage & Terms <strong>of</strong> Trade.3. Benefits <strong>of</strong> Free trade.1. Why do countries trade?• Due to differences in demand and supply conditions, countries specialize. Explain theTheory <strong>of</strong> Comparative Advantage, opportunity cost and terms <strong>of</strong> trade to illustrate thegains from trade and the benefits from trade.• The Principle <strong>of</strong> Comparative Advantage gives a theoretical explanation to theappropriate pattern <strong>of</strong> specialisation based on efficiency among countries.Based on‘comparative advantage’ principle, the pattern <strong>of</strong> specialisation is based onopportunity cost. In spite <strong>of</strong> absolute advantage in all goods, the opportunity cost <strong>of</strong>each good is likely to differ and this country should produce/specialise in goods inwhich the opportunity cost is lowest.


Illustration:Assumptions:• Only two countries• Only two products• No transport costs between countries• Perfect factor mobility between the two product industries• Constant return to scale in productionShoes (units)Machines (units)UK 100 20Portugal 50 5Shoes (units)Machines (units)UK 0.2 machine 5 shoesPortugal 0.1 machine 10 shoesUK’s comparative advantage lies in machine production, where its comparativeadvantage is larger and its opportunity cost is lower. Portugal’s comparative advantagelies in production <strong>of</strong> shoes, where her opportunity cost is lower. Therefore, based on thepattern <strong>of</strong> comparative advantage, UK should specialize in machines and Portugal inshoes.The Terms <strong>of</strong> Trade is the ratio <strong>of</strong> export and import prices. It determines the extent <strong>of</strong>gains from trade.Shoes (units)Machine (units)UK 0.2 machine 5 shoesPortugal 0.1 machine 10 shoesAssuming the trade price between the two goods is 1 machine= 8 units <strong>of</strong> shoesFor both countries to mutually gain from free trade the trade price <strong>of</strong> a good must liewithin the opportunity cost limits <strong>of</strong> the two countries – in this case price <strong>of</strong> machine must


exceed 5 shoes and lower than 10 shoes. In terms <strong>of</strong> price <strong>of</strong> shoes 1 shoe = 0.125machine which is a mutually beneficial trade price for both countries. Terms <strong>of</strong> tradedetermines whether there is a basis for trade although comparative advantage exists.TOT also determines the extent <strong>of</strong> the gain <strong>of</strong> trade.2. Explain free trade and the benefits <strong>of</strong> free trade as a basis for its promotion.a. Free trade is a system in which goods, capital, and labor flow freely between nations,without barriers which could hinder the trade process.b. By encouraging nations to freely exchange goods, free trade can foster internationalcooperation and raise standard <strong>of</strong> living.c. Reducing tariff barriers leads to trade creation. Trade creation occurs whenconsumption switches from high cost producers to low cost producers.d. The removal <strong>of</strong> tariffs leads to lower prices for consumers and an increase in consumersurplus.e. Increased exports: as well as benefits for consumers importing goods, firms exportinggoods where the country has a comparative advantage will also see a big improvementin economic welfare. Lower tariffs on the country’s export will enable a higher quantity<strong>of</strong> export boosting jobs and economic growth.f. Economies <strong>of</strong> scale: if countries can specialize in certain goods they can benefit fromeconomies <strong>of</strong> scale and lower average costs, this is especially true in industries withhigh fixed costs or that require high levels <strong>of</strong> investment. The benefits <strong>of</strong> economiesscale will ultimately lead to lower prices for consumers.g. Increased competition: With more trade domestic firms will face more competition fromabroad therefore there will be more incentives to cut costs and increase efficiency.h. Trade is an engine <strong>of</strong> growth: World trade has increased significantly, causing this tobe one <strong>of</strong> the big contributors to economic growth.L3L2L1Clear explanation <strong>of</strong> all required concepts with links to the economic basis forspecialization and free trade.An explanation <strong>of</strong> comparative advantage without links to the basis forspecialization and free tradeOR explanation <strong>of</strong> only the benefits <strong>of</strong> Free trade.For an answer which has some basic correct facts such as unexplainedcomparative advantage and benefits <strong>of</strong> free trade.


(18) 2011 DHS Prelim H2 Paper Q1Singapore’s trade with major trading partners such as Malaysia, [the European Union],China, US, Indonesia, Japan and Hong Kong increased in 2010 due to improved globaleconomic conditions.Singstat, 14 March 2011a. Explain whether Singapore’s trade patterns agree with the predictions <strong>of</strong> economictheory. [12]b. Discuss the extent to which globalisation has benefitted small and open economiesmore than large and less open ones. [13]<strong>Suggested</strong> Answer SchemePart (a)Explain whether Singapore’s trade patterns agree with the predictions <strong>of</strong> economic theory.1. Introduction: a. Explain theory <strong>of</strong> comparative advantage as follow:A country has a comparative advantage in the production <strong>of</strong> a good if it can produce the good at a lower opportunity cost than its trading partner. The law <strong>of</strong> Comparative Advantage states that if countries specialise in the products in which they have comparative advantage, then trade will be mutually beneficial to all countries. Thesis2. Singapore’s patterns <strong>of</strong> trade agree with the predictions <strong>of</strong> economic theory. a. Singapore is endowed with capital and high-­‐skilled labour. b. Therefore Singapore produces capital and human capital intensive goodssuch integrated circuits, electronic hard disk drives, personal computers,financial services and pharmaceutical drugs with lower opportunity costsrelative to other countries and thus possesses a comparative advantage inproducing these goods and services. According to theory, Singapore shouldbe specialising in their production and export.This is true as we largelyexport such goods to our trading partners such as the US, and Indonesia.c. Explain that according to the theory <strong>of</strong> CA, Singapore should import landand low-skilled labour intensive G&S such as food, textiles, iron and steelfrom countries such as China and Malaysia who are endowed with land andlow-skilledlabour and thus possess a comparative advantage in producingthem as they incur relatively lower opportunity costs in doing so. This islargely true as a large amount <strong>of</strong> imports into Singapore comprises suchgoods and services.


Anti-thesis3. Singapore’s patterns <strong>of</strong> trade do not agree with the predictions <strong>of</strong> economic theory because the assumptions <strong>of</strong> CA do not hold: a. Theory <strong>of</strong> CA assumes no transport costs. i. In reality, there are transport costs and they are significant especially if they have to be air-­‐flown, have special transportation requirements or are bulky to transport. Hence, Singapore may import goods and services from neighbouring countries, despite them not having comparative advantage in producing certain goods and services. ii. China for example produces vegetables at a lower opportunity costs, given by lower prices, than Malaysia. However, most <strong>of</strong> Singapore’s vegetable imports are from Malaysia. b. Theory <strong>of</strong> CA assumes no trade barriers. In reality, many countries impose trade barriers on Singapore made goods and services. Hence, despite Singapore having CA in producing certain goods and services, the after tariff price may be relatively higher and hence Singapore goods and services become less price competitive in the global market. i. Singapore’s pattern <strong>of</strong> trade is also determined by the FTAs that ithas signed with its partners, such as ASEAN, China and US. SuchFTAs, which reduce protectionist measures such as tariffs andquotas, decreases prices <strong>of</strong> imports in and increases exports inforeign currency terms between FTA member states, hence allowingtrade creation and divergence.ii. Hence, Singapore trades more actively (in larger volumes) with countries that it has signed FTAs with, such as the US, ASEAN, China and Japan. This is because Singapore obtains preferential tariff rates when exporting to these countries. Hence, despite Singapore having less comparative advantage in producing a certain good, such as textiles, its relatively lower tariff rate when exporting to an FTA partner country means that its textiles will have CA relative to to a country who may produce textiles at a lower opportunity cost, but which becomes less price competitive in the US market because higher tariffs are imposed on them by the US viz Singapore. Hence, Singapore may also export goods and services that it does not have CA in. c. Theory <strong>of</strong> CA assumes perfect mobility <strong>of</strong> resources within the country; immobility between different countries: i. Without mobility <strong>of</strong> resources between countries, Singapore would not be able to produce labour intensive G&S with CA. However, mobility <strong>of</strong> labour between countries means that Singapore can actually allow foreign labour onto its shores and change its factor endowments such that it can produce certain goods and services, such as oil rigs and ship repairs, with comparative advantage. These, in turn, are presently significant amongst Singapore’s exports. d. Theory <strong>of</strong> CA assumes Constant Unit COP: i. The theory assumes constant returns to scale. In reality, diminishing returns will set in beyond a certain level <strong>of</strong> output, raising the unit costs <strong>of</strong> production. This in turn will lead to the disappearance <strong>of</strong> the Singapore’s comparative advantage in certain goods and services. This explains why


Singapore does not specialise completely in areas where it has CA but will produce some <strong>of</strong> the same types <strong>of</strong> goods that it imports, for example, telecommunications equipment. 4. Singapore’s patterns <strong>of</strong> trade do not agree with the predictions <strong>of</strong> economic theory because <strong>of</strong> strategic goods. a. Goods such as weapons, for example, are produced in Singapore rather than imported from countries with CA in producing weapons, such as Israel and the US, because they are considered strategic in nature and are essential for national defence. 5. Singapore’s patterns <strong>of</strong> trade do not agree with the predictions <strong>of</strong> economic theory because <strong>of</strong> intra-­‐industry trade driven by demand for a variety <strong>of</strong> goods and services. a. Singapore also imports goods that it has CA in, for example, pharmaceuticals and personal computers from USA and Europe. Such intra-­‐industry trade occurs as consumers prefer a variety <strong>of</strong> goods given their differing taste and preferences.Eg, although Singapore produces heart stents with CA, some consumers prefer the branded stents made by the US and French pharmaceutical giants. L3(9-12m)L2(6-8m)L1(1-5m)For a well-developed answer that explains and illustrates how Singapore’strade with its trading partners may or may not conform to the theory <strong>of</strong>comparative advantage:• Makes clear references to the theory <strong>of</strong> CA, using the “opportunity costs” nomenclature judiciously. • Accounts for both exports and imports • Explains a range <strong>of</strong> reasons why Singapore’s pattern <strong>of</strong> trade may not conform to the predictions <strong>of</strong> CA. • Answer is clearly applied in the context <strong>of</strong> Singapore. • Only minor misconceptions are made. For an under-developed answer that explains and illustrates how Singapore’strade with its trading partners may or may not conform to the theory <strong>of</strong>comparative advantage.• Max 6 for a one-sided answer.For an undeveloped answer demonstrates some knowledge <strong>of</strong> comparativeadvantage.• Answer contains severe misconceptions. • Answer is not applied to the context <strong>of</strong> Singapore. • Max 2 marks if theory <strong>of</strong> CA not used.


Part (b)Discuss whether globalisation has benefitted small and open economies like Singaporemore than large and open economies like the USA. [13]Scope requirements: Handles all 3 facets <strong>of</strong> globalization. Touches on micro and macrogoals.1. Introduction:ooooDefine globalization as the free movement <strong>of</strong> goods and services, labour, capital andtechnology.State that Singapore is a relatively small economy with domestic markets that arelimited in size and is an open economy where trade to GDP ratio exceeds 300%,amongst the highest in the world.State that other economies, such as the US and Indonesia, have large domesticmarkets and are less open as trade to GDP ratio is much lower, at about 28 and 53%respectively.Explain that globalization benefits economies when they help the countries achievetheir micro and macroeconomic goals:o Economic growth, low inflation, full employment, healthy BOP position,efficiency, equity, stabilityTheses: Small and open economies such as Singapore benefit more than large andless open economies:oSmall and open economies have larger dependence on external demand in the form <strong>of</strong>exports to drive economic growth:oGlobalisation, with the free movement <strong>of</strong> trade, helps expand markets for small and openeconomies, thus helping raise exports for economic growth.§With more markets open to them, export demand increases. This increases AD andbrings about economic growth, fall in unemployment and BOP CA improvements.[Illustrate using AD-AS model].oAn increase in exports would increase AD from AD0 to AD1 as shown in the diagrambelow. At the existing GPL P0, a shortage develops as firms exporting to foreignmarkets, such as those producing electronic valves and hard-disk drives, drawdown their inventory stocks in order to meet the shortage. They thus faceunplanned disinvestments. In the next production cycle, they will increaseproduction, hiring in the process more FOPs, including labour, thus increasingemployment. Holding labour force constant, this reduces the unemployment rate.


As output increases, real national income increases from Y0 to Y1, indicatingactual economic growth. At Y1, national income would have changed by multipletimes the initial increase in autonomous exports, to a factor <strong>of</strong> 1/marginalpropensity to withdraw which is the strength <strong>of</strong> the income multiplier. This occursbecause households, whose incomes increase when autonomous exportsincrease, will spend some <strong>of</strong> their increased incomes on domestically producedgoods and services, generating more income for other households. This occursuntil total withdrawals in the Singapore economy becomes equivalent to the initialincrease in exports.§Small economies thus benefit more than large and open ones as globalization helpsthem overcome their limited domestic market size to grow their economies. In theprocess, reduce demand deficient unemployment. BOP CA improves, ceterisparibus, because exports increase. This helps Singapore achieve a healthy BOP.§Expansion <strong>of</strong> demand for G&S by catering to world market also helps firms located insmall and open economies to reap IEOS for better productive efficiency [mayillustrate as a movement towards MES using LRAC curve]. This compares to firms incountries with large domestic demand where IEOS can be reaped without exportdemand, hence benefitting less from the increase in trade associated withglobalization.oSmall and open economies also benefit more than large and less open economies fromthe freer trade in goods and services as follow:oSmaller economies like Singapore have smaller domestic markets which generally lackthe breadth and depth <strong>of</strong> those found in large economies like the US. Without freer trade,residents in Singapore necessarily consume a smaller variety <strong>of</strong> goods and services than


their US counterparts. With trade however, they are likely to be able to increase thevariety <strong>of</strong> goods and services consumed more significantly and thus enjoy a largerimprovement in SOL.oSmall and open economies also benefit more than large and less open economies fromthe freer movement <strong>of</strong> capital arising from globalisation:oSmaller economies have smaller domestic markets which lack depth and breadth. Theyhence benefit more from the free movement <strong>of</strong> capital as they are more reliant on foreigncompanies for technological transfer, since their technology base is likely limited due tothe small size <strong>of</strong> their economy, perhaps because <strong>of</strong> smaller firm numbers and smallerfirm sizes (which limits the size <strong>of</strong> accumulated pr<strong>of</strong>its that can be used for R&D).§With technological transfer, productivity in small and open economies like Singaporeis raised à raises potential growth and lowers unit COP à non-inflationary economicgrowth and falling unemployment.oSmaller countries would also benefit more significantly from FDI as small size <strong>of</strong>economy also implies limited capital available domestically. Eg, Singapore has a muchsmaller capital base than the US or Indonesia. FDI inflows help increase domestic fixedcapital formation à promotes capital widening and deepening àraises productivity àraises potential growth and lowers unit COP à non-inflationary economic growth andfalling unemployment.oFDI Improves BOP Financial and Capital Accounts.oSmall and open economies also benefit more than large and less open economies fromthe freer movement <strong>of</strong> labour arising from globalization:oSmall labour force limits potential for economic growth, a problem that is faced by largereconomies to a smaller extent.§Free movement <strong>of</strong> labour resulting from globalisation, both high skilled and lowskilled, augments the quantity and quality <strong>of</strong> labourthus helping overcomeconstraints. This compares to large countries where larger pool <strong>of</strong> talent alreadyensures a larger productive capacity.§As labour force increases through high and low skilled immigration, increase inquantity and quality <strong>of</strong> labour force raises productive capacity à raises potentialgrowth and lowers unit COP à brings about non-inflationary economic growth andfalling unemployment.Anti-­‐thesis: Small and open economies may be more vulnerable to negative effects <strong>of</strong> globalisation than large and less open economies:


oooHigh level <strong>of</strong> integration increases vulnerability <strong>of</strong> smaller and more open economiesby exposing them more significantly to external demand conditions-­‐ exacerbates economic problems: Xd and FDI as a proportion <strong>of</strong> AD is much higher for smaller and more open economies à economy becomes more vulnerable to external environment à economic activity becomes more volatile due to influence <strong>of</strong> trading partners’ economic situations on domestic economy à smaller and more open economies less stable than large and less open ones, who depend more on domestic demand for economic growth. Free movement <strong>of</strong> labour arising from globalisation has significant impact inequity on the domestic economy, especially on wages. §§Influx <strong>of</strong> foreign workers increases supply <strong>of</strong> workers than reducing wages. This is especially important because foreign workers tend to form a larger proportion <strong>of</strong> workers in small and open economies such as Hong Kong and Singapore. Wages <strong>of</strong> low skilled workers may become especially depressed because influx <strong>of</strong> foreign low-­‐skilled worker, in the Singapore example, increase supply more than demand, causing wages to decrease. Skilled workers, on the other hand, find their wages increasing as the growth in demand for them outstrips supply. §ooo§§This worsens inequity more significantly for small and open economies who are more reliant on foreign workers. Remittance <strong>of</strong> wages by foreign workers may also worsen BOP CA through outflows via the income balance/ invisible balance in the current account. Larger dependence on FDI flows may worsen BOP CA <strong>of</strong> small and open economies more significantly: Repatriation <strong>of</strong> pr<strong>of</strong>its may worsen BOP CA through outflows via the income balance/ invisible balance in the current account. Might be a problem for countries managing their exchange rates as this reduces the size <strong>of</strong> trade surpluses and thus reduces the rate <strong>of</strong> accumulation <strong>of</strong> foreign reserves à less ability to manage exchange rates thus worsening economic instability. Currency might depreciate. For small and open economies that are highly reliant on imports, this may lead to import cost push inflation as depreciation makes foreign intermediate and final goods and services more expensive in domestic currency terms. [Support using AD-­‐AS diagram]. Anti-thesis: Large and less open economies may benefit more than small and openeconomies:


Domestic firms are larger and likely more productive efficient due to IEOS. Free movement <strong>of</strong> goods and services makes available more export markets to them. Their firms are more able to penetrate markets <strong>of</strong> small and open economies more effectively for greater pr<strong>of</strong>it, which when remitted improves BOPCA. Eg, Many MNCs are from countries that are large and less open as the large domestic market enables their expansion. Evaluation:• Small and open economies have more to gain because globalisation can help them circumvent the limitations imposed by their small domestic markets in achieving their macroeconomic objectives. This is because globalisation enables them to trade with the world, attract foreign workers and FDI. Such limitations are faced by large and less open economies to a smaller extent. • Given the problems brought about by globalisation, small open economies need to be able to mitigate its costs in order to ensure that they benefit: o Need to assert greater control over their external environment – through exchange rate policies. This requires them to build up sizeable reserves to manage their exchange rate for better stability. o Signing FTAs with a larger number <strong>of</strong> countries to reduce the risk <strong>of</strong> external fluctuations. L3 For a well-developed answer that explains and illustrates the benefits andcosts <strong>of</strong> globalisation to small and open and large and compares them with(7-9m)less open economies using clear economic frameworks.L2(5-6m)L1(1-4m)E2(3-4m)E1(1-2m)• Answer contains a range <strong>of</strong> arguments spanning all 3 facets <strong>of</strong> globalisation: Free movement <strong>of</strong> (i) goods and services (ii) labour (iii) capital and technology. • Contains only minor misconceptions. For an under-developed answer that explains and illustrates the benefits andcosts <strong>of</strong> globalisation to small and open and large and less open economies.• Max 5m if answer does not compare the benefits accruable to small and open economies viz big and less open economies. • Max 5m for an unbalanced answer. • Max 5m for an answer that touches only on the trade facet <strong>of</strong> globalisation. • Max 5m if no clear economic framework is presented. For an undeveloped answer that merely shows knowledge <strong>of</strong> globalisationand/or its benefits and costs.• Answer contains severe misconceptions. Economic judgement with justification using clear economic framework.Candidates need to present their own opinion backed up by economictheories.Economic judgement with justification but lacking in economic reasoning. Amere presentation <strong>of</strong> personal opinions without substantiation usingeconomic theories.


2010 IJC Prelim 2010 H2 Paper Q6(a) Explain how immobility <strong>of</strong> factors <strong>of</strong> production can affect trade between countries.[10](b)(c) Assess the extent to which supply factors, rather than demand factors, are the majordeterminants <strong>of</strong> trade patterns between Singapore and the rest <strong>of</strong> the world.[15]<strong>Suggested</strong> Answer Scheme:(a) Explain how immobility <strong>of</strong> factors <strong>of</strong> production can affect trade betweencountries. [10]IntroductionImmobility <strong>of</strong> factors <strong>of</strong> production à factors <strong>of</strong> production are not able to move from oneuse to another, one industry to another or from one location to another with easeImmobility <strong>of</strong> factors <strong>of</strong> production might affect willingness <strong>of</strong> countries to trade as it affectthe gains from trade1. Immobility <strong>of</strong> factors <strong>of</strong> production between countriesTheory <strong>of</strong> Comparative Advantage is premised on factors <strong>of</strong> production being immobilebetween nationsExplanation <strong>of</strong> theory illustrating how countries can gain from specialization and trade asthere are differences in opportunity cost between countries if factors are immobile2. Immobility <strong>of</strong> factors <strong>of</strong> production within a countriesTheory <strong>of</strong> Comparative Advantage assumes that there is mobility <strong>of</strong> factors <strong>of</strong> productionbetween industries to facilitate specialisation by countries in areas which they have acomparative advantage inImmobility <strong>of</strong> factors <strong>of</strong> production implies comparative advantage that countries possess arenot translated into gains from trade, or the gains are significantly diminished


(b) Assess the extent to which supply factors, rather than demand factors, are themajor determinants <strong>of</strong> the pattern <strong>of</strong> trade between Singapore and the rest <strong>of</strong>the world. [15]Introduction§ Pattern <strong>of</strong> trade - a country’s volume <strong>of</strong> trade and the type <strong>of</strong> exports and imports§ Major trading partners <strong>of</strong> Singapore§ Exports and imports <strong>of</strong> Singapore 1. Supply factors• Comparative advantage Lower opportunity cost <strong>of</strong> producing certain goods and services relative to other countries à countries should specialise in the production <strong>of</strong> those goods they are more efficient in à lower unit cost <strong>of</strong> production Countries should import goods that they do not have comparative in as opportunity costs <strong>of</strong> producing these goods within country are higher • Factor endowment Relative abundance <strong>of</strong> specific factors <strong>of</strong> production à cost <strong>of</strong> the using these factors is lower • Technological advancement and innovation Lowers production cost and improve productive efficiency à lower unit cost • Factor productivity More productive à returns on factor higher à lower unit costL3L2Knowledge, Application, Understanding and AnalysisA well developed analysis on result <strong>of</strong> immobility <strong>of</strong> factors <strong>of</strong> production ontrade.Able to explain to some effect <strong>of</strong> immobility <strong>of</strong> factors <strong>of</strong> production on trade, butnot well developed.L1 Smattering <strong>of</strong> valid points on immobility <strong>of</strong> factors <strong>of</strong> production or trade. 1-47-105-62. Demand factors• Incomes Emerging economies, e.g. China and India


à growing affluence àrise in demand for many goods and servicesàrapid growth fuelling demand for intermediate goods for productionRecession à fall in income à fall in demand for goods and services• Tastes and preferences Changing lifestyles, technological innovations and improved product designà growing demand for high-end products• Population changes Growth in population à rise in demand for goods and services.3. Assessment <strong>of</strong> importance <strong>of</strong> supply and demand factors Both supply and demand factors are important in explaining Singapore’s trade pattern Importance <strong>of</strong> factor depends on which aspect <strong>of</strong> trade pattern is the focus e.g. Supply factors might be important in determine type <strong>of</strong> goods to specialise in, demand factors might affect which market goods are exported to. Knowledge, Application, Understanding and AnalysisL3Candidates are able to provide well-developed analysis <strong>of</strong> the demandand supply factors determining the trade pattern between Singapore andthe rest <strong>of</strong> the world with the use <strong>of</strong> relevant examples.8-11L2Candidates are able to provide some analysis <strong>of</strong> the demand and supplyfactors determining the trade pattern between Singapore and the rest <strong>of</strong>the world. Application to the context might be limited.5-7L1 For an answer that provides a mere smattering <strong>of</strong> valid points. 1-4E2 Insightful evaluative comments 3-4E1 Limited evaluation without justification. 1-2

Hooray! Your file is uploaded and ready to be published.

Saved successfully!

Ooh no, something went wrong!